7 Subject Matter Jurisdiction 7 Subject Matter Jurisdiction

7.1 Subject Matter Jurisdiction Wordcloud 7.1 Subject Matter Jurisdiction Wordcloud

7.2 Introduction to Subject Matter Jurisdiction 7.2 Introduction to Subject Matter Jurisdiction

7.2.1 Overview - The Constitutional Basis 7.2.1 Overview - The Constitutional Basis

     Federal subject matter jurisdiction involves a fundamental aspect of American constitutional governance – the federal government is a government of limited powers. Put differently, the federal government in the United States can only exercise those powers granted to it under the Constitution. While in many areas constitutional amendments such as the 14th Amendment and judicial interpretations have expanded federal power so as to make it seem almost unlimited, where federal courts are concerned the limitations on federal power remain very much alive.

     The U.S. Constitution sets forth the judicial function of the federal government in Article 3. Article 3, Section 2, lists the kinds of cases the federal government may entertain in its courts. No cases can be heard by federal courts that are not included in the categories identified in Article 3, Section 2.

     Unlike the Constitutional limitations on judicial power that we have looked at previously (personal jurisdiction and notice), subject matter jurisdiction is not waivable by the parties. The parties cannot consent to subject matter jurisdiction when it is absent or doubtful. Put differently, the interest here is in limiting the reach of the federal government to the Constitutional scope, and it is not within the power of any litigant, even the government, to extend that power through consent. If at any stage of litigation a lack of subject matter jurisdiction is recognized, the case must be dismissed.

     We will not examine every type of case that Section 2 authorizes for federal court. We will limit our review to federal question cases, and cases that involve diversity or alienage jurisdiction. In addition, we will look at supplemental jurisdiction, which is when the court hearing a case that has a basis for subject matter jurisdiction over some counts can exercise jurisdiction over aspects of the case or controversy that might not on their own qualify for federal jurisdiction, and removal processes, which allow cases to be moved from state courts to federal courts in certain situations.

     As we have seen before, subject matter jurisdiction involves Constitutional and statutory layers. In addition to the Constitutional grant, there must be an enabling statute which grants and sometimes limits the power of federal courts to hear cases that fall within the Article 3, Section 2 grant.

     One term that might prove confusing in this context is 'general jurisdiction.' You remember that term from our discussion of personal jurisdiction. Here, it means something very different. Just as federal courts are courts of 'limited jurisdiction,' state court systems are courts of 'general jurisdiction' in the sense that their reach is not limited by a Constitutional grant. Unless a statute or the Constitution takes a case away from a state court, the state courts have the power to resolve the case. 

     To further complicate things, while state court systems are courts of general jurisdiction, not every court in a state court system will be one of general jurisdiction. Some courts in the state system have a limited purpose - hearing divorce cases or traffic violations, for example. Often, the main state trial courts - sometimes called Circuit Courts or Superior Courts or Courts of Common Pleas, although to be extra confusing New York calls its trial courts Supreme Courts - are referred to as courts of general jurisdiction because they are empowered under state law to hear a full range of cases.

     Don't let the terms lead you astray. Keep in mind the basic principle - federal courts are limited in their power, and can only hear certain kinds of cases. State courts, on the other hand, can hear both federal and state cases. In fact, they can hear any case that is not reserved to the exclusive jurisdiction of the federal courts. There are a few such areas, but they are the exception. (Exclusive areas of federal jurisdiction are copyright cases, admiralty cases, lawsuits involving the military, immigration laws, and bankruptcy proceedings.)

     As a practical matter, this means that state and federal courts often have what is called concurrent jurisdiction. Their jurisdictions overlap, and many cases can be heard in either federal court or state court. State law cases, as we will see in this section, can be heard in federal court if diversity or alienage jurisdiction exists. In addition, even the claim on its own does not meet the requirements of diversity or alienage jurisdiction, so-called 'supplemental jurisdiction' allows claims that arise in the same case or controversy to be in federal court alongside claims for which there is a federal jurisdictional basis. In other cases, even though a federal jurisdictional basis exists, the parties may simply prefer for any number of reasons to proceed in state court on either state or federal claims.

     This means that litigants often have the ability to select a forum. Sometimes, pejoratively, this is called forum shopping. Whether you view it as a good thing, a bad thing, or just an inevitable thing in a system that allows multiple courts to hear the same case, it is a feature of the US system. Understanding federal subject matter jurisdiction in all its complexity is an essential step to knowing when and how forums can be selected.

     In the section that follows, we will explore all that in more detail. For the most part, you will find this complicated with lots of rules, but not hard to understand. Keep in mind that forum selection can be an important and sometimes outcome-determining tool in the litigator's toolbox, so this is worth understanding well.

7.2.2 Article III, Section 2 7.2.2 Article III, Section 2

The judicial Power shall extend to all Cases, in Law and Equity, arising under this Constitution, the Laws of the United States, and Treaties made, or which shall be made, under their Authority; to all Cases affecting Ambassadors, other public Ministers and Consuls; to all Cases of admiralty and maritime Jurisdiction; to Controversies to which the United States shall be a Party; to Controversies between two or more States; between a State and Citizens of another State; between Citizens of different States; between Citizens of the same State claiming Lands under Grants of different States, and between a State, or the Citizens thereof, and foreign States, Citizens or Subjects.

7.2.3 Capron v. Van Noorden, 6 U.S. (2 Cranch) 126 (1804) 7.2.3 Capron v. Van Noorden, 6 U.S. (2 Cranch) 126 (1804)

                                                          Feb Term 1804

                                                                Opinion

     A plaintiff may assign for error the want of jurisdiction in that court to which he has chosen to resort.

     A party may take advantage of an error in his favor, if it be an error of the Court.

     The Courts of the U.S. have not jurisdiction unless the record shews that the parties are citizens of different states, or that one is an alien, &c.

     Error to the Circuit Court of North Carolina. The proceedings stated Van Noorden to be late of Pitt county, but did not allege Capron, the plaintiff, to be an alien, nor a citizen of any state, nor the place of his residence.

     Upon the general issue, in an action of trespass on the case, a verdict was found for the defendant, Van Noorden, upon which judgment was rendered.

     The writ of Error was sued out by Capron, the plaintiff below, who assigned for error, among other things, first “That the circuit court aforesaid is a court of limited jurisdiction, and that by the record aforesaid it doth not appear, as it ought to have done, that either the said George Capron, or the said Hadrianus Van Noorden was an alien at the time of the commencement of said suit, or at any other time, or that one of the said parties was at that or any other time, a citizen of the state of North Carolina where the suit was brought, and the other a citizen of another state; or that they the said George and Hadrianus were for any cause whatever, persons within the jurisdiction of the said court, and capable of suing and being sued there.”

     And secondly, “That by the record aforesaid it manifestly appeareth that the said Circuit Court had not any jurisdiction of the cause aforesaid, nor ought to have held plea thereof, or given judgment therein, but ought to have dismissed the same, whereas the said Court hath proceeded to final judgment therein.”

     Harper, for the plaintiff in error, stated the only question to be whether the plaintiff had a right to assign for error, the want of jurisdiction in that Court to which he had chosen to resort.

     It is true, as a general rule, that a man cannot reverse a judgment for error in process or delay, unless he can shew that the error was to his disadvantage; but it is also a rule, that he may reverse a judgment for an error of the Court, even though it be for his advantage. As if a verdict be found for the debt, damages, and costs; and the judgment be only for the debt and damages, the defendant may assign for error that the judgment was not also for costs, although the error is for his advantage.

     Here it was the duty of the Court to see that they had jurisdiction, for the consent of parties could not give it.

     It is therefore an error of the Court, and the plaintiff has a right to take advantage of it. 2 Bac.Ab.Tit. Error. (K.4.)—8 Co. 59.(a) Beecher's case.—1 Roll.Ab. 759—Moor 692.—1 Lev. 289. Bernard v. Bernard.

     The defendant in error did not appear, but the citation having been duly served, the judgment was reversed.

7.2.4 Notes on Capron v. Van Noorden 7.2.4 Notes on Capron v. Van Noorden

     1. Capron v. Van Noorden. The antiquated language and differences in the judicial system in Capron v. Van Noorden can make it a bit difficult to get started with the case. The facts are quite simple, though: the plaintiff filed a case in federal court, making inadequate jurisdictional allegations; the trial court accepted the case and ruled for the defendant; the plaintiff then sought to have the suit dismissed on grounds that there was never federal subject matter jurisdiction. Even though the plaintiff contributed mightily to the situation, in the end it was the duty of the court to establish that federal subject matter jurisdiction existed. As the allegations were inadequate, there was no federal subject matter jurisdiction, and the case had to be dismissed. Because the court never had subject matter jurisdiction, the proceedings in the trial court were void and of no effect.

     2. Basic Principles.  Some very basic principles are present in Capron v. Van Noorden. First, a federal court must have subject matter jurisdiction or it cannot hear a case - even if the parties wish it to. Second, the basis for jurisdiction must appear in the pleadings or the record; the court cannot assume it or rely upon conclusory allegations. Third, it is the responsibility of the court to establish subject matter jurisdiction. It must, own its own, sua sponte, take the steps necessary to determine jurisdiction and to dismiss the case if jurisdiction is not shown. The parties can raise the issue at any time, and the court should address it on its own if the parties fail to note the problem.

7.2.5 Challenging Subject Matter Jurisdiction 7.2.5 Challenging Subject Matter Jurisdiction

     It should be clear from Capron that a lack of subject matter jurisdiction can be made at any time during the original litigation - even when the case is on appeal at the U.S. Supreme Court. Indeed, if no litigant raises the issue, the court can and should examine the issue on its own. As subject matter jurisdiction cannot be waived, there is no special procedure to avoid waiver. At any point in the litigation, the case must be dismissed if there is no subject matter jurisdiction. You will see examples of that as we go forward in this section.

     One might wonder if, on occasion, litigants might be tempted to trick the other side by litigating a case where they know that subject matter jurisdiction does not exist. If they win, and the other side does not recognize the jurisdictional defect, they stay silent and keep the benefit of the win; if they lose, they can assert the lack of subject matter jurisdiction, the case is dismissed, and they are free to try again. What if defendants wait until after a statute of limitations has passed before revealing the lack of subject matter jurisdiction?

     Something like this arose in Wojan v. General Motors Corp., 851 F.2d 969 (7th Cir.1988). In Wojan, the plaintiff, a resident of Michigan, filed suit in Illinois, stating in its jurisdictional statement that General Motors was incorporated in Delaware and licensed to do business as an out-of-state corporation in Illinois. It did not mention where General Motors had its principal place of business, which quite famously is in Michigan.  In its answer, General Motors admitted that it was incorporated in Delaware and expressed a lack of knowledge as to plaintiff's residence. Neither the judge nor the plaintiff apparently noticed that General Motors said nothing about its principal place of business, which as we will see shortly is relevant to establishing diversity jurisdiction for corporations. General Motors later, in its answer, admitted the existence of diversity jurisdiction, notwithstanding that few U.S. corporations are more firmly and widely associated with their principal place of business than General Motors is with Michigan and the automobile manufacturing center of Detroit, also known as Motor City or Motown.

     After five and a half years of litigation, and apparently having had a long-deferred moment of recognition when a new associate on the case asked why there was subject matter jurisdiction, General Motors finally asserted that there was no subject matter jurisdiction, and the case was dismissed without prejudice allowing, in theory, Wojan to refile in state court. Without researching what statute of limitations applied to such a case in Illinois in the 1980s, it seems highly likely that the applicable statute of limitations had passed when the jurisdictional defect was raised with the court almost eight years after the original accident, and so Wojan's litigation was likely at an end except for a possible malpractice claim against her lawyer.

     Wojan sought Rule 11 sanctions against General Motors for admitting subject matter jurisdiction when it should have denied jurisdiction. While the Seventh Circuit noted that the General Motors lawyers (and the trial judge) were not without fault, it held that it was the duty of the plaintiff to establish the basis for subject matter jurisdiction at the outset, and that awaiting General Motors' response on a jurisdictional allegation that a moment's thought would have shown defective was not reasonable. One can imagine situations where deliberate and cunning failure to alert the court to a lack of subject matter jurisdiction could lead to ethical concerns or sanctions (in Wojan the defense lawyers seem to have been more asleep than intentionally deceptive), but it is clear that in every case, notwithstanding the equities, if subject matter jurisdiction is lacking the court has no choice and has to dismiss the case.

     What happens after a case is over? Can an apparent lack of subject matter jurisdiction be used to set aside a judgment as void upon a collateral attack as can be done when personal jurisdiction is lacking?

     The answer turns out to be a bit complicated. When the case has been adversely litigated between the parties in federal court, it seems fairly clear that the judgment cannot be challenged collaterally on the grounds that there was no subject matter jurisdiction. In Chicot County Drainage Distr. v. Baxter State Bank, 308 U.S. 371 (1940), a federal statute apparently in effect at the time the case was tried provided a basis for federal subject matter jurisdiction. No one, therefore, questioned subject matter jurisdiction and judgment was entered. When that federal statute was found to be unconstitutional, removing the jurisdictional basis, an effort was made to collaterally attack the judgment. The Court held that the subject matter jurisdiction could not be collaterally attacked but should have been litigated in the first proceeding.

     The situation gets a bit murky when the first case involves a default judgment (and a bit murkier because people differ on whether to characterize Chicot as a default case, given that it was actively litigated but where the parties bringing the collateral challenge had not actually appeared in the first litigation). Remember that, whether anyone actively litigates the case or not, the federal court should always determine that it has subject matter jurisdiction, so in theory there should be a determination as to subject matter jurisdiction in every case. Perhaps as a result of this, while there is some academic support for being open to collateral attacks in some situations (see Restatement (Second) of Judgments § 65) courts in general do not set aside judgments based on collateral attacks on subject matter jurisdiction.

7.3 Diversity and Alienage As a Basis For Subject Matter Jurisdiction 7.3 Diversity and Alienage As a Basis For Subject Matter Jurisdiction

Please note that in the section that follows we are talking about cases that arise under state law. What claims, based on state law, nonetheless have a right to be decided in federal court? (Remember that federal law can be created by the federal legislature and to a limited extent by federal courts. Parallel to that state legislatures and state courts can create law that is binding within their states.) The jurisdictional answer with regard to original (versus supplemental) jurisdiction has to do with where the parties are from. The story begins with the diversity and alienage clauses of Article III, Section 2, but the implementation of those grants of constitutional authority by the following statute also matters.

7.3.1 28 U.S. Code § 1332. Diversity of citizenship; amount in controversy; costs 7.3.1 28 U.S. Code § 1332. Diversity of citizenship; amount in controversy; costs

(a) The district courts shall have original jurisdiction of all civil actions where the matter in controversy exceeds the sum or value of $75,000, exclusive of interest and costs, and is between—

     (1) citizens of different States;

     (2) citizens of a State and citizens or subjects of a foreign state, except that the district courts shall not have original jurisdiction under this subsection of an action between citizens of a State and citizens or subjects of a foreign state who are lawfully admitted for permanent residence in the United States and are domiciled in the same State;

     (3) citizens of different States and in which citizens or subjects of a foreign state are additional parties; and

     (4) a foreign state, defined in section 1603(a) of this title, as plaintiff and citizens of a State or of different States.

(b) Except when express provision therefor is otherwise made in a statute of the United States, where the plaintiff who files the case originally in the Federal courts is finally adjudged to be entitled to recover less than the sum or value of $75,000, computed without regard to any setoff or counterclaim to which the defendant may be adjudged to be entitled, and exclusive of interest and costs, the district court may deny costs to the plaintiff and, in addition, may impose costs on the plaintiff.

(c) For the purposes of this section and section 1441 of this title—

     (1) a corporation shall be deemed to be a citizen of every State and foreign state by which it has been incorporated and of the State or foreign state where it has its principal place of business, except that in any direct action against the insurer of a policy or contract of liability insurance, whether incorporated or unincorporated, to which action the insured is not joined as a party-defendant, such insurer shall be deemed a citizen of—

          (A) every State and foreign state of which the insured is a citizen;

          (B) every State and foreign state by which the insurer has been incorporated; and

          (C) the State or foreign state where the insurer has its principal place of business; and

     (2) the legal representative of the estate of a decedent shall be deemed to be a citizen only of the same State as the decedent, and the legal representative of an infant or incompetent shall be deemed to be a citizen only of the same State as the infant or incompetent.

(d) [Omitted for Now]

7.3.2 Determining Domicile 7.3.2 Determining Domicile

7.3.2.1 Mas v. Perry 7.3.2.1 Mas v. Perry

Jean Paul MAS and Judy Mas, Plaintiffs-Appellees, v. Oliver H. PERRY, Defendant-Appellant.

No. 73-3008

Summary Calendar.*

United States Court of Appeals, Fifth Circuit.

Feb. 22, 1974.

Rehearing and Rehearing En Banc Denied April 3, 1974.

*1398Sylvia Roberts, John L. Avant, Baton Rouge, La., for defendant-appellant.

Dennis R. Whalen, Baton Rouge, La., for plaintiffs-appellees.

Before WISDOM, AINSWORTH and CLARK, Circuit Judges.

AINSWORTH, Circuit Judge:

This case presents questions pertaining to federal diversity jurisdiction under 28 U.S.C. § 1332, which, pursuant to article III, section II of the Constitution, provides for original jurisdiction in federal district courts of all civil actions that are between, inter alia, citizens of different States or citizens of a State and citizens of foreign states and in which the amount in controversy is more than $10,000.

Appellees Jean Paul Mas, a citizen of France, and Judy Mas were married at her home in Jackson, Mississippi. Prior to their marriage, Mr. and Mrs. Mas were graduate assistants, pursuing coursework as well as performing teaching duties, for approximately nine months and one year, respectively, at Louisiana State University in Baton Rouge, Louisiana. Shortly after their marriage, they returned to Baton Rouge to resume their duties as graduate assistants at LSU. They remained in Baton Rouge for approximately two more years, after which they moved to Park Ridge, Illinois. At the time of the trial in this case, it was their intention to return to Baton Rouge while Mr. Mas finished his studies for the degree of Doctor of Philosophy. Mr. and Mrs. Mas were undecided as to where they would reside after that.

Upon their return to Baton Rouge after their marriage, appellees rented an apartment from appellant Oliver H. Perry, a citizen of Louisiana. This appeal arises from a final judgment entered on a jury verdict awarding $5,000 to Mr. Mas and $15,000 to Mrs. Mas for damages incurred by them as a result of the discovery that their bedroom and bathroom contained “two-way” mirrors and that they had been watched through them by the appellant during three of the first four months of their marriage.

At the close of the appellees’ case at trial, appellant made an oral motion to dismiss for lack of jurisdiction.1 The motion was denied by the district court. Before this Court, appellant challenges the final judgment below solely on jurisdictional grounds, contending that appel-lees failed to prove diversity of citizenship among the parties and that the requisite jurisdictional amount is lacking with respect to Mr. Mas. Finding no merit to these contentions, we affirm. Under section 1332(a)(2), the federal judicial power extends to the claim of Mr. Mas, a citizen of France, against the appellant, a citizen of Louisiana. Since we conclude that Mrs. Mas is a citizen of Mississippi for diversity purposes, the district court also properly had jurisdiction under section 1332(a) (1) of her claim.

It has long been the general rule that complete diversity of parties is *1399required in order that diversity jurisdiction obtain; that is, no party on one side may be a citizen of the same State as any party on the other side. Strawbridge v. Curtiss, 7 U.S. (3 Cranch) 267, 2 L.Ed. 435 (1806); see eases cited in 1 W. Barron & A. Holtzoff, Federal Practice and Procedure § 26, at 145 n. 95 (Wright ed. 1960). This determination of one’s State citizenship for diversity purposes is controlled by federal law, not by the law of any State. 1 J. Moore, Moore’s Federal Practice j[ 0.74 [1], at 707.1 (1972). As is the case in other areas of federal jurisdiction, the diverse citizenship among adverse parties must be present at the time the complaint is filed. Mullen v. Torrance, 22 U.S. (9 Wheat.) 537, 539, 6 L.Ed. 154, 155 (1824); Slaughter v. Toye Bros. Yellow Cab Co., 5 Cir., 1966, 359 F.2d 954, 956. Jurisdiction is unaffected by subsequent changes in the citizenship of the parties. Morgan’s Heirs v. Morgan, 15 U.S. (2 Wheat.) 290, 297, 4 L.Ed. 242, 244 (1817); Clarke v. Mathewson, 37 U.S. (12 Pet.) 164, 171, 9 L.Ed. 1041, 1044 (1838); Smith v. Sperling, 354 U.S. 91, 93 n. 1, 77 S.Ct. 1112, 1113 n. 1, 1 L.Ed. 2d 1205 (1957). The burden of pleading the diverse citizenship is upon the party invoking federal jurisdiction, see Cameron v. Hodges, 127 U.S. 322, 8 S.Ct. 1154, 32 L.Ed. 132 (1888); and if the diversity jurisdiction is properly challenged, that party also bears the burden of proof, McNutt v. General Motors Acceptance Corp., 298 U.S. 178, 56 S.Ct. 780, 80 L.Ed. 1135 (1936); Welsh v. American Surety Co. of New York, 5 Cir., 1951, 186 F.2d 16, 17.

To be a citizen of a State within the meaning of section 1332, a natural person must be both a citizen of the United States, see Sun Printing & Publishing Association v. Edwards, 194 U.S. 377, 383, 24 S.Ct. 696, 698, 48 L.Ed. 1027 (1904); U.S.Const. Amend. XIV, § 1, and a domiciliary of that State. See Williamson v. Osenton, 232 U.S. 619, 624, 34 S.Ct. 442, 58 L.Ed. 758 (1914); Stine v. Moore, 5 Cir., 1954, 213 F.2d 446, 448. For diversity purposes, citizenship means domicile; mere residence in the State is not sufficient. See Wolfe v. Hartford Life & Annuity Ins. Co., 148 U.S. 389, 13 S.Ct. 602, 37 L.Ed. 493 (1893); Stine v. Moore, 5 Cir., 1954, 213 F.2d 446, 448.

A person’s domicile is the place of “his true, fixed, permanent home and principal establishment, and to which he has the intention of returning whenever he is absent therefrom . ” Stine v. Moore, 5 Cir., 1954, 213 F.2d 446, 448. A change of domicile may be effected only by a combination of two elements: (a) taking up residence in a different domicile with (b) the intention to remain there. Mitchell v. United States, 88 U.S. (21 Wall.) 350, 22 L.Ed. 584 (1875); Sun Printing & Publishing Association v. Edwards, 194 U.S. 377, 24 S.Ct. 696, 48 L.Ed. 1027 (1904).

It is clear that at the time of her marriage, Mrs. Mas was a domiciliary of the State of Mississippi. While it is generally the case that the domicile of the wife — and, consequently, her State citizenship for purposes of diversity jurisdiction — is deemed to be that of her husband, 1 J. Moore, Moore’s Federal Practice ¶ 0.74 [6. — 1], at 708.51 (1972), we find no precedent for extending this concept to the situation here, in which the husband is a citizen of a foreign state but resides in the United States. Indeed, such a fiction would work absurd results on the facts before us. If Mr. Mas were considered a domiciliary of France — as he would be since he had lived in Louisiana as a student-teaching assistant prior to filing this suit, see Chicago & Northwestern Railway Co. v. Ohle, 117 U.S. 123, 6 S.Ct. 632, 29 L.Ed. 837 (1886); Bell v. Milsak, W.D.La., 1952, 106 F.Supp. 219— then Mrs. Mas would also be deemed a domiciliary, and thus, fictionally at least, a citizen of France. She would not be a citizen of any State and could *1400not sue in a federal court on that basis; nor could she invoke the alienage jurisdiction to bring her claim in federal court, since she is not an alien. See C. Wright, Federal Courts 80 (1970). On the other hand, if Mrs. Mas’s domicile were Louisiana, she would become a Louisiana citizen for diversity purposes and could not bring suit with her husband against appellant, also a Louisiana citizen, on the basis of diversity jurisdiction. These are curious results under a rule arising from the theoretical identity of person and interest of the married couple. See Linscott v. Linscott, S.D.Iowa, 1951, 98 F.Supp. 802, 804; Juneau v. Juneau, 227 La. 921, 80 So.2d 864, 867 (1954).

An American woman is not deemed to have lost her United States citizenship solely by reason of her marriage to an alien. 8 U.S.C. § 1489. Similarly, we conclude that for diversity purposes a woman does not have her domicile or State citizenship changed solely by reason of her marriage to an alien.

Mrs. Mas’s Mississippi domicile was disturbed neither by her year in Louisiana prior to her marriage nor as a result of the time she and her husband spent at LSU after their marriage, since for both periods she was a graduate assistant at LSU. See Chicago & Northwestern Railway Co. v. Ohle, 117 U.S. 123, 6 S.Ct. 632, 29 L.Ed. 837 (1886). Though she testified that after her marriage she had no intention of returning to her parents' home in Mississippi, Mrs. Mas did not effect a change of domicile since she and Mr. Mas were in Louisiana only as students and lacked the requisite intention to remain there. See Hendry v. Masonite Corp., 5 Cir., 1972, 455 F.2d 955, cert. denied, 409 U. S. 1023, 93 S.Ct. 464, 34 L.Ed.2d 315. Until she acquires a new domicile, she remains a domiciliary, and thus a citizen, of Mississippi. See Mitchell v. United States, 88 U.S. (21 Wall.) 350, 352, 22 L.Ed. 584, 587-588 (1875); Sun Printing & Publishing Association v. Edwards, 194 U.S. 377, 383, 24 S.Ct. 696, 698, 48 L.Ed. 1027 (1904); Welsh v. American Security Co. of New York, 5 Cir., 1951, 186 F.2d 16, 17.2

Appellant also contends that Mr. Mas’s claim should have been dismissed for failure to establish the requisite jurisdictional amount for diversity cases of more than $10,000. In their complaint Mr. and Mrs. Mas alleged that they had each been damaged in the amount of $100,000. As we have noted, Mr. Mas ultimately recovered $5,000.

It is well settled that the amount in controversy is determined by the amount claimed by the plaintiff in good faith. KVOS, Inc. v. Associated Press, 299 U.S. 269, 57 S.Ct. 197, 81 L. Ed. 183 (1936); 1 J. Moore, Moore’s Federal Practice ¶ 0.92 [1] (1972). Federal jurisdiction is not lost because a judgment of less than the jurisdictional amount is awarded. Jones v. Landry, 5 Cir., 1967, 387 F.2d 102; C. Wright, Federal Courts 111 (1970). That Mr. Mas recovered only $5,000 is, therefore, not compelling. As the Supreme Court stated in St. Paul Mercury Indemnity Co. v. Red Cab Co., 303 U.S. 283, 288-290, 58 S.Ct. 586, 590-591, 82 L.Ed. 845:

[T]he sum claimed by the plaintiff controls if the claim is apparently made in good faith.
It must appear to a legal certainty that the claim is really for less than the jurisdictional amount to justify dismissal. The inability of the plain*1401tiff to recover an amount adequate give the court jurisdiction does not show his bad faith or oust the jurisdiction. . to
. . . His good faith in choosing the federal forum is open to challenge not only by resort to the face of his complaint, but by the facts disclosed at trial, and if from either source it is clear that his claim never could have amounted to the sum necessary to give jurisdiction there is no injustice in dismissing the suit.

Having heard the evidence presented at the trial, the district court concluded that the appellees properly met the requirements of section 1332 with respect to jurisdictional amount. Upon examination of the record in this case, we are also satisfied that the requisite amount was in controversy. See Jones v. Landry, 5 Cir., 1967, 387 F.2d 102.

Thus the power of the federal district court to entertain the claims of appellees in this case stands on two separate legs of diversity jurisdiction: a claim by an alien against a State citizen; and an action between citizens of different States. We also note, however, the propriety of having the federal district court entertain a spouse’s action against a defendant, where the district court already has jurisdiction over a claim, arising from the same transaction, by the other spouse against the same defendant. See ALI Study of the Division of Jurisdiction Between State and Federal Courts, pt. I, at 9-10. (Official Draft 1965.) In the case before us, such a result is particularly desirable. The claims of Mr. and Mrs. Mas arise from the same operative facts, and there was almost complete interdependence between their claims with respect to the proof required and the issues raised at trial. Thus, since the district court had jurisdiction of Mr. Mas’s action, sound judicial administration militates strongly in favor of federal jurisdiction of Mrs. Mas’s claim.

Affirmed.

7.3.2.2 Sheehan v. Gustafson 7.3.2.2 Sheehan v. Gustafson

John D. SHEEHAN, Sr., Appellant, v. Deil O. GUSTAFSON, Appellee.

No. 91-3078.

United States Court of Appeals, Eighth Circuit.

Submitted May 15, 1992.

Decided June 23, 1992.

*1215Timothy D. Kelly, Minneapolis, Minn., argued, for appellant.

Alan L. Kildow, Bloomington, Minn., argued, for appellee.

Before ARNOLD, Chief Judge, LAY, Senior Circuit Judge, and BOWMAN, Circuit Judge.

BOWMAN, Circuit Judge.

John D. Sheehan, Sr., appeals the order of the District Court1 dismissing his action for lack of subject matter jurisdiction. We affirm.

Sheehan, a Nevada citizen, commenced an action in February 1991 against Deil 0. Gustafson alleging breach of an oral contract involving the proceeds of the sale of the Tropicana Hotel and Casino in Las Vegas, Nevada. Sheehan brought his action in federal court in Minnesota, asserting that diversity jurisdiction was proper under 28 U.S.C. § 1332(a) (1988), because Gustaf-son, according to the complaint, was a citizen of Minnesota. Gustafson moved to dismiss the complaint for lack of subject matter jurisdiction. The District Court granted that motion in July 1991, holding that Gustafson, like Sheehan, was a citizen of Nevada and thus there was no diversity of the parties. Sheehan appeals.

A district court’s conclusion as to citizenship for purposes of federal diversity jurisdiction is a mixed question of law and fact (albeit primarily fact). Blakemore v. Missouri Pac. R.R., 789 F.2d 616, 618 (8th Cir.1986). The findings of fact upon which the legal conclusion of citizenship is based thus are subject to review by this Court under the clearly erroneous standard. Id.

The statute conferring diversity jurisdiction in federal court requires that the parties be citizens of different states. 28 U.S.C. § 1332(a)(1). Section 1332(a) must be strictly construed, in view “of the constitutional limitations upon the judicial power of the federal courts, and of the Judiciary Acts in defining the authority of the federal courts when they sit, in effect, as state courts.” Indianapolis v. Chase Nat’l Bank, 314 U.S. 63, 76, 62 S.Ct. 15, 20, 86 L.Ed. 47 (1941) (footnote omitted); accord Owen Equip. & Erection Co. v. Kroger, 437 U.S. 365, 377, 98 S.Ct. 2396, 2404, 57 L.Ed.2d 274 (1978). Thus the burden falls upon the party seeking the federal forum, if challenged, to demonstrate by a preponderance of the evidence that the parties are citizens of different states. Blake-more, 789 F.2d at 618. The District Court found facts that are not clearly erroneous and determined that Sheehan failed to carry his burden. We cannot say the court erred as a matter of law.

Courts look to the facts as of the date an action is filed to determine whether or not diversity of citizenship exists between the parties. Yeldell v. Tutt, 913 F.2d 533, 537 (8th Cir.1990). “For purposes of diversity jurisdiction, the terms ‘domicile’ and ‘citizenship’ are synonymous.” Id. Therefore, to determine if Gustafson is a citizen of a state other than Nevada, the proper analysis is the two-part test for domicile: Gustafson’s presence in the purported state of domicile and his intention to remain there indefinitely. See id.

The facts of this case, as found by the District Court, indicate that in February 1991 Gustafson had a presence in both Nevada and Minnesota (as well as California and Florida). Gustafson was a citizen of Minnesota until 1973 when he moved to Las Vegas, Nevada, to manage the Tropicana Hotel, which he had purchased in 1972. In 1975, he sold eighty percent of his interest in the hotel. In 1983, Gustafson was convicted in federal court in Minnesota of misappropriation of bank funds and was incarcerated from 1984 to 1987.

The facts found by the District Court that are evidence of Minnesota domicile as *1216of February 1991 include Gustafson’s bank and investment accounts in the state; ownership of property in Minnesota by a corporation controlled by Gustafson, including a condominium whose address Gustafson uses as his own in his monthly reports to his probation officer; Gustafson’s use of corporate vehicles when in the state; location of Gustafson’s secretary and office in Minneapolis, where he regularly checks for messages and mail; location of Gustafson’s physician, dentist, and attorney in the state; and his use of Minnesota addresses and bank accounts for some of his Nevada businesses.

The facts found supporting Nevada domicile include Gustafson’s holding a Nevada driver’s license since 1973 and the registration of his personal vehicles in the state; Gustafson’s filing of Minnesota tax returns as a non-resident since 1974, with a Nevada permanent address shown; Gustafson’s voter registration in Nevada since 1973; his current valid passport showing a Nevada address; Gustafson’s last will (dated July 2,1989) containing a statement that he is domiciled in Clark County, Nevada; use of his parents’ home address in Boulder, Nevada, as Gustafson’s permanent address since the mid-1980’s; and current (as of 1991) construction of a new home on his ranch in Ely, Nevada.2

The facts demonstrate Gustafson’s presence and intent to remain in Nevada, that is, domicile in Nevada. Sheehan did not show by a preponderance of the evidence that Gustafson’s domicile was in fact in Minnesota at the time suit was filed. Shee-han’s evidence to show domicile in Minnesota primarily demonstrates Gustafson’s business contacts and occasional presence in the state; the Nevada contacts are more indicative of intent to remain.

Finding no clearly erroneous findings of fact and no error of law, we affirm the decision of the District Court.

7.3.2.3 Belleville Catering Co. v. Champaign Market Place, L.L.C. 7.3.2.3 Belleville Catering Co. v. Champaign Market Place, L.L.C.

BELLEVILLE CATERING CO., et al., Plaintiffs-Appellants, v. CHAMPAIGN MARKET PLACE, L.L.C., Defendant-Appellee.

No. 02-3975.

United States Court of Appeals, Seventh Circuit.

Argued Oct. 22, 2003.

Decided Dec. 1, 2003.

*640Stephen B. Evans (argued), Deeba Sau-ter Herd, St. Louis, MO, for Plaintiffs-Appellants.

Jerome P. Lyke (argued), Flynn, Palmer & Tague, Champaign, IL, for Defendant Appellee.

Before FLAUM, Chief Judge, and EASTERBROOK and WILLIAMS, Circuit Judges.

EASTERBROOK, Circuit Judge.

Once again litigants’ insouciance toward the requirements of federal jurisdiction has caused a waste of time and money. See, e.g., Hart v. Terminex International, 336 F.3d 541 (7th Cir.2003); Meyerson v. Showboat Marina Casino Partnership, 312 F.3d 318 (7th Cir.2002); Meyerson v. Harrah’s East Chicago Casino, 299 F.3d 616 (7th Cir.2002); Indiana Gas Co. v. Home Insurance Co., 141 F.3d 314 (7th Cir.1998); Guaranty National Title Co. v. J.E.G. Associates, 101 F.3d 57 (7th Cir. 1996); Kanzelberger v. Kanzelberger, 782 F.2d 774 (7th Cir.1986).

Invoking the diversity jurisdiction, see 28 U.S.C. § 1332, the complaint alleged that the corporate plaintiff is incorporated in Missouri and has its principal place of business there, and that the five individual plaintiffs (guarantors of the corporate plaintiffs obligations) are citizens of Missouri. It also alleged that the defendant is a “Delaware Limited Liability Company, with its principle [sic] place of business in the State of Illinois.” Defendant agreed with these allegations and filed a counterclaim. The parties agreed that a magistrate judge could preside in lieu of a district judge, see 28 U.S.C. § 636(c), and the magistrate judge accepted these jurisdictional allegations at face value. A jury trial was held, ending in a verdict of $220,000 in defendant’s favor on the counterclaim. Plaintiffs appealed; the jurisdictional statement of their appellate brief tracks the allegations of the complaint. Defendant’s brief asserts that plaintiffs’ jurisdictional summary is “complete and correct.”

It is, however, transparently incomplete and incorrect. Counsel and the magistrate judge assumed that a limited liability company is treated like a corporation and thus is a citizen of its state of organization and its principal place of business. That is not right. Unincorporated enterprises are analogized to partnerships, which take the citizenship of every general and limited partner. See Carden v. Arkoma Associates, 494 U.S. 185, 110 S.Ct. 1015, 108 L.Ed.2d 157 (1990). In common with other courts of appeals, we have held that limited liability companies are citizens of every state of which any member is a citizen. See Cosgrove v. Bartolotta, 150 F.3d 729 (7th Cir.1998). So who are Champaign Market Place LLC’s members, and of what states are they citizens? Our effort to explore jurisdiction before oral argument led to an unexpected discovery: Belleville Catering, the corporate plaintiff, appeared to be incorporated in Illinois rather than Missouri!

At oral argument we directed the parties to file supplemental memoranda addressing jurisdictional details. Plaintiffs’ response concedes that Belleville Catering is (and always has been) incorporated in Illinois. Counsel tells us that, because the lease between Belleville Catering and Champaign Market Place refers to Belleville Catering as “a Missouri corporation,” he assumed that it must be one. That confesses a violation of Fed. *641R.Civ.P. 11. People do not draft leases with the requirements of § 1332 in mind — perhaps the lease meant only that Belleville Catering did business in Missouri — and counsel must secure jurisdictional details from original sources before making formal allegations. That would have been easy to do; the client’s files doubtless contain the certificate of incorporation. Or counsel could have done what the court did: use the Internet. Both Illinois and Missouri make databases of incorporations readily available. Counsel for the defendant should have done the same, instead of agreeing with the complaint’s unfounded allegation.

Both sides also must share the blame for assuming that a limited liability company is treated like a corporation. In the memorandum filed after oral argument, counsel for Champaign Market Place relate that several of its members are citizens of Illinois. Citizens of Illinois thus are on both sides of the suit, which therefore cannot proceed under § 1332. Moreover, for all we can tell, other members are citizens of Missouri. Champaign Market Place says that one of its members is another limited liability company that “is asserting confidentiality for the members of the L.L.C.” It is not possible to litigate under the diversity jurisdiction with details kept confidential from the judiciary. So federal jurisdiction has not been established. The complaint should not have been filed in federal court (for Belleville Catering had to know its own state of incorporation), the answer should have pointed out a problem (for Champaign Market Place’s lawyers had to ascertain the legal status of limited liability companies), and the magistrate judge should have checked all of this independently (for inquiring whether the court has jurisdiction is a federal judge’s first duty in every case).

Failure to perform these tasks has the potential, realized here, to waste time (including that of the put-upon jurors) and run up legal fees. Usually parties accept the inevitable and proceed to state court once the problem becomes apparent. Perhaps the most extraordinary aspect of this proceeding, however, is the following passage in defendant’s post-argument memorandum:

Defendant-Appellee, Champaign Market Place L.L.C., prays that this Court in the exercise of its Appellate jurisdiction decide the case on the merits and affirm the judgment entered on the jury’s verdict. Surely in the past this Court has decided a case on the merits where an examination of the issue would have shown a lack of subject matter jurisdiction in the District Court. It would be unfortunate in the extreme for Cham-paign Market Place L.L.C. to lose a judgment where Belleville Catering Company, Inc. misrepresented (albeit unintentionally) its State of incorporation in its Complaint.... [Tjhere was no reason for Champaign Market Place L.L.C. to question diversity of citizenship, since it is not, and never has been, a citizen of Missouri.

This passage — and there is more in the same vein — leaves us agog. Just where do appellate courts acquire authority to decide on the merits a case over which there is no federal jurisdiction? The proposition that the Seventh Circuit has done so in the past — a proposition unsupported by any citation — accuses the court of dereliction combined with usurpation. “A court lacks discretion to consider the merits of a case over which it is without jurisdiction”. Firestone Tire & Rubber Co. v. Risjord, 449 U.S. 368, 379, 101 S.Ct. 669, 66 L.Ed.2d 571 (1981). And while counsel feel free to accuse the judges of ultra vires conduct, and to invite some more of it, they exculpate themselves. Lawyers for *642defendants, as well as plaintiffs, must investigate rather than assume jurisdiction; to do this, they first must learn the legal rules that determine whose citizenship matters (as defendant’s lawyers failed to do). And no entity that claims confidentiality for its members’ identities and citi-zenships is well situated to assert that it could believe, in good faith, that complete diversity has been established.

One more subject before we conclude. The costs of a doomed foray into federal court should fall on the lawyers who failed to do their homework, not on the hapless clients. Although we lack jurisdiction to resolve the merits, we have ample authority to govern the practice of counsel in the litigation. See, e.g., Willy v. Coastal Corp., 503 U.S. 131, 112 S.Ct. 1076, 117 L.Ed.2d 280 (1992); Cooter & Gell v. Hartmarx Corp., 496 U.S. 384, 393-98, 110 S.Ct. 2447, 110 L.Ed.2d 359 (1990); Szabo Food Service, Inc. v. Canteen Corp., 823 F.2d 1073 (7th Cir.1987). The best way for counsel to make the litigants whole is to perform, without additional fees, any further services that are necessary to bring this suit to a conclusion in state court, or via settlement. That way the clients will pay just once for the litigation. This is intended not as a sanction, but simply to ensure that clients need not pay for lawyers’ time that has been wasted for reasons beyond the clients’ control.

The judgment of the district court is vacated, and the proceeding is remanded with instructions to dismiss the complaint for want of subject-matter jurisdiction.

7.3.2.4 Notes on Diversity of the Parties 7.3.2.4 Notes on Diversity of the Parties

     1.  Complete versus Minimal Diversity. Strawbridge v. Curtiss, 7 U.S. (3 Cranch) 267 (1806) announced a rule of complete diversity. At the time, it was unclear whether the rule was based on the Constitution or on the predecessor statute to Section 1332. It is now clear that the case is to be read as one of statutory interpretation, with the Constitution only requiring minimal diversity. See State Farm v. Tashire, 386 U.S. 523 (1967). You might ask: what is complete diversity and what is minimal diversity? Complete diversity requires that no defendant be from the same state as any plaintiff. For example, NY & NJ v. PA & DE satisfies complete diversity; NY & NJ v. PA & NJ does not. Minimal diversity merely requires that some defendant be from a different state from some plaintiff. NY & NJ v. NJ does satisfy minimal diversity. Later on in the course we will discuss a part of Section 1332 we are skipping over now, the Class Action Fairness Act. This statute allows certain types of class actions to be brought in federal court where there is only minimal diversity. For ordinary § 1332(a) diversity claims, however, full diversity is required.

     2.  Determining Domicile Generally.  The test for creating a new domicile for individuals has two elements: 1) residence in a state with 2) intent to remain 'indefinitely.' Indefinitely does not include an uncertain period of time but with a departure expected (for example, 'until I finish my degree'); it means an intent to remain perhaps permanently. Once fixed, it takes more than a change of residence to reset domicile. There must be an intention to stay indefinitely. Without that intent, domicile remains where it has been. What would have happened in Mas if the plaintiff had decided at any point while she was resident in Louisiana that she wanted to make Louisiana her permanent home? What would her domicile be if after the facts in this case she moved to, say, Ohio, but did not form an intent to remain indefinitely?

     3.  Determining Domicile for Corporations and Unincorporated Entities. As Judge Easterbrook reminds us, the test for determining domicile depends on the nature of the party. Individuals are treated differently from corporations, which are treated differently from partnerships, LLCs, and other unincorporated associations such as labor unions. For unincorporated associations, we look to the residence of each partner or member. As a practical matter, for some organizations such as national labor unions this makes bringing a diversity claim essentially impossible as there are likely to be members in every state. For corporations, the rule makes clear that there are possibly two domiciles, very similar to what we saw in Daimler - the state of incorporation and the principal place of business. In Hertz Corp. v. Friend, 559 U.S. 77 (2010), the Court made clear that the principal place of business is not where a corporation conducts the most business or makes the most sales, but where its 'nerve center' is. As a practical matter, in almost every case the nerve center will be the location of the corporate headquarters. 

     4.  Note on LLCs: You might not have covered Limited Liability Companies (LLCs) in Business Associations, which often focuses on corporations and partnerships. LLCs are a relatively recent innovation in company structure, having originated in the latter part of the 20th century in the US. They combine some aspects of partnerships that are often attractive to investors (such as tax pass-throughs so losses are deductible by investors) and aspects of corporations that are attractive as well (such as limited liability). They have become a very popular structure for smaller, privately held companies. Beyond LLCs, partnerships, and corporations, note that the variety of business organizations in the US can be bewildering. It is important to pay close attention to what the structure is. The situation gets more complicated, and beyond the scope of this course, when foreign business entities are involved. China has LLCs, for example. What would be the domicile of a Chinese LLC that has US domiciliaries and Chinese domiciliaries as members? So far as we know, no case to date has confronted that.

     5.  Time for determining domicile of parties. People and companies, including parties and potential parties to litigation, sometimes move. At what time do we assess domicile? At the time the action arose? At the time the lawsuit was filed? At the time the court is deciding the issue? The answer is simple - at the time the lawsuit is filed. Subsequent moves will not divest the court of jurisdiction. Moves that take place before filing, even if they have the effect of either creating or denying diversity, will not normally be questioned so long as the move is a real move in good faith and not a pretense. What would be the analysis if Mas had moved to Ohio, permanently and with an intention to stay, before filing suit? What would the analysis be if after the facts giving rise to the lawsuit but before filing she had decided to remain in Louisiana indefinitely?

     6.  Pleading Domicile. The party asserting a claim must plead those facts essential to establish subject matter jurisdiction. Because courts are not free to hear cases that fall outside the limited subject matter reach of the federal courts, this is not something federal judges will ignore. A litigant who makes conclusory or unclear allegations of subject matter jurisdiction may - and probably will - be allowed an opportunity to amend the complaint to make the allegations sufficient, but if the litigant fails to get the job done the case will be dismissed. This will happen even if the true, but unalleged facts, would support a finding of subject matter jurisdiction if the court knew about them. See Sienna Ventures, LLC v. Halley Equipment Leasing, LLC, 2018 WL 3153475 (E.D.N.Y Apr. 2, 2018) (Dismissing for lack of subject matter jurisdiction where after three tries plaintiff's jurisdictional allegations of “subject matter jurisdiction exists in this case" were not enough even though Sienna Ventures, LLC is a New York Limited Liability Company and its sole member is a citizen and resident of New York, and Halley Equipment Leasing, LLC is a Texas Limited Liability Company.” The complaint failed to state the domiciles of all the LLC members, which if they had been alleged would have satisfied subject matter jurisdiction).

     7.  Proving Domicile. As both Mas and Sheehan v. Gustafson note, establishing domicile depends on a two part test - presence in a state and intent to remain. But how do we determine domicile when it is disputed? Do we just take a party's word at face value? Sheehan v. Gustafson makes several things clear. First, it is the burden of the claimant to establish subject matter jurisdiction. Second, we do not simply accept assertions at face value, but look to evidence such as where one votes or has a driver's license to inform the assessment. Third, while the burden is on the claimant, if the issue is disputed the court will normally allow discovery in order to establish the facts (with the costs and process of discovery something that should be taken into account before domicile is challenged). Finally, the determination is one within the discretion of the trial court, and reviewing courts will apply an abuse of discretion standard when reviewing the determination of the trial court. In some cases, and Sheehan v. Gustafson is an excellent example, determining residence can be far from straightforward.  You will note from Sheehan v. Gustafson that a number of documents can be used to show domicile, from passports to driver's licenses to voter registrations, and they may not be consistent. Compare that to China, where official residency is determined by a hukou document, with important consequences that flow from the official documentation.

     8.  U.S. Citizens Domiciled in No State: Can diversity or alienage jurisdiction be asserted over someone who is a U.S. citizen but not domiciled in any state in the U.S.? In the age of globalization, this has become not uncommon, as U.S. citizens establish residency in foreign countries and for tax or other reasons have no connection with any single US state. Are they citizens of a state? Are they subjects or citizens of a foreign country? If you think about it, you will see that they are neither - still US citizens but not resident in any state. Note what the court in Mas said when it considered what would be the impact of imputing her husband's residency to her: "She would not be a citizen of any State and could not sue in a federal court on that basis; nor could she invoke the alienage jurisdiction to bring her claim in federal court, since she is not an alien." 

     9.  Citizenship of Wife. Mas v. Perry discusses a rule where for US citizens the domicile of the wife was set conclusively as that of the husband. Like many other legal rules from earlier eras that assumed that wives were subordinate to husbands, that rule no longer applies. The established domicile of a spouse, either spouse, might be probative as a factual matter in a close factual case, but today there will be no conclusions reached based solely on the domicile of the husband.

     10.  Why Diversity?  Diversity jurisdiction has long been controversial. After all, in every case a state court could also resolve the dispute, and there are those who argue (mostly not experienced trial lawyers) that prejudice against out of state parties is a thing of the past. Maintaining diversity access has many costs. First, it complicates things procedurally. As you should have noticed over and over again in the progression of this course, battling over forum selection is something lawyers spend a lot of time engaged in, and diversity jurisdiction creates opportunities for extensive procedural battles related to such issues. It also imposes significant burdens on the federal courts. In 2019, of 286,289 total cases filed in district court that year 94,206 - or nearly 33 percent - were diversity cases. Arguments are also made that diversity cases are more time consuming than the average case, and more likely to go trial. Diversity cases also shift resolution of state law issues to the federal courts, whereas without diversity at least some of these state legal issues would be resolved within the state system whose law is involved. On the other hand, diversity jurisdiction allows national procedural tools such as multidistrict litigation to be employed on mass tort or other aggregate cases. Many lawyers and parties also passionately believe that the days of local prejudice against outsiders are not an artifact of history but remain a live concern. Then, there are those who appreciate and value the forum selection potential - they might prefer a federal jury venire, which tends to draw from a wider area than a state jury pool, or they may prefer federal judges, who tend to be highly capable. In any event, diversity jurisdiction abides, and you should not expect it to quickly or easily go away. 

7.3.2.6 Problems and Hypotheticals 7.3.2.6 Problems and Hypotheticals

     See if you can work through the following problems. Answers will eventually be provided, but you will be much better prepared for the exam (and life as a lawyer who must resolve such issues with no one giving you the answer) if you make the effort to solve problems like this on your own.

     Judy Moore was born and raised in the university town of Oxford, Mississippi, where both her parents were professors. At age 18 she enrolled in college in Wellesley, Massachusetts, and after graduation with a degree in economics began a graduate program in Cambridge, Massachusetts. Early on during her ten years in Massachusetts she learned that she didn't like snow, and hoped that when she graduated she could secure a teaching job in a warmer climate. Near the end of her doctoral studies, she traveled for a job interview to Athens, Georgia, where she was offered and quickly accepted a job as a university professor. She liked Athens, and hoped she could spend the rest of her life there. While in Athens, unfortunately, she was injured when a car owned by LaGlace Enterprises struck her. The car was driven by Justah Dope, a student at the University of Georgia. Dope was originally from Chattanooga, Tennessee, and planned to move to New York, New York, to pursue a career as an actor upon graduation, but he has little likelihood of actually securing a job offer there and may be forced to remain in Georgia indefinitely. LaGlace Enterprises is a limited liability company created under the laws of Mississippi and headquartered in Columbia, South Carolina. The members (owners) are Perry Miroir, a resident of Tuscaloosa, Alabama, and Edgar Espejo, a resident of Athens, Georgia. Once back in Massachusetts, Moore consulted with a lawyer, who advised her to bring a lawsuit. The lawsuit was filed in federal district court in Georgia shortly before Moore obtained her PH.D. degree against Dope and LaGlace Enterprises. Shortly after the lawsuit was filed but before the judge could hear a motion to dismiss for lack of subject matter jurisdiction, Moore moved to Georgia to start her new job. With regard only to the diversity of the parties, please analyze the issues.

     MuchoDinero, LLC, is a limited liability company created under the laws of Nevada and headquartered in Tucson, Arizona. It has members (owners) from Arizona, California, and Utah. LotsaMonie, LLC, is a limited liability company created under the laws of Washington state and headquartered in Portland, Oregon. It has members from Oregon, Washington state, and Idaho. MuchoDinero filed a suit against LotsaMonie in federal district court in Portland. The complaint alleged “subject matter jurisdiction exists in this case as MuchoDinero is a Nevada Limited Liability Company and LotsaMonie is a Washington state Limited Liability Company.” After giving MuchoDinero repeated opportunities to amend its complaint, the judge dismissed the lawsuit for lack of subject matter jurisdiction. Did the judge act properly?

7.3.3 Alienage Jurisdiction 7.3.3 Alienage Jurisdiction

7.3.3.1 Hebei Tiankai Wood & Land Constr. Ltd. v. Chen, 348 F Supp 3d 198 (EDNY 2018). 7.3.3.1 Hebei Tiankai Wood & Land Constr. Ltd. v. Chen, 348 F Supp 3d 198 (EDNY 2018).

MEMORANDUM & ORDER

JACK B. WEINSTEIN, Senior United States District Judge:

Table of Contents

I. Introduction...200

II. Facts...200

A. Transaction...200

B. Dispute...201

C. Motion to Realign and Dismiss...202

III. Law...202

A. Summary Judgment Standard of Review...202

B. Diversity Jurisdiction...203

C. Antagonism Doctrine...203

IV. Application of Law to Facts...204

A. Plaintiff's Showing of Antagonism...204

B. Importance of Alienage Jurisdiction...204

V. Conclusion...206

I. Introduction

This litigation arises from a transnational business transaction gone wrong. In 2017, Hebei Tiankai Wood & Land Construction Co. Ltd. (“Plaintiff”) and Kirin Transportation, Inc. (“Kirin”) agreed that Plaintiff would become majority shareholder of Kirin. At the time of the transaction, Frank Chen (“Chen”) was the sole owner of Kirin. Plaintiff alleges that Chen made false representations to induce Plaintiff to invest $300,000 in Kirin and then misused the money.

Chen moved to dismiss under Federal Rule of Civil Procedure 12(b)(1) for lack of subject matter jurisdiction. The court ordered the motion converted to one for summary judgement.

Plaintiff is a citizen of China, while Chen and Kirin, the nominal defendant, are citizens of New York. Chen argues that Plaintiff's fraudulent inducement and breach of fiduciary duty claims are derivative, properly belonging to Kirin, so Kirin should be aligned as a plaintiff. Realignment of Kirin as plaintiff would destroy diversity, requiring the case to be dismissed.

Plaintiff argues that the “doctrine of antagonism”—applicable between Kirin and Plaintiff—applies, so that realignment is not required.

Chen disagrees. He contends that Plaintiff is Kirin's majority shareholder so that there is no antagonism between these two.

Chen's argument is not persuasive. First, even if Plaintiff has de jure control of Kirin, Chen has de facto control. Chen is alleged to have forcefully rebuffed Plaintiff's attempts to take control of Kirin, even though Plaintiff is now majority shareholder of Kirin. The caselaw is clear that a court should ensure that the parties are aligned so that there is a real collision of interests.

The real collision of interests is between Plaintiff on one side and Chen and Kirin on the other. Second, at its core, this is a dispute between nationals of the United States and of China, making it desirable to try the case in federal court if subject matter jurisdiction exists.

Kirin should remain as a named defendant. This court has jurisdiction over Plaintiff's claims. Chen's motion is denied.

II. Facts

A. Transaction

Plaintiff is a construction company based in China. Compl. ¶ 4, ECF No. 1, *201 May 10, 2018; Answer Ex. C, ECF No. 9-3. Chinese nationals Qiuxiang Shi (“Shi”) and Yingjie Li (“Li”) are the sole owners, officers, and managers of Plaintiff. Compl. ¶¶ 8, 17. In November 2017, Shi and Li traveled to New York and were introduced to Chen, then the sole shareholder of Kirin. Id. ¶¶ 9–11; Answer ¶¶ 9–11, ECF No. 9, May 29, 2018. Chen is a resident of the State of New York; Kirin is a New York corporation. Compl. ¶¶ 5–6; Answer ¶¶ 5–6. Kirin is alleged to be only a nominal defendant. Compl. ¶ 6.

Chen invited Shi and Li to Kirin's headquarters in Queens, New York. Id. ¶ 12; Answer ¶ 12. He wanted the duo to invest in Kirin. Id. Plaintiff alleges that Chen represented that Kirin was a successful company looking to expand its business; he gave Shi and Li documents to “highlight the financial strength” of the company. Compl. ¶¶ 13–14. It is undisputed that Chen stated that if Shi and Li invested in Kirin, Kirin would sponsor Shi for an L-1 visa. Id. ¶ 18; Answer ¶ 18. Plaintiff alleges that the visa would have allowed Shi to travel to the United States to participate in the management of Kirin and monitor operations of the company. Compl. ¶ 18. Chen organized meetings with law firms specializing in immigration law to demonstrate his intent to help obtain the visa if Plaintiff invested in Kirin. Id. ¶ 19; Answer ¶ 19.

The parties agree that Chen offered Shi and Li 51% of the shares of Kirin for $300,000 in capital investment. Compl. ¶ 20; Answer ¶ 20. Plaintiff alleges that Chen also promised to pay Shi a substantial salary as a manager. Compl. ¶ 20. On November 28, 2017, Chen gave Shi and Li a contract memorializing the agreement; in exchange for $300,000, Kirin agreed to tender 51% of Kirin's shares to Plaintiff and to file an L-1 visa petition for Shi. Id. ¶¶ 20, 22; Answer Ex. E, ¶¶ 1–2, ECF No. 9-5. Chen signed an engagement letter with a lawyer to work with Kirin to obtain the visa. Compl. ¶ 26; Answer ¶ 26. On the same day, Plaintiff tendered a certified check to Kirin for $300,000. Compl. ¶ 28; Answer ¶ 28. A stock certificate in turn issued to Plaintiff was for 102 shares, a majority of those in the company. See Decl. Frank Chen Ex. B, ECF No. 18-4.

B. Dispute

What had seemed like a straightforward transaction then began to unravel. In January 2018, per Plaintiff's allegations, Chen contacted Shi and Li asking for additional funds to be invested in Kirin so that Kirin could help obtain a visa for Shi. Compl. ¶ 30. Allegedly, Chen represented that the funds already invested “had been used up.” Id. ¶ 31. Plaintiff became suspicious and asked Kirin for documents and records, which Chen refused to produce. Id. ¶ 32. The parties agree that in April 2018, Plaintiff discovered that it was not listed as a shareholder on Kirin's 2017 tax returns. Id. ¶ 35; Answer ¶ 35.

Plaintiff attempted to establish control of Kirin. Plaintiff alleges that it sent a representative to take over Kirin, but its employees (pursuant to Chen's instructions), refused the representative access to the premises and inspection of books and records. Compl. ¶ 33. Alleged is that Kirin's bank refused to make account records available, and that Chen had never told Kirin's employees of Plaintiff's investment, never informed the landlord of the property where Kirin operated of Plaintiff's investment, and never made Plaintiff an authorized signatory on any of Kirin's bank accounts. Id. ¶¶ 33–34.

Chen denies these allegations. Answer ¶¶ 33–34. But, based on the record, they appear to be true. See Hr'g Tr. 4:15–19, *202 5:1–3, Dec. 19, 2018 (parties agree no evidentiary hearing was required).

In April 2018, Shi, as chair and director of Kirin, removed Chen from all positions at Kirin and named Shi chief executive officer of Kirin. Decl. Frank Chen Ex. D, ECF No. 18-5. Shi also named an interim chief executive officer of Kirin, effective immediately. Decl. Frank Chen Ex. D, ECF No. 18-6.

Chen took steps to counter these moves. On April 17, 2018, his attorney Xiangan Gong (“Gong”) wrote to Plaintiff's counsel opposing the “attempted take-over of the business.” Decl. Jacob Chen Opp. Frank Chen’s Mot. Dismiss Ex. A, ECF No. 23-2 (“Pl. Ex. B”). Gong wrote:

The stock transfer agreement between the parties entered last November was never finalized: (1) Frank Chen was and still is the one who actually supervises and manages the company's daily business operation, Qiuxiang Shi had never participated into company's management or business operation; (2) the lease was signed by Frank Chen; (3) Frank Chen was the only authorized personnel for the company's bank account.

Id. (emphasis added). In the same email, Gong wrote, “[Chen] CANNOT LEAVE THE COMPANY AND LET YOUR CLIENT OPERATE THE BUSINESS WITH HIS LEASE, SUBJECT HIM TO VIOLATION OF THE LEASE.” Id. (capitalization in original).

On May 1, 2018, Gong repeated Chen's position that he properly maintained control of Kirin. Decl. Jacob Chen Opp. Frank Chen’s Mot. Dismiss Ex. B, ECF No. 23-3 (“Pl. Ex. B”). Gong wrote,

[Chen] prefers to do a clean-cut closing then your client takes full control of the corporation and its business operation.... [Chen] cannot gives [sic] up the management right when the lease agreement and most importantly the company's TLC license was under his name.

Id.

C. Motion to Realign and Dismiss

In May, Plaintiff brought three causes of action against Chen and Kirin: (1) fraudulent inducement; (2) breach of contract; and (3) breach of fiduciary duty. Compl. at 6–9. Alleged is that a demand for Kirin to institute suit against Chen would be futile. Id. ¶ 7. Chen and Kirin filed a joint answer in response to Plaintiff's complaint. Answer at 1. By August, Chen had hired new counsel for himself, but he declined to hire new counsel for Kirin. Minute Entry, ECF No. 25, Aug. 15, 2018.

Chen's new attorney filed a motion to dismiss for lack of subject matter jurisdiction, arguing that Kirin should be realigned as a plaintiff, thus destroying the complete diversity of citizenship required for this court to have subject matter jurisdiction. See Def. Chen's Notice Mot. & Mot. to Dismiss Under Rule 12(b)(1) for Lack of Subject Matter Jurisdiction at 4, ECF No. 18, Aug. 5, 2018.

This court converted Chen's motion to dismiss to a motion for summary judgment. Order, ECF No. 26, Aug. 27, 2018. A hearing on the motion was held on December 19, 2018. See Hr'g Tr., Dec. 19, 2018. The parties agreed that the motion could be decided without an evidentiary hearing. Id. 4:15–19, 5:1–3.

III. Law

A. Summary Judgment Standard of Review

12At the summary judgment stage, “the judge's function is not ... to weigh the evidence and determine the truth of the matter but to determine whether there is a genuine issue for trial.” *203 Anderson v. Liberty Lobby, Inc., 477 U.S. 242, 249, 106 S.Ct. 2505, 91 L.Ed.2d 202 (1986). The movant must show that “there is no genuine dispute as to any material fact and the movant is entitled to judgment as a matter of law.” Fed. R. Civ. P. 56(a). A genuine dispute of material fact exists for summary judgment purposes when, “the evidence, viewed in the light most favorable to the nonmoving party, is such that a reasonable jury could decide in that party's favor.” Chaohui Tang v. Wing Keung Enterprises, Inc., 210 F.Supp.3d 376, 388 (E.D.N.Y. 2016) (quotation marks and citation omitted).

B. Diversity Jurisdiction

3District courts have original jurisdiction of civil actions between a citizen of one state and a citizen of a foreign state when the amount in controversy exceeds $75,000. 28 U.S.C. § 1332(a). Alienage jurisdiction is a subset of diversity jurisdiction.

456“[D]iversity jurisdiction is available only when all adverse parties to a litigation are completely diverse in their citizenships.” Herrick Co. v. SCS Communications, Inc., 251 F.3d 315, 321 (2d Cir. 2001). Federal diversity jurisdiction cannot be conferred by the parties' own determination of who should be plaintiffs and who should be defendants. Rather, “the federal courts are required to realign parties according to their real interests so as to produce an actual collision of interests.” Lewis v. Odell, 503 F.2d 445, 447 (2d Cir. 1974); see also City of Indianapolis v. Chase Nat. Bank of City of New York, 314 U.S. 63, 69, 62 S.Ct. 15, 86 L.Ed. 47 (1941) (it is the court's duty to “look beyond the pleadings, and arrange the parties according to their sides in the dispute” (citation omitted)).

C. Antagonism Doctrine

78Generally, a shareholder has no individual cause of action for a wrong against a corporation. Abrams v. Donati, 66 N.Y.2d 951, 498 N.Y.S.2d 782, 489 N.E.2d 751, 751 (N.Y. 1985). This results in the corporation being aligned as the plaintiff in litigation brought to remedy harm to a corporation because the corporation is the real party in interest. See Liddy v. Urbanek, 707 F.2d 1222, 1224 (11th Cir. 1983).

91011When, however, “management is aligned against the stockholder and defends a course of conduct which [the stockholder] attacks,” there is “antagonism” and the corporation should be aligned as a defendant. Smith v. Sperling, 354 U.S. 91, 95, 77 S.Ct. 1112, 1 L.Ed.2d 1205 (1957). Fraud, breach of trust, and illegality on behalf of the management in control are strong indicators of antagonism. Id. (collecting cases); ZB Holdings, Inc. v. White, 144 F.R.D. 42, 46 (S.D.N.Y. 1992) (“[I]f the complaint in a derivative action alleges that the controlling shareholders or dominant officials of the corporation are guilty of fraud or malfeasance, then antagonism exists, and the corporation should be aligned as a defendant.”). Antagonism may also exist where management is hostile to the interests of shareholders. Smith, 354 U.S. at 97, 77 S.Ct. 1112; Rogers v. Valentine, 426 F.2d 1361, 1363 (2d Cir. 1970) (upholding the district court's decision that a corporation should be aligned as a defendant when management had refused to institute suit on behalf of the corporation and demand would be futile).

12Courts should normally determine the issue of antagonism without delving into the merits of the claims. See Smith, 354 U.S. at 95, 77 S.Ct. 1112 (“the instant case is a good illustration” of why not “to delve into the merits” “for it has been over *204 eight years in the courts on this question of jurisdiction”).

IV. Application of Law to Facts

A. Plaintiff's Showing of Antagonism

13It is assumed for purposes of this opinion that Plaintiff's claims are derivative and intended to remedy wrongs to the corporation. Nevertheless, Kirin remains properly aligned as a defendant because of antagonism.

First, though Plaintiff has de jure control of Kirin, Chen has de facto control. It is undisputed that Chen is managing Kirin and Plaintiff's attempts to obtain control have failed. Those attempts are alleged to have been forcefully rebuffed by Chen. At his direction, Kirin's employees denied Plaintiff access to Kirin's operating premises and books and records. Chen also denied Plaintiff access to Kirin's bank by failing to make Plaintiff a signatory on Kirin's bank account.

Chen has explicitly refused to cede control of Kirin to Plaintiff. In April 2018, Shi, chair and sole director of the corporation, “removed” Chen from all positions in Kirin and appointed a new chief executive officer of Kirin. But Chen refused to cede management of Kirin to Shi's appointee. See Pl. Ex. A (“Mr. Chen DOES oppose the attempted take-over of the business.” (emphasis in original) ); id. (“Frank Chen was and still is the one who actually supervises and manages the company's daily business operation”); Pl. Ex. B (“[Chen] cannot gives [sic] up the management right”).

In view of Chen's actual management of Kirin and Plaintiff's alleged inability to wrest control from him despite being majority shareholder, it is highly unlikely that Chen would institute either of Plaintiff's derivative claims against Kirin. Based on the facts alleged in the complaint, such a demand would be futile. In these circumstances, antagonism exists. See Rogers, 426 F.2d at 1363 (upholding the district court's decision that a corporation should be aligned as a defendant when management had refused to institute suit on behalf of the corporation and demand would be futile).

14Second, Plaintiff alleges that Chen has engaged in fraud by wasting the investment in Kirin by Plaintiff and by engaging in other acts of malfeasance, including failing to list Plaintiff as a shareholder on Kirin's 2017 tax returns. When the dominant official of the corporation is accused of fraud, the corporation is appropriately aligned as a defendant. See ZB Holdings, 144 F.R.D. at 46 (concluding that “the requisite antagonism” existed because plaintiff alleged that the corporation's directors—the “dominant officials”—had engaged in common law fraud and misrepresentation).

Chen argues that antagonism cannot be found because Plaintiff is the majority shareholder of Kirin. Majority “control” of a corporation, however, is only one “circumstance indicating a lack of antagonism.” Taylor v. Swirnow, 80 F.R.D. 79, 83 (D. Md. 1978) (concluding that the parties lacked antagonism when plaintiffs owned a majority share in the corporation, among other reasons).

The court's role is to determine where the real collision of interests lies. The pleadings show that collision between Plaintiff on one side and Chen and Kirin on the other.

B. Importance of Alienage Jurisdiction

This litigation, at its core, is a transnational business dispute between a Chinese corporation and a New York citizen. It is the transnational nature of the action—and the fact that all relevant activity took place *205 in the United States—that make it vital that the parties be able to litigate in a United States court. Reliability of the United States courts is apparent.

Alienage jurisdiction—the jurisdiction of the federal courts over suits between citizens of the United States and citizens of a foreign state—is guaranteed by the Constitution. U.S. Const. art III, § 2 (“The judicial Power shall extend ... to Controversies between ... a State, or the Citizens thereof, and foreign States, Citizens or Subjects.”). The primary motivation for establishment of alienage jurisdiction in the Constitution was the difficulty British creditors had collecting from American debtors in state court prior to the Constitutional Convention. JPMorgan Chase Bank v. Traffic Stream (BVI) Infrastructure Ltd., 536 U.S. 88, 94–95, 122 S.Ct. 2054, 153 L.Ed.2d 95 (2002) (“This penchant of the state courts to disrupt international relations and discourage foreign investment led directly to the alienage jurisdiction provided by Article III of the Constitution.”); see also Koehler v. Bank of Bermuda (New York) Ltd., 229 F.3d 187, 188 (2d Cir. 2000) (“the fundamental purpose of alienage jurisdiction—to void offense to foreign nations”); Kevin R. Johnson, Why Alienage Jurisdiction? Historical Foundations and Modern Justifications for Federal Jurisdiction over Disputes Involving Noncitizens, 21 Yale J. Int'l L. 1, 15 (1996) (describing the origins of the alienage clause of the Constitution); Wythe Holt, The Origins of Alienage Jurisdiction, 14 Okla. City U. L. Rev. 547, 564 (1989) (“Alienage jurisdiction, as embodied in the Constitution and as put into action in the Judiciary Act of 1789, was ... an attempted solution to a major political problem of international dimensions.”). The Founders concluded that the burgeoning American economy could grow only if foreign companies had the “proper security” provided by the federal government's regularized review of contract disputes. See JPMorgan Chase Bank, 536 U.S. at 95–96, 122 S.Ct. 2054 (relying on contemporaneous statements of James Wilson, James Madison, and Alexander Hamilton).

The First Congress, in its attempt to further the goals emanating from these motivations, gave the federal courts jurisdiction of cases in which “an alien is a party” to litigation. See id. (relying on Judiciary Act of 1789, ch. 20, § 11, 1 Stat. 78). The Supreme Court, however, concluded that this jurisdictional grant did not include suits solely between two aliens because it exceeded the constitutional jurisdictional grant. Mossman v. Higginson, 4 U.S. 12, 14, 4 Dall. 12, 1 L.Ed. 720, (1800). “[T]he legislative power of conferring jurisdiction on the federal Courts is, in this respect, confined to suits between citizens and foreigners.” Id. (emphasis omitted).

Congress has since excepted from federal jurisdiction categories of cases involving alien litigants. Though the Constitution requires only “minimal diversity” among parties, State Farm Fire & Casualty Co. v. Tashire, 386 U.S. 523, 530–31, 87 S.Ct. 1199, 18 L.Ed.2d 270 (1967), Congress has required complete diversity to support federal jurisdiction, Herrick, 251 F.3d at 322. Complete diversity requires that aliens be on both sides of the litigation along with fully diverse domestic opposing parties. Bayerische Landesbank, New York Branch v. Aladdin Capital Mgmt. LLC, 692 F.3d 42, 49 (2d Cir. 2012) (“[W]e do not have diversity jurisdiction over cases between aliens. More specifically, ‘diversity is lacking ... where the only parties are foreign entities, or where on one side there are citizens and aliens and on the opposite side there are only aliens.’ ” (citation omitted)). In other circuits, courts adhere to a more traditional view denying diversity jurisdiction when there are aliens on both sides of a controversy, regardless of the presence of opposing domestic *206 parties. See Actions Involving Aliens—General Principles, 14A Fed. Prac. & Proc. Juris. § 3661 (4th ed.) (collecting cases).

The design of federal practice to protect both aliens and American citizens remains today. Transnational commercial interactions have become more commonplace and more complex. Litigating in state court could be burdensome, since foreign parties might be subject to the civil procedures of more than one jurisdiction. By contrast, litigating in a foreign court might subject United States parties to a system of civil procedure not yet fully developed, and in which it might not be possible to obtain personal jurisdiction over all necessary parties. It could be argued that it would be most prudent for the federal courts to have diversity jurisdiction of all cases in which an alien is a party so long as a citizen of the United States is also a party.

The court has considered the pleadings and the record and finds an antagonistic relationship requiring Kirin to remain a nominal defendant. Since Kirin is properly aligned as a defendant, there is complete diversity among the parties.

V. Conclusion

Defendant's motion to dismiss is denied. Kirin is properly aligned as a nominal defendant here. Since Plaintiff is a citizen of China and defendants are citizens of New York, there is complete diversity among the parties. The court has subject matter jurisdiction of Plaintiff's claims.

The parties are ordered to expedite discovery.

SO ORDERED.

 

SIDEBAR ON JUDGE WEINSTEIN:

The judge in this case, Jack B. Weinstein, was one of the most consequential and influential federal district court judges ever to serve. He presided over many mass tort matters, and was noted for his creative approaches to such cases. He helped innovate many of the techniques that allow courts to dispose of controversies with both many claimants and substantial damages. Born in 1921, he took the bench in 1967, served as an active judge until 1993, and continued to hear cases on Senior status until 2020. At the time he issued the opinion in this case he was 97 years old. 

7.3.3.2 Comments on Alienage Jurisdiction 7.3.3.2 Comments on Alienage Jurisdiction

     1. Special Issues With Alienage Jurisdiction. Alienage jurisdiction often is lumped in with diversity jurisdiction without a detailed consideration of the differences, but in a transnational setting we think it is worth taking a bit more of a look.  The Constitution refers to suits "between a state, or the citizens thereof, and foreign states, citizens or subjects." Section 1332(a)(2) makes the grant parallel to the clause before, which allows jurisdiction for suits between citizens of different states, by allowing suits between "citizens of a State and citizens or subjects of a foreign state." As Judge Weinstein notes, this was originally adopted as a concession to foreign investors, who wanted access to a US forum. 

          Citizens vs. Subjects. One question that sometimes arises in the minds of students is what difference, if any, there between a citizen of a  foreign state and a subject of a foreign state? As a practical matter, there is no difference - in either case, someone is treated as belonging to a foreign country. Historically, the difference seems to be one between monarchies and empires - such as China was in 1789, where those living in the state are the subjects of the king or emperor - and republics, such as China is today, where the state is made of its citizens.

          Alien vs US State Citizen. The alienage clause allows lawsuits between a foreign national and a citizen of a state to go forward with federal subject matter jurisdiction. For example, if a person resident in Shenzhen who is a citizen of the People's Republic of China brings a lawsuit for more than $75,000 against a resident of California, the alienage clause as implemented by Section 1332 provides a basis for federal subject matter jurisdiction.

          Alien vs Alien.  Note that nowhere in the Constitution or Section 1332 is there a provision for suits exclusively between citizens of foreign nations, even if those are different nations. Put differently, if a citizen of China wants to sue a citizen of Japan in US federal court on a state law claim, there will be no federal subject matter jurisdiction.

          Aliens With US Permanent Residency. The second part of Section 1332 (a)(2) addresses the situation where a foreign national has acquired permanent residency status (but not citizenship) in the United States and is domiciled in a particular US state. This happens not infrequently in the United States, where at our last look there were somewhere above 13 million foreign citizens holding permanent resident 'green cards.' In such situations, the alien is treated effectively as both an alien and a resident of a state in terms of duplications that would defeat federal jurisdiction. Some examples may help make this clear. Assume that a citizen of China has acquired permanent residency in California. This person - let's call her Zhang - wants to bring a lawsuit against a citizen of Canada. She cannot establish federal subject matter jurisdiction, because the court would treat that a suit between aliens, for which there is no provision for subject matter jurisdiction. Now, let's assume that Zhang wants to sue Evil, a resident of California. Once again, she cannot, because even though this is a lawsuit between a citizen of a foreign state and a resident of a US state, the language in section 1332 makes clear that there is no federal subject matter jurisdiction between a permanent resident alien living in California and resident of California. What if Zhang wants to sue Evil's cousin, Wicked, a US citizen who lives in New York? In that case, on these facts, there would be alienage subject matter jurisdiction.

          Incomplete Alienage Diversity. What if alienage jurisdiction exists between an alien and a citizen of a state, but an additional alien is added to the side that did not previously have an alien? (For example, NY v. Alien becomes NY + Alien v. Alien). Or, what if a citizen of state appears on both sides of the litigation, as would have happened here if the parties were realigned (Alien v. NY + NY becomes Alien + NY v. NY).  Judge Weinstein indicates that would defeat the necessary complete diversity, but wonders if this is the right policy choice given the breadth of the Constitutional language and the difficulties that might arise in litigating abroad. While this is parallel to the normal diversity rule, and while we suspect Judge Weinstein is correct, this is not something the Supreme Court has ever ruled on.

          Diversity Between State Citizens With Additional Aliens. Section 1332 (a)(3) addresses a different situation, where a diversity suit exists between citizens of different states, to which suit aliens are added. According to Wright and Miller, the language of this statute allows aliens on both sides of the case so long as the required diversity between states exists. Wright & Miller, Federal Practice and Procedure: Jurisdiction 2d § 3661.

          One issue remains unclear, however. What if one of the aliens is a permanent resident from the same state as one of the parties on the other side of the case? For example, imagine a Texan and a California resident alien bring a lawsuit against a Californian. Would there be subject matter jurisdiction? Section 1332(a)(2) indicates not, but Section 1332(a)(3) suggests so. We don't know.

          Citizens of a Foreign State and Foreign Sovereign Entities. Another issue that arises, particularly with respect to China, is whether certain corporations are corporate citizens of a foreign state or branches of the foreign state itself. This issue is beyond the scope of this course but may arise in the course of US litigation involving Chinese state-owned enterprises. Some such enterprises have asserted sovereign immunity as a defense against civil claims. If they are branches of sovereign states, that would impact the basis for federal jurisdiction. Whether a party is private or a foreign state entity also will affect issues such as venue and whether, absent a waiver, liability can be established. Addressing these issues in depth is beyond the scope of this course, but might be relevant to some practice situations someday, so if you are representing a Chinese state-owned enterprise in litigation in the United States just be aware that this is an issue you should look into.

     2. Realignment. In both diversity and alienage situations, realignment can be an issue. In the Hebei case, there was a dispute about where one of the parties belonged with regard to being on the same or different sides of the V. The defendant argued that the company, a New York resident, should be aligned as a plaintiff since the derivative suit was technically on the company's behalf. If this had been accepted, there would have been New York residents on both sides of the V, defeating alienage. Judge Weinstein found the company was under the de facto control of the other defendant, making it antagonistic to the plaintiff, and positioned it as a defendant. As a result, alienage jurisdiction existed. It's not unusual for defendants to claim that plaintiffs have manipulated the positioning of parties to create or defeat federal jurisdiction, and seek to have the parties realigned. As Wright & Miller notes:

In determining whether diversity of citizenship jurisdiction exists, the federal court is not bound by the way the plaintiff formally aligns the parties in his or her original pleading. It is the district judge's duty to “look beyond the pleadings, and arrange the parties according to their sides in the dispute,” and that is precisely what federal courts do.

Federal Practice and Procedure, Section 3607, Realignment of Parties.

7.3.3.3 Alienage Problems 7.3.3.3 Alienage Problems

     Again, the answers will be provided but see if you can sort this out on your own.

     Wendy Wu is a citizen of the People's Republic of China, but she has permanent US residency in the state of California.  She enters into a contract with Xavier Xu, a US citizen resident in Massachusetts, and Yoshi Yang, a US citizen resident in California. Xu and Yang breach the contract, and Wu wishes to bring a lawsuit seeking $100,000 each from Xu and Yang in federal court. Please analyze.

     Wendy Wu is a citizen of the People's Republic of China, but she has permanent US residency in the state of California.  She enters into a contract with Sven Swedenborg, a Swedish resident who has permanent residence in the state of New York. Swedenborg breaches, and she wishes to bring a lawsuit for $100,000 in federal court. Please analyze.

     Wendy Wu and Yoshi Yang resolve their dispute and become partners . Wu is still a Chinese citizen resident in California and Yang is still a US citizen and California resident. They enter into a contract with Otis Olafson, who promptly breaches. Olafson is a US citizen resident in Virginia. Wu and Yang both have claims against Olafson in excess of $100,000, which they wish to bring jointly in federal court. Please analyze.

     Assume that before suit is filed, Yang settles with Olafson. Wu still wishes to pursue her claim in federal court. Please analyze.

7.3.4 Amount-in-Controversy 7.3.4 Amount-in-Controversy

You will remember that this issue came up in the Mas case. How do we decide if the amount in controversy requirement has been met? We look at it again here.

7.3.4.1 JTH Tax, Inc. v. Frashier 7.3.4.1 JTH Tax, Inc. v. Frashier

JTH TAX, INCORPORATED, d/b/a Liberty Tax Service, Plaintiff-Appellant, v. Harry F. FRASHIER, II, Defendant-Appellee.

No. 09-2262.

United States Court of Appeals, Fourth Circuit.

Argued: Sept. 23, 2010.

Decided: Nov. 10, 2010.

*636ARGUED: David Andrew Lindley, Jr., Liberty Tax Service, Virginia Beach, Virginia, for Appellant. Dannielle C. Hall-Melvor, Taylor & Walker, PC, Norfolk, Virginia, for Appellee. ON BRIEF: Joseph Knowles, Liberty Tax Service, Virginia *637Beach, Virginia, for Appellant. Todd M. Gaynor, Taylor & Walker, PC, Norfolk, Virginia, for Appellee.

Before MOTZ, KING, and DAVIS, Circuit Judges.

Reversed by published opinion. Judge MOTZ wrote the opinion, in which Judge KING and Judge DAVIS joined.

OPINION

DIANA GRIBBON MOTZ, Circuit Judge:

JTH Tax, Inc. (“Liberty”) appeals from an order dismissing its complaint for lack of subject matter jurisdiction. Liberty contends that the district court erred in holding that its complaint failed to meet the $75,000 amount in controversy requirement for diversity jurisdiction under 28 U.S.C. § 1332(a). We agree and so reverse.

I.

Liberty franchises thousands of tax preparation offices nationwide. The dispute before us arises from its relationship with Harry Frashier, one of its franchisees. In 2006, Frashier signed a franchise agreement with Liberty granting him the right to operate Liberty Tax Service franchises in a designated area of West Virginia. In return, Frashier agreed to several post-termination provisions, including a covenant not to compete and a requirement that he return all customer lists and equipment to Liberty.

Frashier operated a Liberty franchise tax office without incident until 2008. On August 26 of that year, after Liberty had filed, but then dismissed, a lawsuit alleging Frashier’s breach of the agreement, Frashier offered to sell Liberty a right of first refusal for the purchase of Frashier’s franchise territory for $80,000. When the parties failed to agree on the terms of a sale, Frashier closed his franchise, which prompted Liberty to terminate its agreement with Frashier.

This dispute centers on what happened next. On January 28, 2009, Liberty filed a complaint in the Eastern District of Virginia, seeking $80,000 in damages and a permanent injunction compelling Frashier’s compliance with the post-termination provisions of the franchise agreement. Specifically, Liberty claimed that Frashier breached his post-termination duties by using his former office to support a competing tax enterprise and by failing to return the requisite materials to Liberty. Frashier responded that he merely leased office equipment and furniture to a different tax venture, actions he described as consistent with the agreement. He further claimed that he now offers only free tax preparation services to the indigent.

Liberty never amended its complaint, but in its subsequent motion for summary judgment, Liberty refined its damages calculation, seeking $60,456.25 in money damages and injunctive relief. The district court sua sponte dismissed Liberty’s complaint for failure to meet the $75,000 amount in controversy requirement for diversity jurisdiction. When Liberty timely moved for alteration of the judgment under Federal Rule of Civil Procedure 59(e), the court denied the motion.

Liberty then noted this appeal. We review de novo the judgment of the district court dismissing the complaint for lack of subject matter jurisdiction. See Pitt Cnty. v. Hotels.com, L.P., 553 F.3d 308, 311 (4th Cir.2009).*

*638II.

In most cases, the “sum claimed by the plaintiff controls” the amount in controversy determination. St. Paul Mercury Indem. Co. v. Red Cab Co., 303 U.S. 283, 288, 58 S.Ct. 586, 82 L.Ed. 845 (1938). If the plaintiff claims a sum sufficient to satisfy the statutory requirement, a federal court may dismiss only if “it is apparent, to a legal certainty, that the plaintiff cannot recover the amount claimed.” Id. at 289, 58 S.Ct. 586 (emphasis added).

Defendants, seeking dismissal of diversity actions for lack of a sufficient amount in controversy, must therefore shoulder a heavy burden. They must show “the legal impossibility of recovery” to be “so certain as virtually to negative the plaintiffs good faith in asserting the claim.” Wiggins v. N. Am. Equitable Life Assurance Co., 644 F.2d 1014, 1017 (4th Cir.1981) (internal quotation omitted). A mere dispute over the mathematical accuracy of a plaintiffs damages calculation does not constitute such a showing. See McDonald v. Patton, 240 F.2d 424, 425 (4th Cir.1957) (noting that plaintiffs may secure federal jurisdiction even when “it is apparent on the face of the claim” that the claim to the requisite amount is subject to a “valid defense”).

With these controlling principles in mind, we turn to the case at hand.

III.

Courts generally determine the amount in controversy by reference to the plaintiffs complaint. See Wiggins, 644 F.2d at 1016 (“Ordinarily the jurisdictional amount is determined by the amount of the plaintiffs original claim, provided that the claim is made in good faith.”). If the complaint in good faith alleges a sufficient amount in controversy, “[ejvents occurring subsequent” to the filing of the complaint “which reduce the amount recoverable below the statutory limit do not oust jurisdiction.” St. Paul Mercury, 303 U.S. at 289-90, 58 S.Ct. 586.

Here, Liberty’s complaint — which it has not amended — alleges $80,000 in damages, a sum sufficient to exceed the $75,000 amount necessary for diversity jurisdiction. Liberty’s later downward adjustment made in its motion for summary judgment (but not in any amended complaint) does not constitute a “subsequent reduction of the amount claimed” sufficient to “oust the district court’s jurisdiction.” Id. at 295, 58 S.Ct. 586; see also Griffin v. Red Run Lodge, Inc., 610 F.2d 1198, 1204 (4th Cir.1979). (holding that “[o]nce jurisdiction exists, subsequent events, such as the determination that one of the aggregated claims was without merit, do not destroy” jurisdiction).

In other words, jurisdiction turns not on ' the sum contained in Liberty’s summary judgment motion, but on the good faith of the allegation in its complaint of an adequate jurisdictional amount. The district court did not find, nor has Frashier even argued, that Liberty made a bad faith claim of $80,000 in its complaint. Accordingly, the complaint appears sufficient to allege an adequate jurisdictional amount.

IV.

To be sure, even a plaintiff whose complaint alleges a sufficient amount in controversy cannot secure jurisdiction “if, from the proofs, the court is satisfied to a [legal] certainty that the plaintiff never was entitled to recover that amount.” St. Paul Mercury, 303 U.S. at 289, 58 S.Ct. 586. But even if Liberty’s reassessment of its damages demonstrated to a legal certainty that it could recover only the $60,456.25 requested in its summary judg*639ment motion, dismissal for lack of jurisdiction would still constitute error here.

This is so because, like requests for money damages, requests for injunctive relief must be valued in determining whether the plaintiff has alleged a sufficient amount in controversy. See Hunt v. Wash. State Apple Adver. Comm’n, 432 U.S. 333, 347, 97 S.Ct. 2434, 53 L.Ed.2d 383 (1977) (“In actions seeking declaratory or injunctive relief, it is well established that the amount in controversy is measured by the value of the object of the litigation.”); Glenwood Light & Water Co. v. Mut. Light, Heat & Power Co., 239 U.S. 121, 125, 36 S.Ct. 30, 60 L.Ed. 174 (1915) (finding jurisdiction by looking at future value generated by injunction). Moreover, plaintiffs may aggregate smaller claims in order to reach the jurisdictional threshold. See Shanaghan v. Cahill, 58 F.3d 106, 109 (4th Cir.1995). Therefore, the district court should have considered not only the amount of money damages Liberty requested but also the injunctive relief it sought when determining jurisdiction.

Consideration of the requested injunctive relief compels the conclusion that Liberty’s claim alleges a sufficient amount in controversy. Even if the $60,456.25 alleged in its summary judgment motion constitutes the sole money damages sought by Liberty, its requested injunctive relief need only have a good faith worth of $14,543.76, i.e. the amount necessary to yield a combined value in excess of $75,000.

We ascertain the value of an injunction for amount in controversy purposes by reference to the larger of two figures: the injunction’s worth to the plaintiff or its cost to the defendant. See Dixon v. Edwards, 290 F.3d 699, 710 (4th Cir.2002). In this case, Liberty has demonstrated that the injunction, whether valued for the benefit it confers on Liberty or the detriment it imposes on Frashier, arguably yields a figure that exceeds the necessary jurisdictional amount.

With respect to the first, Liberty proposes two distinct ways of calculating the value of the injunction, both of which produce a figure well over $14,543.76. First, Liberty proposes adhering to its regular accounting practice of valuing franchises at 130% of the previous year’s net receipts. Using such a formula, it values Frashier’s former franchise, and thus the injunction forbidding his alleged improper use of that franchise, at $78,593.13. Second, it proposes a focus on the reputational value generated by the sought injunction. Under this approach, courts consider the ongoing diminution in Liberty’s market credibility allegedly caused by Frashier’s intransigence. See MultiChannel TV Cable Co. v. Charlottesville Quality Cable Operating Co., 22 F.3d 546, 552 (4th Cir.1994) (noting that “the potential loss of goodwill also supports] a finding of irreparable harm” for the purpose of granting injunctive relief). Liberty estimates the reputational value of the injunction to be $12,817,482 — the amount it spent on advertising in fiscal year 2009.

As for cost to Frashier, Liberty calculates the amount to be a minimum of $30,000. Liberty derives that figure from Frashier’s 5-year lease of his former Liberty office at $500 per month, arguing that an injunction prohibiting such a lease would cost Frashier $30,000 in lost profits.

We pass no judgment on the merits of any of these formulations. See United States v. North Carolina, 180 F.3d 574, 580 (4th Cir.1999) (noting that “[w]hen a factual attack on subject matter jurisdiction involves the merits of a dispute, the proper course of action ... is to find that jurisdiction exists and deal with the objection as a direct attack on the merits” (in*640ternal quotation and alteration omitted)). For our purposes, all that matters is that we cannot say with legal certainty that Liberty’s injunction is worth less than the requisite amount. Indeed, all of Liberty’s calculations employ reasoning that is at least facially plausible, and Frashier proposes no methodology of his own suggesting that the' injunction lacks the requisite value.

V.

For the foregoing reasons, the judgment of the district court is

REVERSED.

7.3.4.2 Notes on Amount In Controversy 7.3.4.2 Notes on Amount In Controversy

     1.  Origins of Amount in Controversy. The Constitution contains no amount in controversy provision, but since the Judiciary Act of 1789 provided the first statutory grant for diversity jurisdiction there has been a threshold amount. In 1789, it was $500 (equivalent to about $14,000 in 2020 dollars). The nominal amount has increased steadily over time, as the chart following shows. As you will learn, at one time there was also an amount in controversy requirement for federal question claims, but at present there is no amount in controversy requirement for such claims. The minimum amount spares the federal courts of adjudicating certain very low-value state law claims.

     2.  Must Exceed. The statute states that diversity jurisdiction exists only when the claim "exceeds the sum or value of $75,000, exclusive of interest and costs." What happens to the careless plaintiff who asserts a claim for exactly $75,000? The claim must be dismissed for lack of subject matter jurisdiction, a fate that would not have happened had the claim been for $75,000.01.

     3.  Time for Assessing Sufficiency of Amount in Controversy.  What happens if a claim is made for more than $75,000 but as the case develops it becomes clear that the plaintiff will not recover that much? As a general rule, if the original claim was in good faith, the court retains jurisdiction of the case. If there is a situation where it is clear from the outset that the claim cannot amount to more than $75,000 - say, for example, the plaintiff is suing for twelve months of $5,000 a month rent, with no penalties or other damages, and makes a conclusory allegation that the amount exceeds $75,000 - the court will do the math and find the jurisdictional allegation insufficient. On the other hand, if the original claim was for sixteen months' rent at $5,000 per month, but the defendant conclusively establishes that it should not have to pay for one of the months because the apartment was uninhabitable, that development does not divest the court of jurisdiction. If you think about it, this is the way it has to be - otherwise, every time a defendant wins, the court would lose jurisdiction, making the defendant's win void.  

     4.  Aggregation of claims by the same plaintiff against the same defendant. JPH Tax involved a situation where the argument was made that the claim for damages fell below the statutory threshold. The court then looked at the second claim, which in this case involves injunctive relief, and found that if the two claims were added together they would clearly be above the statutory minimum. This illustrates an aspect of diversity jurisdiction – claims by the same plaintiff can be added together to reach the statutory minimum. In this case, the claims arose from the same facts and circumstances, but for the purposes of aggregation, there does not need to be any connection between the claims which are added so long as the claims are brought by the same plaintiff. A plaintiff can bring a contract claim and unrelated tort claim and the court will add them together without worrying about any relationship between the claims other than that they were brought by the same party.  One interesting twist involves a situation where a plaintiff is asserting diversity claims worth less than $75,000 alongside an unrelated federal question claim that would bring the total to above $75,000 - can the federal question claim be included in the aggregation? The answer appears to be yes. See generally, 14A Charles A. Wright, et al., Federal Practice and Procedure § 3704. 

     5.  Valuing injunctive claims. Another aspect of JPH Tax involved a claim for injunctive relief. Typically, claims for injunctive relief do not involve a fixed dollar figure, unlike claims for damages. How are such claims valued for the purposes of assessing the statutory threshold? There are three possible methods. The court can look at the cost to the defendant to comply with the injunction. The court can look at the benefit to the plaintiff if the defendant complies with the injunction. Taking into account the different perspective of removal, the court can look to the value with regard to the person asserting federal jurisdiction -  the defendant if the case is removed, and the plaintiff if it is initially filed in federal court. Finally, the court can look at both the defendant's and plaintiff's perspectives, choosing the one which is higher. While perhaps the most common approach is to look to the value to the plaintiff, different courts have taken different approaches to this. If this issue arises in practice, you will need to see what rule applies where the case is located. See generally Brittain Shaw, The $75,000 Question: What Is the Value of Injunctive Relief?, 6 Geo. Mason L. Rev. 1013 (1998);  Christopher A. Pinahs, Diversity Jurisdiction and Injunctive Relief: Using a "Moving-Party Approach" to Value the Amount in Controversy, 95 Minn.L.Rev. 1930 (2011); 14A Charles A. Wright, et al., Federal Practice and Procedure § 3703.

     6.  Aggregation of claims by or against different parties. What happens when two plaintiffs join together to bring a lawsuit against the same defendant – can their claims be joined together in order to meet the statutory? In general, the answer is no. Each plaintiff needs to stand on their own. There is an exception when multiple plaintiffs are jointly asserting a common indivisible claim. The same logic applies when one plaintiff brings claims against two or more defendants - the claims cannot be aggregated, but one common claim that exceeds $75,000 will stand even if there are two defendants facing that claim. Some examples might help explain. Donnie Driver is careless and hits Pamela Plaintiff and her brother Paul Plaintiff as he drives through a parking lot. Each claims $40,000 in personal injury damages, arising from the same event. The claims cannot be aggregated as each plaintiff's claims and injuries are unique and separate - Paul does not feel Pamela's pain. On the other hand, imagine that Carl Claimant claims that he is the true owner of Blackacre, a relatively modest estate worth only $80,000. At present, the owners are listed as Omar Owner and his wife Olga Owner, who hold title jointly and undivided. Even though the average claim against each is only $40,000, the threshold will be met. Carl really has one claim, worth $80,000, and it is not reduced because there are two joint owners.  Similarly, multiple plaintiffs can join together "to enforce a single title or right, in which they have a common or undivided interest." Zahn v. International Paper Co., 414 U.S. 291, 294, 94 S. Ct. 505, 508 (1973). See generally, 14A Charles A. Wright, et al., Federal Practice and Procedure § 3704.  

     7.  Exclusive of Interest and Costs. You will note that the amount in controversy is "exclusive of interest and costs." What does this mean? In some cases, post-judgment interest can be awarded. For example, under some applicable laws interest runs from the time of the judgment until collection. This kind of interest is excluded. On the other hand, sometimes interest is an essential part of the claim, and is included. For example, if a plaintiff sues on a bond and claims that interest payments were not made as scheduled, that kind of interest is not excluded. The lawsuit is about collecting the interest. Costs are those costs awarded as part of litigation. For example, costs of service of process might be awarded to the prevailing party. Usually, in the American system, these costs are not substantial relative to the claim on the merits, and in any event they don't count toward the amount in controversy. One issue that can get technical, and so is a bit beyond our scope, is the awarding of attorney's fees to the prevailing party. In some cases a statute will include an award of attorneys' fees, and in some cases a claim for those fees can be included in the amount in controversy. For the purposes of this course, be aware, but don't worry about it.

 

7.3.4.4 Amount in Controversy Problems 7.3.4.4 Amount in Controversy Problems

     Paula Plaintiff wishes to bring a federal lawsuit against her 'frenemy' Debbie Defendo. Plaintiff had signed a contract to do some work for Defendo, and when she was finished Defendo claimed the quality was no good and refused to pay her any of the $25,000 due under the terms of the contract. Plaintiff publicly maintains that her work was 'perfect' and that the full sum is owning, but has privately admitted that perhaps some small offset for deficiencies might be reasonable. While they were discussing the problem with the contract, Defendo physically pushed Plaintiff. Plaintiff was not physically injured but she was intimidated and scared. Her attorney has researched similar cases and feels a claim for $50,000 is the most that can be made in good faith for such an incident and intends to claim for exactly that amount in addition to the $25,000 due under the contract. Privately, he feels he would be extremely lucky to win $5,000 for an assault where there was no injury. Finally, Plaintiff has remembered that three years ago she loaned $1 to Defendo so she could buy a Coca-Cola from a vending machine. She intends to sue for that as well, even though it is totally unrelated. If asserted, the local statute of limitations would almost certainly bar this claim. Plaintiff wants to bring the three claims in federal court (she and Defendo are diverse). Please analyze the amount-in-controversy issues.

     Ima Claimant wishes to bring a lawsuit against Ben Sued. The first claim is for $50,000, and arises from a state law claim. The second claim is completely unrelated, and arises from a federal cause of action. This claim would be for $30,000. The parties are diverse. Please analyze the amount-in-controversy issues.

     Professor Evil has become quite old, and no long can ride or control his bicycle. Instead, he rides an adult tricycle around his retirement community in Florida, which he also has difficulty controlling. One day, two former students - Shiza Great and Summa Laudey - are coming to visit Evil when he comes suddenly around a corner and runs them down in the same accident at the exact same time. Reluctantly, they bring suit against their old professor in federal court. (Both are from New York). Each sues Evil for $50,000. Please analyze the amount-in-controversy issues.

7.3.5 Diversity and Alienage Jurisdiction Reviewed 7.3.5 Diversity and Alienage Jurisdiction Reviewed

7.3.5.1 Final Notes on Diversity and Alienage Jurisdiction 7.3.5.1 Final Notes on Diversity and Alienage Jurisdiction

     1.  Realignment. On occasion, it can be hard to tell whether a party should be considered a plaintiff or a defendant. We saw that in Hebei Tiankai Wood & Land Constr. Ltd. v. Chen, where there was some discussion about where a party should be aligned. On other occasions, parties may artfully plead someone as a plaintiff or a defendant despite their real interests in order to create or deny the appearance of diversity jurisdiction. As Hebei Tiankai Wood & Land Constr. Ltd. v. Chen indicates, courts can and will realign parties in line with their actual interests in determining jurisdiction.

     2. Collusive Joinder. Another issue that can arise is 'collusive joinder.' An example would be where one party assigns its claim to another party from a different jurisdiction so as to affect the existence of federal subject matter jurisdiction. This is addressed by 28 USC Section 1359 which provides that the federal court has no jurisdiction in cases “in which any party, by assignment or otherwise, has been improperly or collusively made or joined to invoke the jurisdiction of such court.” Some assignments, of course, are legitimate and have nothing to do with invoking jurisdiction. The leading Supreme Court case to deal with Section 1359, Kramer v. Caribbean Mills, Inc., 394 U.S. 823 (1969), instructs the court to look at the totality of the circumstances in deciding whether an assignment was improper or collusive. If it is, it is disregarded for the purpose of establishing or denying federal subject matter jurisdiction (although it may be a valid assignment for other purposes).

     3.  Judicially Created Exceptions to Diversity and Alienage Jurisdiction. As you have seen, Congress has narrowed the reach of both diversity and alienage jurisdiction relative to what is permitted in the Constitution. As it turns out, there are also some judicially created exceptions, even though neither the Constitution nor the enabling statutes require or even hint at these exceptions. One has to do with family law or 'domestic relations' as the exception is known. This does not mean that no matters involving families can invoke diversity. What it means is that family specific litigation such as divorce, alimony, and child custody cannot be heard pursuant to diversity or alienage jurisdiction. Ankenbrandt v. Richards, 504 U.S. 689, 704 (1992). Another exception is probate. Probate is the process through which a person's belongings are distributed upon his or her death through the execution of the person's will. This also has traditionally been an area of state jurisdiction that does not come into the federal courts through diversity.

     4.  Framework. By now you should have all the diversity and alienage issues crisply organized in your mind. Issues you will want to work through are:

  • Where are the parties domiciled?
    • Individuals
    • Corporations
    • LLCs, Partnerships, and other joint non-incorporated entities
    • Aliens resident abroad
    • Aliens permanently resident in the United States
  • What is the amount in controversy?
    • Single damage claims
    • Aggregated damage claims between one plaintiff and one defendant
    • Injunctive relief
    • Claims aggregated among multiple parties
    • Interest (distinguishing collecting missed interest payments which are the basis for the claim and interest added either prejudgment or post-judgment to a recovery based on other damages), costs (note the special case of awardable attorneys' fees), and other excludable damages
  • Assessing sufficiency of jurisdiction
    • Time
    • Pleading
    • Burden of Proof
  • Special Considerations
    • Realignment
    • Collusion
    • Domestic Relations
    • Probate

7.4 Federal Question Jurisdiction 7.4 Federal Question Jurisdiction

In this section, we look at jurisdiction where the basis for a claim is federal law, as opposed to state law. Federal law includes the Constitution, the statutes passed into law by Congress, and treaties to which the US is subject.

7.4.1 Title 28: § 1331 7.4.1 Title 28: § 1331

The district courts shall have original jurisdiction of all civil actions arising under the Constitution, laws, or treaties of the United States.

7.4.2 Louisville & Nashville R. Co. v. Mottley 7.4.2 Louisville & Nashville R. Co. v. Mottley

211 U.S. 149 (1908)

LOUISVILLE AND NASHVILLE RAILROAD COMPANY
v.
MOTTLEY.

No. 37.

Supreme Court of United States.

Argued October 13, 1908.
Decided November 16, 1908.

APPEAL FROM THE CIRCUIT COURT OF THE UNITED STATES FOR THE WESTERN DISTRICT OF KENTUCKY.

[151] Mr. Henry Lane Stone for appellant.

Mr. Lewis McQuown and Mr. Clarence U. McElroy for appellees.

By leave of court, Mr. L.A. Shaver, in behalf of The Interstate Commerce Commission, submitted a brief as amicus curioe.

MR. JUSTICE MOODY, after making the foregoing statement, delivered the opinion of the court.

Two questions of law were raised by the demurrer to the bill, were brought here by appeal, and have been argued before us. They are, first, whether that part of the act of Congress of June 29, 1906 (34 Stat. 584), which forbids the giving of free passes or the collection of any different compensation for transportation of passengers than that specified in the tariff filed, makes it unlawful to perform a contract for transportation of persons, who in good faith, before the passage of the act, had accepted such contract in satisfaction of a valid cause of action against the railroad; and, second, whether the statute, if it should be construed to render such a contract unlawful, is in [152] violation of the Fifth Amendment of the Constitution of the United States. We do not deem it necessary, however, to consider either of these questions, because, in our opinion, the court below was without jurisdiction of the cause. Neither party has questioned that jurisdiction, but it is the duty of this court to see to it that the jurisdiction of the Circuit Court, which is defined and limited by statute, is not exceeded. This duty we have frequently performed of our own motion. Mansfield, &c.; Railway Company v. Swan, 111 U.S. 379, 382; King Bridge Company v. Otoe County, 120 U.S. 225; Blacklock v. Small, 127 U.S. 96, 105; Cameron v. Hodges, 127 U.S. 322, 326; Metcalf v. Watertown, 128 U.S. 586, 587; Continental National Bank v. Buford, 191 U.S. 119.

There was no diversity of citizenship and it is not and cannot be suggested that there was any ground of jurisdiction, except that the case was a "suit . . . arising under the Constitution and laws of the United States." Act of August 13, 1888, c. 866, 25 Stat. 433, 434. It is the settled interpretation of these words, as used in this statute, conferring jurisdiction, that a suit arises under the Constitution and laws of the United States only when the plaintiff's statement of his own cause of action shows that it is based upon those laws or that Constitution. It is not enough that the plaintiff alleges some anticipated defense to his cause of action and asserts that the defense is invalidated by some provision of the Constitution of the United States. Although such allegations show that very likely, in the course of the litigation, a question under the Constitution would arise, they do not show that the suit, that is, the plaintiff's original cause of action, arises under the Constitution. In Tennessee v. Union & Planters' Bank, 152 U.S. 454, the plaintiff, the State of Tennessee, brought suit in the Circuit Court of the United States to recover from the defendant certain taxes alleged to be due under the laws of the State. The plaintiff alleged that the defendant claimed an immunity from the taxation by virtue of its charter, and that therefore the tax was void, because in violation of the provision of the Constitution of the United [153] States, which forbids any State from passing a law impairing the obligation of contracts. The cause was held to be beyond the jurisdiction of the Circuit Court, the court saying, by Mr. Justice Gray (p. 464), "a suggestion of one party, that the other will or may set up a claim under the Constitution or laws of the United States, does not make the suit one arising under that Constitution or those laws." Again, in Boston & Montana Consolidated Copper & Silver Mining Company v. Montana Ore Purchasing Company, 188 U.S. 632, the plaintiff brought suit in the Circuit Court of the United States for the conversion of copper ore and for an injunction against its continuance. The plaintiff then alleged, for the purpose of showing jurisdiction, in substance, that the defendant would set up in defense certain laws of the United States. The cause was held to be beyond the jurisdiction of the Circuit Court, the court saying, by Mr. Justice Peckham (pp. 638, 639).

"It would be wholly unnecessary and improper in order to prove complainant's cause of action to go into any matters of defence which the defendants might possibly set up and then attempt to reply to such defence, and thus, if possible, to show that a Federal question might or probably would arise in the course of the trial of the case. To allege such defence and then make an answer to it before the defendant has the opportunity to itself plead or prove its own defence is inconsistent with any known rule of pleading so far as we are aware, and is improper.

"The rule is a reasonable and just one that the complainant in the first instance shall be confined to a statement of its cause of action, leaving to the defendant to set up in his answer what his defence is and, if anything more than a denial of complainant's cause of action, imposing upon the defendant the burden of proving such defence.

"Conforming itself to that rule the complainant would not, in the assertion or proof of its cause of action, bring up a single Federal question. The presentation of its cause of action would not show that it was one arising under the Constitution or laws of the United States.

[154] "The only way in which it might be claimed that a Federal question was presented would be in the complainant's statement of what the defence of defendants would be and complainant's answer to such defence. Under these circumstances the case is brought within the rule laid down in Tennessee v. Union & Planters' Bank, 152 U.S. 454. That case has been cited and approved many times since, . . ."

The interpretation of the act which we have stated was first announced in Metcalf v. Watertown, 128 U.S. 586, and has since been repeated and applied in Colorado Central Consolidated Mining Company v. Turck, 150 U.S. 138, 142; Tennessee v. Union & Planters' Bank, 152 U.S. 454, 459; Chappell v. Waterworth, 155 U.S. 102, 107; Postal Telegraph Cable Company v. Alabama, 155 U.S. 482, 487; Oregon Short Line & Utah Northern Railway Company v. Skottowe, 162 U.S. 490, 494; Walker v. Collins, 167 U.S. 57, 59; Muse v. Arlington Hotel Company, 168 U.S. 430, 436; Galveston &c.; Railway v. Texas, 170 U.S. 226, 236; Third Street & Suburban Railway Company v. Lewis, 173 U.S. 457, 460; Florida Central & Peninsular Railroad Company v. Bell, 176 U.S. 321, 327; Houston & Texas Central Railroad Company v. Texas, 177 U.S. 66, 78; Arkansas v. Kansas & Texas Coal Company & San Francisco Railroad, 183 U.S. 185, 188; Vicksburg Waterworks Company v. Vicksburg, 185 U.S. 65, 68; Boston & Montana Consolidated Copper & Silver Mining Company v. Montana Ore Purchasing Company, 188 U.S. 632, 639; Minnesota v. Northern Securities Company, 194 U.S. 48, 63; Joy v. City of St. Louis, 201 U.S. 332, 340; Devine v. Los Angeles, 202 U.S. 313, 334. The application of this rule to the case at bar is decisive against the jurisdiction of the Circuit Court.

It is ordered that the

Judgment be reversed and the case remitted to the Circuit Court with instructions to dismiss the suit for want of jurisdiction.

7.4.3 Notes on L&N R. Co. v. Mottley: The "Face of the Complaint" Rule 7.4.3 Notes on L&N R. Co. v. Mottley: The "Face of the Complaint" Rule

     1.  The Face of the Complaint or Well-Pleaded Complaint Rule. In the Mottley case, federal issues were certain to be important to the outcome.  The statute relied on by the railroad to cut off their free passes was a federal law; in return, the Mottleys were sure to cite to the Constitution's prohibition of taking away property, such as free passes, without due process of law.  No one seemed to dispute that absent the federal law the Mottleys' passes were valid and should be honored, making the state law right background to the real dispute in the case. Despite the likely centrality of federal issues to the case, the Court held that federal subject matter jurisdiction did not lie under Section 1331.

    The rule the Court announced has come to be known as the "face of the complaint" or the "well-pleaded complaint" rule. Put simply, the claim for relief has to be based on a federal question. In Mottley, the claim for relief was based on state contract law. The federal law came in as a defense, but the court held that federal questions that come in as defenses are not enough to invoke federal question jurisdiction. You will find that the same is true for counterclaims asserted by the defendant back against the plaintiff, or for cross-claims asserted between parties aligned on the same side of the case.

     One way to think about the Mottley rule has to do with the idea of critical path. A critical path is the steps that have to be taken in order for a given result to be reached. For a case to arise under the federal question statute, resolving the federal question has to be on the critical path. If there is a path to a plaintiff's recovery that does not require decision of the federal question, it fails the face of the complaint rule. In Mottley, recovery could have been obtained on the state law issue alone.

     2.  Constitutional Versus Statutory Standard for Sufficiency of Federal Questions.  Mottley involved an interpretation of Section 1331, not the Constitution. Interestingly, there was no federal question statute until after the Civil War, when the first version of Section 1331 was enacted, in part to give freedmen and former slaves access to federal courts if their federal rights were interfered with. As was true with diversity, the Constitutional standard is broader than the statutory standard. So far as the Constitution is concerned, a case or controversy meets the federal question standard if the federal issue appears likely to arise in the case. Osborn v. Bank of the United States, 22 U.S. (9 Wheat.) 738 (1824).

     3.  Explicit and Implied Federal Causes of Action. Not all federal (or state) statutes create a right for private litigants to bring a lawsuit. Sometimes, for example, only an administrative or regulatory agency is empowered to enforce the law. The explicit creation (by Congress) or the implication from statutory silence (by courts) of a private right of action dramatically shifts the enforcement of the law from agencies to private litigants. While, in general, such rights of action are consistent with the US practice of ex-post regulation through litigation, the creation and existence of such rights of actions often is controversial. Implication is even more controversial than Congressional creation of private rights as it involves courts sometimes choosing to find implied a cause of action that Congress had chosen not to create explicitly. 

     4.  Exceptions to the Face of the Complaint Rule. In general, the face of the complaint rule is not elusive. If the case is filed pursuant to a federal law and a federal question must be resolved for the plaintiff to recover, the face of the complaint rule is satisfied. There are, however, some exceptions. Some won't concern us - for example, in Shoshone Mining Co. v. Rutter, 177 U.S. 505 (1900), a federal statute created a right for miners to file patents on claims and to settle conflicting claims, but left resolution of the claim to local customs and laws. The Court held that as federal law was not necessarily involved, the case was not one which "necessarily arises under the Constitution and law of the United States." Another exception, discussed in more detail below, involves the situation where a state statute references and incorporates federal law.

7.4.4 Federal Questions Embedded in a State Statutes 7.4.4 Federal Questions Embedded in a State Statutes

     What happens when a state statute references and thereby incorporates a federal law? Does federal question jurisdiction exist (albeit created by a state and imposed on the federal courts) or is it just a state statute providing no direct access to federal court (albeit one that will require courts to build precedent interpreting a federal statute)?

     In these cases, it can be argued, resolving the federal question is essential for imposing liability. At the same time, the law at base is a state law, created by a state government, and one may ask whether a state government should have the power to throw open the doors to federal courthouses. In addition, in some cases bringing such cases into federal court could federalize issues that traditionally have been resolved by state courts, potentially burdening the federal courts. We've run out of hands, but if we had one more we would note that in some cases the federal question involved is quite important, and perhaps one that deserves uniform federal resolution, rather than resolution by the various state courts. When can we say that such claims 'arise under' federal law so as to satisfy § 1331?

     One straightforward answer was given in a case authored by Justice Oliver Wendell Holmes in Americal Well Works Co. v. Layne & Bowler Co., 242 U.S. 257 (1916). American Well Works claimed that Layne & Bowler had interfered maliciously with their business by falsely claiming that their pump violated a patent held by Layne & Bowler. The trade libel statute was created under state law but the case would require a determination as to whether American Well Works' pump did in fact violate the patent held by Layne & Bowler, which would require interpreting federal law. (Can you see how this is different from Mottley? In Mottley, did the plaintiffs' theory of the case necessarily require an interpretation of federal law?). Justice Holmes wrote:

A suit arises under the law that creates the cause of action. The fact that the justification may involve the validity and infringement of a patent is no more material to the question under what law the suit is brought than it would be in an action of contract.

     Five years later, the Court went in a different direction. In Smith v. Kansas City Title & Trust Co., 255 U.S. 180 (1921), the plaintiff claimed under a state law prohibiting corporations from investing in unlawfully issued investments that the corporation was buying bonds unconstitutionally issued under the Federal Farm Loan Act. In examining this seemingly similar case, the Court put forth a different standard:

The general rule is that, where it appears from the bill or statement of the plaintiff that the right to relief depends upon the construction or application of the Constitution or laws of the United States, and that such federal claim is not merely colorable, and rests upon a reasonable foundation, the District Court has jurisdiction under this provision.

    The Court zigged back 13 years later in deciding Moore v. Chesapeake & Ohio Railway, 291 U.S., 205 (1934). In that case the suit was brought under a state employer's liability law; because contributory negligence could not be asserted if the employer had violated a law enacted for "the safety of employees" the case would turn on whether the employer had violated the Federal Safety Appliance Act. The court held no federal jurisdiction existed.

     Similarly, in Gully v., First National Bank in Meridian, 299 U.S. 109 (1936), a state wanted to tax a national bank. While a federal statute ultimately controlled whether a national bank could be taxed, the Court rejected federal jurisdiction on the grounds that the federal issue was not central to the case but was only "lurking" and might not be reached.

     More recently, in Merrell Dow Pharmaceuticals, Inc., v. Thompson, 478 U.S. 804 (1986), the plaintiffs argued that a violation of the Federal Food, Drug, and Cosmetic Act established negligence under state law. The claim, similar to the cases above, was that even though the cause of action was created by a state the interpretation of federal law would prove essential to the relevant count of the complaint based on the FDCA violation. An important issue in the case was that Congress did not create an explicit private right of action under the FDCA, and the parties agreed that there was not an implied one. That means, if the case was treated as federal law, a federal forum for violations of the FDCA would be provided in federal court where Congress had not explicitly provided one and courts had not found one implied. The practical impact would be to open the federal courts to a large number of tort claims that otherwise would be in state court. Relying heavily on the lack of federal private right of action, the Court concluded, "We conclude that a complaint alleging a violation of a federal statute as an element of a state cause of action, when Congress has determined that there should be no private, federal cause of action for the violation, does not state a claim 'arising under the Constitution, laws, or treaties of the United States.'" A forceful dissent by Justice Brennan argued, in part, that it would be better to have federal courts engaged in defining and applying the important federal statute involved.

      This set the stage for the case that follows.

     

     

7.4.5 Grable & Sons Metal Products, Inc. v. Darue Engineering & Mfg. 7.4.5 Grable & Sons Metal Products, Inc. v. Darue Engineering & Mfg.

545 U.S. 308 (2005)

GRABLE & SONS METAL PRODUCTS, INC.
v.
DARUE ENGINEERING & MANUFACTURING

No. 04-603.

Supreme Court of United States.

Argued April 18, 2005.
Decided June 13, 2005.

CERTIORARI TO THE UNITED STATES COURT OF APPEALS FOR THE SIXTH CIRCUIT

[309] SOUTER, J., delivered the opinion for a unanimous Court. THOMAS, J., filed a concurring opinion, post, p. 320.

Eric H. Zagrans argued the cause for petitioner. On the briefs was Charles E. McFarland.

[310] Michael C. Walton argued the cause for respondent. With him on the brief were John M. Lichtenberg, Gregory G. Timmer, and Mary L. Tabin.

Irving L. Gornstein argued the cause for the United States as amicus curiae urging affirmance. With him on the brief were Acting Solicitor General Clement, Assistant Attorney General O'Connor, Deputy Solicitor General Hungar, and Gilbert S. Rothenberg.[1]

JUSTICE SOUTER delivered the opinion of the Court.

The question is whether want of a federal cause of action to try claims of title to land obtained at a federal tax sale precludes removal to federal court of a state action with nondiverse parties raising a disputed issue of federal title law. We answer no, and hold that the national interest in providing a federal forum for federal tax litigation is sufficiently substantial to support the exercise of federal-question jurisdiction over the disputed issue on removal, which would not distort any division of labor between the state and federal courts, provided or assumed by Congress.

I

In 1994, the Internal Revenue Service seized Michigan real property belonging to petitioner Grable & Sons Metal Products, Inc., to satisfy Grable's federal tax delinquency. Title 26 U. S. C. § 6335 required the IRS to give notice of the seizure, and there is no dispute that Grable received actual notice by certified mail before the IRS sold the property to respondent Darue Engineering & Manufacturing. Although Grable also received notice of the sale itself, it did not exercise its statutory right to redeem the property within 180 days of the sale, § 6337(b)(1), and after that period [311] had passed, the Government gave Darue a quitclaim deed, § 6339.

Five years later, Grable brought a quiet title action in state court, claiming that Darue's record title was invalid because the IRS had failed to notify Grable of its seizure of the property in the exact manner required by § 6335(a), which provides that written notice must be "given by the Secretary to the owner of the property [or] left at his usual place of abode or business." Grable said that the statute required personal service, not service by certified mail.

Darue removed the case to Federal District Court as presenting a federal question, because the claim of title depended on the interpretation of the notice statute in the federal tax law. The District Court declined to remand the case at Grable's behest after finding that the "claim does pose a `significant question of federal law," Tr. 17 (Apr. 2, 2001), and ruling that Grable's lack of a federal right of action to enforce its claim against Darue did not bar the exercise of federal jurisdiction. On the merits, the court granted summary judgment to Darue, holding that although § 6335 by its terms required personal service, substantial compliance with the statute was enough. 207 F. Supp. 2d 694 (WD Mich. 2002).

The Court of Appeals for the Sixth Circuit affirmed. 377 F. 3d 592 (2004). On the jurisdictional question, the panel thought it sufficed that the title claim raised an issue of federal law that had to be resolved, and implicated a substantial federal interest (in construing federal tax law). The court went on to affirm the District Court's judgment on the merits. We granted certiorari on the jurisdictional question alone,[2] 543 U. S. 1042 (2005), to resolve a split within the Courts of Appeals on whether Merrell Dow Pharmaceuticals Inc. v. Thompson, 478 U. S. 804 (1986), always requires [312] a federal cause of action as a condition for exercising federal-question jurisdiction.[3] We now affirm.

II

Darue was entitled to remove the quiet title action if Grable could have brought it in federal district court originally, 28 U. S. C. § 1441(a), as a civil action "arising under the Constitution, laws, or treaties of the United States," § 1331. This provision for federal-question jurisdiction is invoked by and large by plaintiffs pleading a cause of action created by federal law (e. g., claims under 42 U. S. C. § 1983). There is, however, another longstanding, if less frequently encountered, variety of federal "arising under" jurisdiction, this Court having recognized for nearly 100 years that in certain cases federal-question jurisdiction will lie over state-law claims that implicate significant federal issues. E. g., Hopkins v. Walker, 244 U. S. 486, 490-491 (1917). The doctrine captures the commonsense notion that a federal court ought to be able to hear claims recognized under state law that nonetheless turn on substantial questions of federal law, and thus justify resort to the experience, solicitude, and hope of uniformity that a federal forum offers on federal issues, see ALI, Study of the Division of Jurisdiction Between State and Federal Courts 164-166 (1968).

The classic example is Smith v. Kansas City Title & Trust Co., 255 U. S. 180 (1921), a suit by a shareholder claiming that the defendant corporation could not lawfully buy certain bonds of the National Government because their issuance was unconstitutional. Although Missouri law provided the cause of action, the Court recognized federal-question jurisdiction because the principal issue in the case was the federal constitutionality of the bond issue. Smith thus held, in a [313] somewhat generous statement of the scope of the doctrine, that a state-law claim could give rise to federal-question jurisdiction so long as it "appears from the [complaint] that the right to relief depends upon the construction or application of [federal law]." Id., at 199.

The Smith statement has been subject to some trimming to fit earlier and later cases recognizing the vitality of the basic doctrine, but shying away from the expansive view that mere need to apply federal law in a state-law claim will suffice to open the "arising under" door. As early as 1912, this Court had confined federal-question jurisdiction over state-law claims to those that "really and substantially involv[e] a dispute or controversy respecting the validity, construction or effect of [federal] law." Shulthis v. McDougal, 225 U. S. 561, 569. This limitation was the ancestor of Justice Cardozo's later explanation that a request to exercise federal-question jurisdiction over a state action calls for a "common-sense accommodation of judgment to [the] kaleidoscopic situations" that present a federal issue, in "a selective process which picks the substantial causes out of the web and lays the other ones aside." Gully v. First Nat. Bank in Meridian, 299 U. S. 109, 117-118 (1936). It has in fact become a constant refrain in such cases that federal jurisdiction demands not only a contested federal issue, but a substantial one, indicating a serious federal interest in claiming the advantages thought to be inherent in a federal forum. E. g., Chicago v. International College of Surgeons, 522 U. S. 156, 164 (1997); Merrell Dow, supra, at 814, and n. 12; Franchise Tax Bd. of Cal. v. Construction Laborers Vacation Trust for Southern Cal., 463 U. S. 1, 28 (1983).

But even when the state action discloses a contested and substantial federal question, the exercise of federal jurisdiction is subject to a possible veto. For the federal issue will ultimately qualify for a federal forum only if federal jurisdiction is consistent with congressional judgment about the sound division of labor between state and federal courts governing [314] the application of § 1331. Thus, Franchise Tax Bd. explained that the appropriateness of a federal forum to hear an embedded issue could be evaluated only after considering the "welter of issues regarding the interrelation of federal and state authority and the proper management of the federal judicial system." Id., at 8. Because arising-under jurisdiction to hear a state-law claim always raises the possibility of upsetting the state-federal line drawn (or at least assumed) by Congress, the presence of a disputed federal issue and the ostensible importance of a federal forum are never necessarily dispositive; there must always be an assessment of any disruptive portent in exercising federal jurisdiction. See also Merrell Dow, supra, at 810.

These considerations have kept us from stating a "single, precise, all-embracing" test for jurisdiction over federal issues embedded in state-law claims between nondiverse parties. Christianson v. Colt Industries Operating Corp., 486 U. S. 800, 821 (1988) (STEVENS, J., concurring). We have not kept them out simply because they appeared in state raiment, as Justice Holmes would have done, see Smith, supra, at 214 (dissenting opinion), but neither have we treated "federal issue" as a password opening federal courts to any state action embracing a point of federal law. Instead, the question is, does a state-law claim necessarily raise a stated federal issue, actually disputed and substantial, which a federal forum may entertain without disturbing any congressionally approved balance of federal and state judicial responsibilities.

III

A

This case warrants federal jurisdiction. Grable's state complaint must specify "the facts establishing the superiority of [its] claim," Mich. Ct. Rule 3.411(B)(2)(c) (West 2005), and Grable has premised its superior title claim on a failure by the IRS to give it adequate notice, as defined by federal [315] law. Whether Grable was given notice within the meaning of the federal statute is thus an essential element of its quiet title claim, and the meaning of the federal statute is actually in dispute; it appears to be the only legal or factual issue contested in the case. The meaning of the federal tax provision is an important issue of federal law that sensibly belongs in a federal court. The Government has a strong interest in the "prompt and certain collection of delinquent taxes," United States v. Rodgers, 461 U. S. 677, 709 (1983), and the ability of the IRS to satisfy its claims from the property of delinquents requires clear terms of notice to allow buyers like Darue to satisfy themselves that the Service has touched the bases necessary for good title. The Government thus has a direct interest in the availability of a federal forum to vindicate its own administrative action, and buyers (as well as tax delinquents) may find it valuable to come before judges used to federal tax matters. Finally, because it will be the rare state title case that raises a contested matter of federal law, federal jurisdiction to resolve genuine disagreement over federal tax title provisions will portend only a microscopic effect on the federal-state division of labor. See n. 3, infra.

This conclusion puts us in venerable company, quiet title actions having been the subject of some of the earliest exercises of federal-question jurisdiction over state-law claims. In Hopkins, 244 U. S., at 490-491, the question was federal jurisdiction over a quiet title action based on the plaintiffs' allegation that federal mining law gave them the superior claim. Just as in this case, "the facts showing the plaintiffs' title and the existence and invalidity of the instrument or record sought to be eliminated as a cloud upon the title are essential parts of the plaintiffs' cause of action."[4]Id., at [316] 490. As in this case again, "it is plain that a controversy respecting the construction and effect of the [federal] laws is involved and is sufficiently real and substantial." Id., at 489. This Court therefore upheld federal jurisdiction in Hopkins, as well as in the similar quiet title matters of Northern Pacific R. Co. v. Soderberg, 188 U. S. 526, 528 (1903), and Wilson Cypress Co. v. Del Pozo y Marcos, 236 U. S. 635, 643-644 (1915). Consistent with those cases, the recognition of federal jurisdiction is in order here.

B

Merrell Dow Pharmaceuticals Inc. v. Thompson, 478 U. S. 804 (1986), on which Grable rests its position, is not to the contrary. Merrell Dow considered a state tort claim resting in part on the allegation that the defendant drug company had violated a federal misbranding prohibition, and was thus presumptively negligent under Ohio law. Id., at 806. The Court assumed that federal law would have to be applied to resolve the claim, but after closely examining the strength of the federal interest at stake and the implications of opening the federal forum, held federal jurisdiction unavailable. Congress had not provided a private federal cause of action for violation of the federal branding requirement, and the Court found "it would . . . flout, or at least undermine, congressional intent to conclude that federal courts might nevertheless exercise federal-question jurisdiction and provide remedies for violations of that federal statute solely because the violation . . . is said to be a . . . `proximate cause' under state law." Id., at 812.

[317] Because federal law provides for no quiet title action that could be brought against Darue,[5] Grable argues that there can be no federal jurisdiction here, stressing some broad language in Merrell Dow (including the passage just quoted) that on its face supports Grable's position, see Note, Mr. Smith Goes to Federal Court: Federal Question Jurisdiction over State Law Claims Post-Merrell Dow, 115 Harv. L. Rev. 2272, 2280-2282 (2002) (discussing split in Courts of Appeals over private right of action requirement after Merrell Dow). But an opinion is to be read as a whole, and Merrell Dow cannot be read whole as overturning decades of precedent, as it would have done by effectively adopting the Holmes dissent in Smith, see supra, at 314, and converting a federal cause of action from a sufficient condition for federal-question jurisdiction[6] into a necessary one.

In the first place, Merrell Dow disclaimed the adoption of any bright-line rule, as when the Court reiterated that "in exploring the outer reaches of § 1331, determinations about federal jurisdiction require sensitive judgments about congressional intent, judicial power, and the federal system." 478 U. S., at 810. The opinion included a lengthy footnote explaining that questions of jurisdiction over state-law claims require "careful judgments," id., at 814, about the "nature of the federal interest at stake," id., at 814, n. 12 (emphasis deleted). And as a final indication that it did not mean to make a federal right of action mandatory, it expressly approved the exercise of jurisdiction sustained in Smith, despite the want of any federal cause of action available to Smith's shareholder plaintiff. 478 U. S., at 814, n. 12. [318] Merrell Dow then, did not toss out, but specifically retained, the contextual enquiry that had been Smith's hallmark for over 60 years. At the end of Merrell Dow, Justice Holmes was still dissenting.

Accordingly, Merrell Dow should be read in its entirety as treating the absence of a federal private right of action as evidence relevant to, but not dispositive of, the "sensitive judgments about congressional intent" that § 1331 requires. The absence of any federal cause of action affected Merrell Dow's result two ways. The Court saw the fact as worth some consideration in the assessment of substantiality. But its primary importance emerged when the Court treated the combination of no federal cause of action and no preemption of state remedies for misbranding as an important clue to Congress's conception of the scope of jurisdiction to be exercised under § 1331. The Court saw the missing cause of action not as a missing federal door key, always required, but as a missing welcome mat, required in the circumstances, when exercising federal jurisdiction over a state misbranding action would have attracted a horde of original filings and removal cases raising other state claims with embedded federal issues. For if the federal labeling standard without a federal cause of action could get a state claim into federal court, so could any other federal standard without a federal cause of action. And that would have meant a tremendous number of cases.

One only needed to consider the treatment of federal violations generally in garden variety state tort law. "The violation of federal statutes and regulations is commonly given negligence per se effect in state tort proceedings."[7] Restatement [319] (Third) of Torts § 14, Reporters' Note, Comment a, p. 195 (Tent. Draft No. 1, Mar. 28, 2001) (hereinafter Restatement). See also W. Keeton, D. Dobbs, R. Keeton, & D. Owen, Prosser and Keeton on Law of Torts § 36, p. 221, n. 9 (5th ed. 1984) ("[T]he breach of a federal statute may support a negligence per se claim as a matter of state law" (collecting authority)). A general rule of exercising federal jurisdiction over state claims resting on federal mislabeling and other statutory violations would thus have heralded a potentially enormous shift of traditionally state cases into federal courts. Expressing concern over the "increased volume of federal litigation," and noting the importance of adhering to "legislative intent," Merrell Dow thought it improbable that the Congress, having made no provision for a federal cause of action, would have meant to welcome any state-law tort case implicating federal law "solely because the violation of the federal statute is said to [create] a rebuttable presumption [of negligence] . . . under state law." 478 U. S., at 811-812 (internal quotation marks omitted). In this situation, no welcome mat meant keep out. Merrell Dow's analysis thus fits within the framework of examining the importance of having a federal forum for the issue, and the consistency of such a forum with Congress's intended division of labor between state and federal courts.

As already indicated, however, a comparable analysis yields a different jurisdictional conclusion in this case. Although Congress also indicated ambivalence in this case by providing no private right of action to Grable, it is the rare state quiet title action that involves contested issues of federal law, see n. 3, supra. Consequently, jurisdiction over actions like Grable's would not materially affect, or threaten to affect, the normal currents of litigation. Given the absence of threatening structural consequences and the clear interest the Government, its buyers, and its delinquents have in the availability of a federal forum, there is no good reason to [320] shirk from federal jurisdiction over the dispositive and contested federal issue at the heart of the state-law title claim.[8]

IV

The judgment of the Court of Appeals, upholding federal jurisdiction over Grable's quiet title action, is affirmed.

It is so ordered.

JUSTICE THOMAS, concurring.

The Court faithfully applies our precedents interpreting 28 U. S. C. § 1331 to authorize federal-court jurisdiction over some cases in which state law creates the cause of action but requires determination of an issue of federal law, e. g., Smith v. Kansas City Title & Trust Co., 255 U. S. 180 (1921); Merrell Dow Pharmaceuticals Inc. v. Thompson, 478 U. S. 804 (1986). In this case, no one has asked us to overrule those precedents and adopt the rule Justice Holmes set forth in American Well Works Co. v. Layne & Bowler Co., 241 U. S. 257 (1916), limiting § 1331 jurisdiction to cases in which federal law creates the cause of action pleaded on the face of the plaintiff's complaint. Id., at 260. In an appropriate case, and perhaps with the benefit of better evidence as to the original meaning of § 1331's text, I would be willing to consider that course.[9]

[321] Jurisdictional rules should be clear. Whatever the virtues of the Smith standard, it is anything but clear. Ante, at 313 (the standard "calls for a `common-sense accommodation of judgment to [the] kaleidoscopic situations' that present a federal issue, in `a selective process which picks the substantial causes out of the web and lays the other ones aside'" (quoting Gully v. First Nat. Bank in Meridian, 299 U. S. 109, 117-118 (1936))); ante, at 314 ("[T]he question is, does a state-law claim necessarily raise a stated federal issue, actually disputed and substantial, which a federal forum may entertain without disturbing any congressionally approved balance of federal and state judicial responsibilities"); ante, at 317, 318 ("`[D]eterminations about federal jurisdiction require sensitive judgments about congressional intent, judicial power, and the federal system'"; "the absence of a federal private right of action [is] evidence relevant to, but not dispositive of, the `sensitive judgments about congressional intent' that § 1331 requires" (quoting Merrell Dow, supra, at 810)).

Whatever the vices of the American Well Works rule, it is clear. Moreover, it accounts for the "`vast majority'" of cases that come within § 1331 under our current case law, Merrell Dow, supra, at 808 (quoting Franchise Tax Bd. of Cal. v. Construction Laborers Vacation Trust for Southern Cal., 463 U. S. 1, 9 (1983)) — further indication that trying to sort out which cases fall within the smaller Smith category may not be worth the effort it entails. See R. Fallon, D. Meltzer, & D. Shapiro, Hart and Wechsler's The Federal [322] Courts and the Federal System 885-886 (5th ed. 2003). Accordingly, I would be willing in appropriate circumstances to reconsider our interpretation of § 1331.

[1] Mr. Zagrans filed a brief for Jerome R. Mikulski et ux. as amici curiae urging reversal.

[2] Accordingly, we have no occasion to pass upon the proper interpretation of the federal tax provision at issue here.

[3] Compare Seinfeld v. Austen, 39 F. 3d 761, 764 (CA7 1994) (finding that federal-question jurisdiction over a state-law claim requires a parallel federal private right of action), with Ormet Corp. v. Ohio Power Co., 98 F. 3d 799, 806 (CA4 1996) (finding that a federal private action is not required).

[4] The quiet title cases also show the limiting effect of the requirement that the federal issue in a state-law claim must actually be in dispute to justify federal-question jurisdiction. In Shulthis v. McDougal, 225 U. S. 561 (1912), this Court found that there was no federal-question jurisdiction to hear a plaintiff's quiet title claim in part because the federal statutes on which title depended were not subject to "any controversy respecting their validity, construction, or effect." Id., at 570. As the Court put it, the requirement of an actual dispute about federal law was "especially" important in "suit[s] involving rights to land acquired under a law of the United States," because otherwise "every suit to establish title to land in the central and western states would so arise [under federal law], as all titles in those States are traceable back to those laws." Id., at 569-570.

[5] Federal law does provide a quiet title cause of action against the Federal Government. 28 U. S. C. § 2410. That right of action is not relevant here, however, because the Federal Government no longer has any interest in the property, having transferred its interest to Darue through the quitclaim deed.

[6] For an extremely rare exception to the sufficiency of a federal right of action, see Shoshone Mining Co. v. Rutter, 177 U. S. 505, 507 (1900).

[7] Other jurisdictions treat a violation of a federal statute as evidence of negligence or, like Ohio itself in Merrell Dow Pharmaceuticals Inc. v. Thompson, 478 U. S. 804 (1986), as creating a rebuttable presumption of negligence. Restatement § 14, Reporters' Note, Comment c, at 196. Either approach could still implicate issues of federal law.

[8] At oral argument Grable's counsel espoused the position that after Merrell Dow, federal-question jurisdiction over state-law claims absent a federal right of action could be recognized only where a constitutional issue was at stake. There is, however, no reason in text or otherwise to draw such a rough line. As Merrell Dow itself suggested, constitutional questions may be the more likely ones to reach the level of substantiality that can justify federal jurisdiction. 478 U. S., at 814, n. 12. But a flat ban on statutory questions would mechanically exclude significant questions of federal law like the one this case presents.

[9] This Court has long construed the scope of the statutory grant of federal-question jurisdiction more narrowly than the scope of the constitutional grant of such jurisdiction. See Merrell Dow Pharmaceuticals Inc. v. Thompson, 478 U. S. 804, 807-808 (1986). I assume for present purposes that this distinction is proper — that is, that the language of 28 U. S. C. § 1331, "[t]he district courts shall have original jurisdiction of all civil actions arising under the Constitution, laws, or treaties of the United States" (emphasis added), is narrower than the language of Art. III, § 2, cl. 1, of the Constitution, "[t]he judicial Power shall extend to all Cases, in Law and Equity, arising under this Constitution, the Laws of the United States, and Treaties made, or which shall be made, under their Authority . . ." (emphases added).

7.4.6 Note on Embedded Federal Questions 7.4.6 Note on Embedded Federal Questions

     1. The Grable TestGrable represents an effort to balance between federalizing what have been state issues and leaving it to the states to provide guidance on federal issues of national importance.

Grable sets forth a three part test:

Instead, the question is, does a state-law claim necessarily raise a stated federal issue, actually disputed and substantial, which a federal forum may entertain without disturbing any congressionally approved balance of federal and state judicial responsibilities.

     In the subsequent case of Gunn v. Minton, 568 U.S. 251 (2013), the Court tweaked the Grable test so that it clearly became a four part test:

That is, federal jurisdiction over a state law claim will lie if a federal issue is: (1) necessarily raised, (2) actually disputed, (3) substantial, and (4) capable of resolution in federal court without disrupting the federal-state balance approved by Congress.

Where all four of these requirements are met, we held, jurisdiction is proper because there is a “serious federal interest in claiming the advantages thought to be inherent in a federal forum,” which can be vindicated without disrupting Congress's intended division of labor between state and federal courts.

     The first two legs of the test are pretty straightforward.  Litigants will not be able to achieve federal jurisdiction if federal issues are simply stapled on the body of what remains a state law case, nor will they get into federal court if the federal issues are so readily resolved that there is no actual dispute over them. The 'substantial' leg is a bit more difficult but made clear by Gunn. In this element of the test, the court is looking for issues that matter not just to the litigants but to the system of justice as a whole. In Gunn, the plaintiff claimed that his former attorneys had committed malpractice by failing to make arguments in a case involving a patent he had filed but that was found invalid. The interpretation of patent law as it applied to the plaintiff's case was surely substantial with regard to the outcome of that case, but it was not substantial with regard to the system of patent law as a whole (as distinguished from the generally applicable IRS notice procedures at issue in Grable). Finding the federal issues to be actually disputed but not substantial, the Court held there was no federal question jurisdiction,  Finally, the last leg of the test deals with the issue that controlled in Merrell Dow. If Congress has chosen to not create a private right of action, or if an area of litigation has traditionally been resolved under state law, the fourth leg of the test will counsel against opening federal courts.

7.4.7 Federal Question Jurisdiction Problems 7.4.7 Federal Question Jurisdiction Problems

Think back to our case involving a pig. The pig disappears from the first farmer's pen. He files a lawsuit against the second farmer, alleging a common claim of unlawful taking of property. The second farmer asserts as a defense the federal Protection of Pigs Act, which allows pigs to be removed from unsafe conditions and supersedes any contrary state laws. Does federal question jurisdiction exist?

Still on the pig, but this time the pig is still in his original pigpen. The second farmer files a lawsuit seeking an injunction to have the pig removed from the pigpen, using as a basis the federal Protection of Pigs Act. Does federal question jurisdiction exist?

Still on the pig, and the pig has been removed from the original pigpen and taken to the pigpen of the second farmer. The second farmer left behind a note at the scene, explaining that the pig was removed in connection with the federal Protection of Pigs Act. The first farmer files a lawsuit to get return of the pig, invoking a state law that creates a right of action for anyone injured by a mistaken or fraudulent invocation of the federal Protection of Pigs Act. Success on this claim will necessarily involve interpretation of the Federal Protection of Pigs Act. Please explain how a court would analyze this.

7.5 Supplemental Jurisdiction 7.5 Supplemental Jurisdiction

7.5.1 Historical Development of Supplemental Jurisdiction 7.5.1 Historical Development of Supplemental Jurisdiction

     If you look back at Article III, Section 2 you will note that it talks about a federal court having jurisdiction over a 'case' or over a 'controversy.' How exactly does a 'case or controversy' relate to a legal claim? More specifically, how does a court deal with a situation where federal subject matter jurisdiction exists for one legal claim but not for a different but closely related claim arising from the same set of facts?

     This is the issue that the Supreme Court faced in United Mine Workers v. Gibbs, 383 U.S. 715 (1966). In Gibbs,  a labor dispute arose between a union and the operators of a coal mine in the mountains of Tennessee. Gibbs, an employee of and contractor to the coal mining company, brought claims against the union. The claims included a federal law claim alleging violations of labor law, for which federal question jurisdiction existed, and state law claims alleging violation of Tennessee law, which did not have their own basis for federal jurisdiction.

     The question (well, at least the question we care about) before the Court was whether it could resolve the state law claims along with the federal law claim. They all arose from the same facts, and the proof would substantially overlap. Creating a doctrine that came to be known as pendent claim jurisdiction, the Court held that since the federal claim created a case or controversy that was properly before the trial court, it could hear the state law claims as well. The claims all had to arise from a "common nucleus of operative fact" and the relationship between the federal claims and the pendent claims had to be such that ordinarily a plaintiff would expect to try them all at the same time.

     The court took care to note that the doctrine was discretionary - that is, the trial could hear the state law claims but was not obligated to. Taking into account issues such as "judicial economy, convenience and fairness to litigants" the court could decide to exercise pendent jurisdiction or not.

     There was not, at this time, any federal statute creating pendent claim jurisdiction although there was, of course, a statute creating jurisdiction over the federal question claim. The extension of pendent jurisdiction to additional claims was an exercise of the Court's rulemaking power, but drew on the trial court's already being empowered to hear the core of the "case or controversy" with the rulemaking dealing with whether the whole case or controversy could be brought into federal court.

     Note that in Gibbs the parties were the same on all claims. No additional parties were added to the case. 

     The issue of pendent parties came up in later cases. In Aldinger v. Howard, 427 U.S. 1 (1976), the Court suggested in dicta that pendent party jurisdiction might be proper unless Congress passed a contrary statute.

     The next case to address pendent parties involved a case where the original basis for federal jurisdiction was based on diversity. The plaintiff then wanted to add a claim against a third party who had been added as a third party defendant by the original defendant. At the time the plaintiff added this defendant it appeared that this new defendant was from the same state as the original defendant; it later came to light that the added party was from the same state as the plaintiff. Because of the requirement of complete diversity, the case could not have been brought originally with this defendant as part of the original lineup. The plaintiff argued that since it was originally in court with proper subject matter jurisdiction, it could add claims against pendent parties for which there was no original subject matter jurisdiction. Owen Equipment & Erection Co. v. Kroger, 437 U.S. 365 (1978). The Court rejected the extension of pendent party jurisdiction in this setting, holding that it would conflict with 28 U.S.C. § 1332.

     In Finley v. United States, 490 U.S. 545 (1989), the court faced the issue of adding a pendent party when the original claim was a federal question claim. Following a plane crash, the widow of one of the decedents brought a claim against the United States government under the Federal Tort Claims Act, a federal statute providing federal jurisdiction but only against the federal government. She attempted to add to the same lawsuit a state law tort claim against a city government also involved in operating the same airfield. The Court, in an opinion by Justice Scalia, held that it was up to Congress, not the Court, to authorize such pendent party jurisdiction when it was not included in the FTCA. (Note how the prior dicta in Aldinger v. Howard, 427 U.S. 1 (1976), however clear or on point, was not controlling on a later court).

     Congress responded with 28 U.S.C. § 1367. Please be clear on one thing: setting aside the Constitutional concerns around what makes up a 'case or controversy,' and setting aside issues about whether courts have discretion beyond what is supplied in the statute, § 1367, and not the cases that led up to it, provides the governing law now on pendent jurisdiction, or as it has since been called, supplemental jurisdiction. To keep things somewhat simple (and they get complicated enough), we will focus on the statute and its requirements.

7.5.2 28 U.S. Code § 1367 - Supplemental jurisdiction 7.5.2 28 U.S. Code § 1367 - Supplemental jurisdiction

(a) Except as provided in subsections (b) and (c) or as expressly provided otherwise by Federal statute, in any civil action of which the district courts have original jurisdiction, the district courts shall have supplemental jurisdiction over all other claims that are so related to claims in the action within such original jurisdiction that they form part of the same case or controversy under Article III of the United States Constitution. Such supplemental jurisdiction shall include claims that involve the joinder or intervention of additional parties.

(b) In any civil action of which the district courts have original jurisdiction founded solely on section 1332 of this title, the district courts shall not have supplemental jurisdiction under subsection (a) over claims by plaintiffs against persons made parties under Rule 14, 19, 20, or 24 of the Federal Rules of Civil Procedure, or over claims by persons proposed to be joined as plaintiffs under Rule 19 of such rules, or seeking to intervene as plaintiffs under Rule 24 of such rules, when exercising supplemental jurisdiction over such claims would be inconsistent with the jurisdictional requirements of section 1332.

(c) The district courts may decline to exercise supplemental jurisdiction over a claim under subsection (a) if—

      (1) the claim raises a novel or complex issue of State law,

      (2) the claim substantially predominates over the claim or claims over which the district court has original jurisdiction,

      (3) the district court has dismissed all claims over which it has original jurisdiction, or

      (4) in exceptional circumstances, there are other compelling reasons for declining jurisdiction.

(d) The period of limitations for any claim asserted under subsection (a), and for any other claim in the same action that is voluntarily dismissed at the same time as or after the dismissal of the claim under subsection (a), shall be tolled while the claim is pending and for a period of 30 days after it is dismissed unless State law provides for a longer tolling period.

(e) As used in this section, the term “State” includes the District of Columbia, the Commonwealth of Puerto Rico, and any territory or possession of the United States.

(Added Pub. L. 101–650, title III, § 310(a), Dec. 1, 1990, 104 Stat. 5113.)

7.5.3 § 1367 Explained 7.5.3 § 1367 Explained

     The first thing to note is that § 1367 overrules the result in Finley by allowing pendent parties to be added, but otherwise carries forward the law as developed by the Court before. The claim to be added must be closely related to the underlying federal claim - while the statute adopts the Constitutional standard of 'case or controversy' as a practical matter the inquiry into relatedness is pretty much the same. It must arise from the same cluster of operative facts. The doctrine remains discretionary - courts are not required to exercise supplemental jurisdiction. Parties who have one claim that has a basis for jurisdiction can add other claims against the same party. 

     Pendent parties may now be added. If the original claim is based only on diversity, and adding a new party would defeat complete diversity, § 1367(b) will prevent a plaintiff from adding that party, consistent with the result in Kroger. On the other hand, parties may be added to a diversity case if their addition does not destroy complete diversity or otherwise frustrate § 1332. Non-diverse parties may also be added outside the rather specific limitations of § 1367(b). If the original claim is a federal question claim, § 1367(b) will not apply, and the claim may be freely added subject to the discretionary takeback of § 1367(c).

     In terms of practical analysis, the place to start is with the claim that provides that original basis for federal subject matter jurisdiction. In order to sear this into your memory, I want you to take a moment to listen to one of the worst pieces of American pop music ever recorded, He Aint Heavy by The Hollies.  The song recounts what is said to be a true story. One cold, snowy night two young boys appeared at the door of an orphanage called Boys Town, with the older brother carrying the exhausted younger brother on his back. When asked if his brother was not too heavy for him to carry, the older brother is said to have replied, "He ain't heavy - he's my brother."

     Think of the original claim as the big brother. The big brother has to be a claim that can walk - that is, invoke federal subject matter jurisdiction - on its own. Otherwise, it cannot carry anything.

     The second claim has to be related. The older brother is not carrying some random child; he's carrying his closest relation, his brother. No claim comes in that is not related to the first claim.

     Finally, entry into the orphanage is discretionary. The guardian at the door has the power to exclude the younger brother. In most cases, though, the little waif riding on big brother's back will be allowed in.

     It doesn't fit into our family analogy, but you must also keep in mind the Kroger issue - if the supplemental claim would have destroyed complete diversity if brought by the claimant at the outset, it cannot be added under supplemental jurisdiction.

     An important thing to remember about supplemental jurisdiction is that it applies to all claims in the case, not just claims by the plaintiff. For example, imagine a lawsuit between Perry Plaintiff (NY) and Don Defendant (NJ). The original claim is for $100,000, and so full diversity is satisfied. Now, imagine that Don Defendant has a counterclaim arising from the same transaction as the original claim, but it is only for $50,000. The sum would be too low for diversity jurisdiction, and it cannot be aggregated to Plaintiff's claim. However, supplemental jurisdiction would allow it to be included. 

     Pay attention to Section 1367(b)'s takeback. Note that it only applies to claims by plaintiffs, and only to claims where the sole basis for federal subject matter jurisdiction is diversity. For our hypothetical above, further imagine that Defendant's counterclaim is not just against Plaintiff, but also against Tammy Third (NJ). Here, in addition to the amount in controversy problem there also is no diversity. No problem - the claim is not by the original plaintiff, and since it is sufficiently related to the original claim there is a basis for supplemental jurisdiction.

7.5.4 Itar-Tass Russian News Agency v. Russian Kurier, Inc. 7.5.4 Itar-Tass Russian News Agency v. Russian Kurier, Inc.

ITAR-TASS RUSSIAN NEWS AGENCY et al., Plaintiff, v. RUSSIAN KURIER, INC. et al., Defendant. Al J. DANIEL, Jr. and Michael Newcity, Appellants, v. ITAR-TASS RUSSIAN NEWS AGENCY, et al, and Julian H. Lowenfeld and Moskovsky Komsomolets and AR Publishing Co. Inc, Appellees.

Docket No. 97-7444.

United States Court of Appeals, Second Circuit.

Argued Oct. 22, 1997.

Decided April 3, 1998.

*444Al J. Daniel, Jr., New York City, pro se.

Michael Neweity, Durham, NC, pro se.

Julian H. Lowenfeld, New York City, pro se and for Plaintiffs-Appellees.

William J.T. Brown, New York City (Donovan Leisure Newton & Irvine), for PlaintiffsAppellees Moskovsky Komsomolets and AR Publishing Co., Inc.

Before: WINTER, Chief Judge, MESKILL, Circuit Judge, and POLLACK, District Judge.*

POLLACK, Senior District Judge:

PRELIMINARY

This Circuit’s opinion is that the district court has a “responsibility to protect its own officers in such matters as fee disputes.” Cluett, Peabody & Co. v. CPC Acquisition Co., 863 F.2d 251, 256 (2d Cir.1988).

BACKGROUND

During the final phases of a non-jury copyright litigation, after the case was tried but while it was sub judice, appellant Al J. Daniel (“Daniel”), an attorney for plaintiffs and counsel to the principal attorney for plaintiffs, Julian H. Lowenfeld (“Lowenfeld”), moved for leave to withdraw from further representation of the plaintiffs and Lowenfeld in the suit. Appellant, Michael Neweity (“Neweity”), are expert witness for plaintiffs, moved for his fees and expenses. Specifically, appellants claimed non-payment of alleged contractual compensation due them from the plaintiffs and Lowenfeld and invoked the supplemental jurisdiction of the district court pursuant to 28 U.S.C. § 1367 to fix their fees and expenses pursuant to retainer agreements, and to fix the amount of Daniel’s charging lien under New York Judiciary Law § 475.

The motions to withdraw as counsel and for fees, respectively, were unopposed and were granted. However, the district judge declined to exercise his supplemental jurisdiction over the motions for fees by Neweity and Daniel and dismissed them without prejudice. The district judge added that the plaintiffs and their attorneys are in no way prejudiced thereby in their claims for statutory fees under the copyright law, by his failure to resolve their internecine (sic) fee disputes. On the same day, the district judge announced that the plaintiffs were entitled to judgment against the defendants on the merits for deliberate copyright infringement and directed the submission of an appropriate judgment. The district judge further directed the plaintiffs to submit a separate application for “modest” statutory attorneys’ fees.

Accordingly, the Judgment subsequently submitted and signed provided that “the plaintiffs may submit any appropriate application for the costs of the litigation and reasonable attorneys’ fees, as allowed by law, to be determined on the basis of applications submitted separately in accordance with Fed*445eral Rule of Civil Procedure 54(d).” All attorneys complied.

Appellees, and plaintiffs’ other counsel entitled to a statutory award under 17 U.S.C. § 505, filed the information required by the district court. Appellees sought a judgment requiring defendants to pay 100% of the time and expenses for which Newcity had billed plaintiffs, and for 80% of the time and expenses billed by Daniel. The statutory claim for Daniel’s fees and expenses against defendants was based upon the identical statements upon which his contractual claims against plaintiffs and Lowenfeld (except as to quantum) were based. Daniel’s statutory fee claim was voluntarily reduced by 20% in light of Hensley v. Eckerhart 461 U.S. 424, 430 and n. 3, 103 S.Ct. 1933, 1937-38 and n. 3, 76 L.Ed.2d 40 (1983), and other factors.

On June 10, 1997, the district court issued an Opinion and Order awarding plaintiffs a total of $360,398.52 against defendants for statutory attorneys’ fees and costs. Included in said total was a statutory award for Lowenfeld of $243,948.86 which was for fees and costs including Newcity’s total unpaid claim for services as an expert witness; $65,656.58 for Daniel, consisting of all of his expenses and 50% of his statutory fee claim; $28,-802.58 of legal fees to another attorney, one Trope; and $21,990.50 to yet another attorney, one Berman. Judgment was- entered accordingly on July 3,1997.

The defendants have appealed the Judgment of liability for copyright infringement against them. That appeal is pending. The defendants did not appeal the judgment for attorneys’ fees and costs.

Appellants Daniel and Newcity have appealed herein from the dismissal of the motion invoking supplemental jurisdiction to enforce their alleged contractual fee arrangements with the Appellees, subject of course to any offset by way of a collection of the statutory award which depends on the outcome of the Judgment against the defendants.

DISCUSSION

The district court had obtained total familiarity with the subject matter of the suit and the professional services of the moving parties thereon and of the virtual totality of all the compensation arrangements contended for and disputed, all of which were fully disclosed on the record of the proceedings before the court. All of these would have to be considered anew and relitigated in possibly more than one state court unfamiliar with the proceedings and the trial services if supplemental jurisdiction is not exercised.

A. Supplemental Jurisdiction

The district court declined to exercise supplemental jurisdiction over appellants’ motion for attorneys’ fees because, “the disputes over fees are entirely separate from the underlying action,” the “adjudication of [appellants’] claims would not serve the interests of judicial economy and would require considerable judicial resources,” and rendering a decision over the supplemental claim “might cast aspersions ... and compromise some of [the parties] arguments for statutory attorney’s fees.”

In rendering its decision, the district court made no reference to 28 U.S.C. § 1367, which provides federal judges with both the power to exercise supplemental jurisdiction and the discretion, in specified circumstances, to decline to entertain such claims. The district court’s decision to decline supplemental jurisdiction was based loosely on the discretionary factors set forth in United Mine Workers of America v. Gibbs, 383 U.S. 715, 726, 86 S.Ct. 1130, 1139, 16 L.Ed.2d 218 (1966), which emphasized that “pendent jurisdiction is a doctrine of discretion, not of plaintiffs right.” Gibbs directed federal courts, in determining whether to exercise jurisdiction over a case involving state-law claims, to “consider and weigh in each case, at every stage of the litigation, the values of judicial economy, convenience, fairness, and comity.” Camegie-Mellon Univ. v. Cohill, 484 U.S. 343, 350, 108 S.Ct. 614, 619, 98 L.Ed.2d 720 (1988). See generally Executive Software N. Am., Inc. v. United States Dist. Court, 24 F.3d 1545, 1552-55 (9th Cir.1994) (discussing the “flexibility” of the Gibbs doctrine and the criticism resulting from that flexibility).

*446The primary issue for this court is whether section 1367 altered the Gibbs analysis. Section 1367 reads, in pertinent part, as follows:

(a) Except as provided in subsection[ ] ... (c) ..., in any civil action of which the district courts have original jurisdiction, the district courts shall have supplemental jurisdiction over all other claims that are so related to claims in the action within such original jurisdiction that they form part of the same case or controversy under Article III of the United States Constitution ____
(c) The district courts may decline to exercise supplemental jurisdiction over a claim in subsection (a) if—
(1) the claim raises a novel or complex issue of State law,
(2) the claim substantially predominates over the claim or claims over which the district court has original jurisdiction,
(3) the district court has dismissed all claims over which it has original jurisdiction, or
(4) in exceptional circumstances, there are other compelling reasons for declining jurisdiction.

Facially, the statute appears to have transformed the Gibbs analysis in important respects. See Executive Software, 24 F.3d at 1555-57 (discussing differences and similarities between Gibbs and section 1367). Nowhere within subsection 1367(e), for example, are the underlying criteria of judicial economy, convenience, fairness, and comity mentioned. We have previously stated that “the discretion implicit in the word ‘may’ in subdivision (c) of § 1367 permits the district court to weigh and balance several factors, including considerations of judicial economy, convenience, and fairness to litigants.” Purgess v. Sharrock, 33 F.3d 134, 138 (2nd Cir.1994) (citations omitted). In other words, this Circuit has recognized that the Gibbs factors are merged into the subsection 1367(c) analysis, but it has not stated exactly how they are merged.

Other circuits have answered this question to varying degrees and with varying results. The Seventh Circuit has taken the approach that subsection 1367(e) merely incorporates the discretionary factors set forth in Gibbs and its progeny. See Brazinski v. Amoco Petroleum Additives Co., 6 F.3d 1176, 1182 (7th Cir.1993). The First, Third, and D.C. Circuits have taken a similar approach. See Rodriguez v. Doral Mortgage Corp., 57 F.3d 1168, 1177 (1st Cir.1995) (“the district court, in reaching its discretionary determination on the jurisdictional question, will have to assess the totality of the attendant circumstances”); Borough of West Mifflin v. Lancaster, 45 F.3d 780, 788 (3d Cir.1995) (“Section 1367(e) ... was intended simply to codify the preexisting pendent jurisdiction law, enunciated in Gibbs and its progeny____”); Diven v. Amalgamated Transit Union & Local 689, 38 F.3d 598, 601 (D.C.Cir.1994) (“Despite Congress’ use of ‘shall’ [in subsection 1367(a) ], the statute fairly exudes deference to judicial discretion—at least once the threshold determinations have been met and the court moves on to consider the exceptions.”).

The Ninth Circuit has adopted a different approach. The Ninth Circuit has held that by listing the four circumstances in subsection 1367(c), Congress intended to cabin the previous common law analysis. See Executive Software N. Am., Inc. v. United States Dist. Court, 24 F.3d 1545, 1556-57 (9th Cir.1994). Once a court identifies one of the factual predicates which corresponds to one of the subsection 1367(c) categories, the exercise of discretion “is informed by whether remanding the pendent state claims comports with the underlying objective of ‘most sensibly accommodat[ing]’ the values of ‘economy, convenience, fairness, and comity.’ ” Id. at 1557 (citation omitted). In addition, the Court found that the catch-all created by subsection 1367(c)(4) should be employed when circumstances are “quite unusual.” 24 F.3d at 1558. This would require a district court to “articulate why the circumstances of the case are exceptional in addition to inquiring whether the balance of the Gibbs values provide compelling reasons for declining jurisdiction in such circumstances.” Id.

*447The Ninth Circuit’s model has been adopted by the Eighth and Eleventh Circuits. See McLaurin v. Prater, 30 F.3d 982, 985 (8th Cir.1994) (“[Subsection 1367(c) ] plainly allows the district court to reject jurisdiction over supplemental claims only in the four instances described therein.”); See Palmer v. Hospital Auth., 22 F.3d 1559, 1569 (11th Cir.1994) (“The breadth of discretion afforded federal courts in these cases has been codified by section 1367(c)---- [Wjhile supplemental jurisdiction must be exercised in the absence of any of the four factors of subsection 1367(c), when one or more of these factors is present, the additional Gibbs considerations may, by their presence or absence, influence the court in its decision concerning the exercise of such discretion.”).

We now adopt the interpretation of Section 1367 set forth in Executive Software, and hold that section 1367 has indeed altered Gibbs ’ discretionary analysis. Initially, we note that the Ninth Circuit’s approach is consistent with the terms and structure of section 1367. On the one hand, subsection 1367(a) uses the term “shall” which implies that once it is determined that a supplemental claim is related to the claim within the court’s original jurisdiction such that they form the same case or controversy, supplemental jurisdiction over the related claim is mandatory. On the other hand, the use of “may” in subsection 1367(c) seems to confer on federal courts at least some discretion to not hear claims over which there is supplemental jurisdiction in the enumerated circumstances. If subsection 1367(c) were merely to incorporate Gibbs, subsection 1367(c) would be rendered superfluous. See United States v. Menasche 348 U.S. 528, 538-39, 75 S.Ct. 513, 519-20, 99 L.Ed. 615 (1955) (“[it is] our duty to give effect, if possible, to every clause and word of a statute”); Ohio Power Co. v. Federal Energy Regulatory Comm’n, 880 F.2d 1400, 1406 (D.C.Cir.1989), rev’d on other grounds, 498 U.S. 73, 111 S.Ct. 415, 112 L.Ed.2d 374 (1990).

This interpretation not only comports with the text and structure of the statute, but is also consistent with the legislative history. On July 26, 1990, Representative Kastenmeier and Representative Moorhead introduced House Bill 5381, which contained a supplemental jurisdiction proposal. The proposal sought to circumscribe judicial discretion to dismiss supplemental claims which they have the power to entertain by codifying the three examples mentioned in Gibbs. 1 See Federal Courts Study Committee Implementation Act and Civil Justice Reform Act: Hearings on H.R. 5381 and H.R. 3898 Before the Sub-comm. on Courts, Intellectual Property, and the Administration of Justice of the House Comm, on the Judiciary, 101st Cong., 2d Sess. (1990) at 30 [hereinafter “Hearings”].

At the hearings on the bill, Judge Joseph Weis, Jr., former chair of the Federal Courts Study Committee, proposed a substitute which contained four grounds for discretionary dismissal: when the supplemental claim “raises a complex or novel issue of State law;” when the non-federal claim “predominates” over the federal claim; when the judge has dismissed the federal claim; and when “there are other appropriate reasons, such as judicial economy, convenience, and fairness to litigants, for declining jurisdiction.” Hearings at 98. By using language derived directly from Gibbs, it appears that Judge Weis was, in effect, attempting to codify the Gibbs discretionary prong. A proposal introduced on September 11, 1990 by Professors Rowe, Burbank, and Mengler also *448contained these four bases for dismissal. Hearings at 722.

On September 13, 1990 the subcommittee met and favorably reported House Bill 5381, which adopted Judge Weis’ proposal as to subsection 1367(e), but contained two changes. First, they reinserted “substantially” back into the phrase “substantially predominates.” Next, they narrowed the catchall exception by restricting it to “exceptional circumstances [when] there are other compelling reasons for declining jurisdiction.” H.R.Rep. No. 734, 101st. Cong., 2d Sess. at 11. Thus, what remained is not simply a codification of Gibbs.

iN his Opinion and Order dated March 10, 1997, the district judge gave as reasons for declining supplemental jurisdiction concerns of judicial economy,2 convenience, fairness,3 and the possibility of the easting of “aspersions” against the parties. However, the discretion to decline supplemental jurisdiction is available only if founded upon an enumerated category of subsection 1367(e).

None of the supplemental claims raises “a novel or complex issue of State law----” 28 U.S.C. § 1367(c)(1). Appellants’ claims are straightforward and the relevant facts are easily determined. Appellees do not deny that there were agreements with both Daniel and Newcity, and that Daniel and Newcity performed legal services in the action. The only issue which remains is which of the Appellees are liable for the undisputed fees and expenses owed to the Appellants.

Subsection 1367(e)(2) is not implicated because Appellants’ supplemental claims do not “substantially predominate” over the copyright claims. In contrast to the extensive and complex copyright litigation involved in the action, Appellants merely seek payment for services rendered and fees incurred throughout the course of the litigation.

Subsection 1367(c)(3), which empowers a federal court to dismiss motions seeking supplemental jurisdiction in a case if it has “dismissed all claims over which it has original jurisdiction” has no bearing in this instance. The federal claims in this action were not dismissed; they were sustained five months after the motions involving supplemental jurisdiction were served.

Subsection 1367(c)(4) authorizes federal courts, upon the recognition of “exceptional circumstances,” to decline supplemental jurisdiction. The use of “exceptional circumstances” indicates that “Congress has sounded a note of caution that the bases for declining jurisdiction should be extended beyond the circumstances identified in subsections (c)(l)-(3) only if the circumstances are quite unusual.” Executive Software, 24 F.3d at 1558. In other words, declining jurisdiction outside the ambit of 1367(c)(1)-(3) appears as the exception rather than the rule. Thus, federal courts “must ensure that the reasons identified as ‘compelling’ are not deployed in circumstances that threaten this principle.” Id. None of the reasons cited by the district court qualify as adequate grounds for declining supplemental jurisdiction under 1367(e)(4).

B. Daniel’s Charging Lien Under New York Judiciary Law § &75

New York Judiciary Law § 475 (“Section 475”) governs attorneys’ charging liens in federal courts sitting in New York. See Markakis v. S.S. Mparmpa Christos, 267 F.2d 926, 927 (2d Cir.1959) (Section 475 “creates an equitable right and remedy cognizable in the federal courts”); Brooks v. Mandel-Witte Co., 54 F.2d 992, 994 (2d Cir.1932), cert. denied Mandel-Witte Co. v. Brooks, 286 U.S. 559, 52 S.Ct. 641, 76 L.Ed. 1292 (1932) (“federal courts sitting in a state have en*449forced statutes of that state creating attorney’s liens whether the suit for services in which the hen was claimed was originally brought in a state court or in a federal court”).

Moreover, the Second Circuit has “long recognized that the lien created by section 475 ... is enforceable in federal courts in accordance with its interpretation by New York courts.” In re Chesley v. Union Carbide Corp., 927 F.2d 60, 67 (2d Cir.1991); see also In re McCrory Stores Corp., 19 F.Supp. 691, 693 (S.D.N.Y.1937) (“The lien being a matter of substantive law created by a statute of New York, the bounds placed on it by authoritative decisions of the New York courts are bounds on it here.”)

1. History of Section Jf.75

Judicial remedies “for the protection of attorneys against the knavery of their clients” took root at common law. Goodrich v. McDonald, 112 N.Y. 157, 163, 19 N.E. 649 (1889). Lord Kenyon observed that it had been “settled long ago, that a party should not run away with the fruits of the cause without satisfying the legal demands of his attorney, by whose industry, and in many instances at whose expense, those fruits are obtained.” Read v. Dupper, 101 Eng. Rep. 595, 596 (1795). The theory underlying the attorney’s lien is “ in analogy to the lien which a mechanic has upon any article which he manufactures.” Williams v. Ingersoll, 89 N.Y. 508, 517 (1882).

In New York, the beginnings of the doctrine of an attorney’s charging lien are found in 1841. At that time, the New York Court for the Correction of Errors4 declared that there was no doubt “that by the law of this state, as it has always existed from the time of its first settlement, the lawyer, as well as the physician, was entitled to recover a compensation for his services; and that such services were never considered here as gratuitous or honorary merely.” Adams v. Stevens & Cogger, 26 Wend. 451, 455 (1841). In 1858, the Court of Appeals noted: “As in other cases of lien, the attorney is protected, because it is by his labor and skill that the judgment has been recovered. The judgment being under the control of the court, and the parties within its jurisdiction, it will see that no injustice is done to its own officers.” Rooney v. Second Ave. R. Co., 18 N.Y. 368, 369 (1858).

The common law doctrine of an attorney’s charging lien was codified in New York in section 66 of the Code of Civil Procedure in 1876. As amended in 1899, section 66 of the Code of Civil Procedure was endowed with the same statutory language that we apply today. Section 475, enacted in 1909, was lifted bodily from section 66 of the Code of Civil Procedure after that statute’s demise:

From the commencement of an action, special or other proceeding in any court or before any state, municipal or federal department, except a department of labor, or the service of an answer containing a counterclaim, the attorney who appears for a party has a lien upon his client’s cause of action, claim or counterclaim, which attaches to a verdict, report, determination, decision, judgment or final order in his client’s favor, and the proceeds thereof in whatever hands they may come; and the lien cannot be affected by any settlement between the parties before or after judgment, final order or determination. The court upon the petition of the client or attorney may determine and enforce the lien.

N.Y. Judiciary Law § 475 (McKinney’s 1997). Writing in 1903, the New York Court of Appeals observed that, with the 1899 amendment of section 66 of the Code of Civil Procedure, “[t]he law has made great progress in protecting members of the bar____” Fischer-Hansen v. Brooklyn Heights R. Co., 173 N.Y. 492, 495, 66 N.E. 395 (1903). There, the court stated:

[T]he statute created a lien in favor of the attorney on his client’s cause of action, in whatever form it may assume in the course of the litigation, and enables him to follow *450the proceeds into the hands of third parties, without regard to any settlement before or after judgment; that all the world must take notice of the lien; and that it was unnecessary for the attorney to give notice of his claim to the other party.

Id. at 498, 66 N.E. 395 (citing Peri v. N.Y.C. & H.R.R. Co., 152 N.Y. 521, 46 N.E. 849 (1897) (quotations omitted)). Moreover, the court held: “The statute is remedial in character, and hence should be construed liberally in aid of the object sought by the legislature, which was to furnish security to attorneys by giving them a lien upon the subject of the action.” Fischer-Hansen, 173 N.Y. at 499, 66 N.E. 395.

2. Attorney of Record

Despite the liberal interpretation afforded Section 475, the New York cases make clear that the charging lien provided for by Section 475 is for the benefit of an “attorney of record” only. Weinstein v. Seidmann, 173 A.D. 219, 220, 159 N.Y.S. 371 (1st Dept.1916). See also Cataldo v. Budget Rent A Car Corp., 226 A.D.2d 574, 574, 641 N.Y.S.2d 122 (2d Dept.1996) (“[t]he Court of Appeals has clearly stated that the emphasized language grants a lien to the ‘attorney of record’ ”) (citations omitted); In re Sebring, 238 A.D. 281, 285, 264 N.Y.S. 379 (4th Dept.1933) (Section 475 gives a lien to “the attorney who appears for a party. It is not broad enough to include counsel; it is confined to the attorney of record”); Gary v. Cohen, 34 Misc.2d 971, 973, 231 N.Y.S.2d 394 (N.Y.Sup.Ct.1962) (“[a]n attorney’s statutory or charging hen is confined to the attorney of record in the cause; trial counsel is not thus protected”).

Appellees contend that Daniel was not an attorney of record in the matter and is therefore not entitled to a charging lien under Section 475. They contend that Daniel served merely “of counsel” to Lowenfeld according to Daniel’s own characterization of himself sometimes as “of counsel” in various pleadings and at oral argument on this appeal. We find that Daniel was not, in fact, merely “of counsel,” in the sense advocated by Appellees, but that he was an “attorney of record” for the purposes of Section 475.

Rodriguez v. City of New York, 66 N.Y.2d 825, 498 N.Y.S.2d 351, 489 N.E.2d 238 (1985) offers the most intelligible statement regarding who may be called an “attorney of record” for the purposes of Section 475. There, an attorney seeking a charging hen was originally retained by the plaintiff to be her attorney in the action. Because of his advanced age and limited practice, the attorney retained a second attorney to act “of counsel” and the two agreed to split the fee evenly. However, the attorney hired to act “of counsel” in fact handled all of the pleadings in the ease as well as the entire trial.

The court held that the initially retained attorney was not entitled to a charging hen because “it is undisputed that [his] name never appeared on any of the pleadings, motion papers, affidavits, briefs or record in plaintiffs action.” Id. at 827, 498 N.Y.S.2d at 353, 489 N.E.2d at 240. The “of counsel” attorney was, in fact, the sole attorney of record in the case because he was the only one who ever appeared in the case. The fact that the initially retained attorney’s name appeared on a retainer statement in a negligence ease, filed pursuant to the rules of the Appellate Division, was insufficient to qualify him as an attorney of record. The absence of his name anywhere in the record showed that he was not, as he claimed, an attorney of record.

The restriction provided by Rodriguez on who may be an ‘attorney of record’ for the purposes of Section 475 has been implemented on several occasions by the New York courts. See, e.g., Cataldo, 226 AJD.2d at 574, 641 N.Y.S.2d 122 (“before an attorney can be granted a hen pursuant to [Section 475] he or she must have appeared for the client by participating in a legal proceeding on the client’s behalf or by having his [or her] name affixed to the pleadings, motions, records, briefs, or other papers submitted in the matter” (quotations omitted)); Ebert v. New York City Health & Hosp. Corp., 210 A.D.2d 292, 292, 619 N.Y.S.2d 756 (2d Dept.1994). (“[T]he mere existence of a retainer agreement is insufficient to create a charging hen pursuant to [Section 475]. That statute provides that an attorney ‘appear’ for the chent, in the sense of participating in a legal pro*451ceeding on the client’s behalf or by having his name affixed to the pleadings, motions, records, briefs, or other papers submitted in the matter.”); Lai Ling Cheng v. Modansky Leasing Co., 137 A.D.2d 781, 783, 525 N.Y.S.2d 328 (2d Dept.1988) (“an attorney whose name nowhere appears in the pleadings, motion papers, affidavits, briefs or record in a plaintiffs action is not entitled to seek a ... charging lien under [Section 475]”).

The court in Rodriguez was not troubled, as we are not, by the parties’ use of the term “of counsel” as a further description of their relationship. The record plainly demonstrated, as it does here, that the “of counsel” attorney was, in fact, an attorney of record. The record is replete with evidence that Daniel was an “attorney of record” in this case. Daniel signed and submitted numerous papers and letters to the court, and appeared at trial, as “attorney for the plaintiffs.” There is no indication anywhere in the record that Daniel was treated by the Clerk of the Court, the Magistrate Judge or the District Court Judge as anything other than an attorney of record for plaintiffs. The Joint Pretrial Order, signed on the very same page by Judge Koeltl, and Messrs. Lowenfeld and Daniel, respectively, is executed as follows:

“Dated: June 20,1996
“Julian H. Lowenfeld Julian H. Lowenfeld
[Address]
Attorney for Plaintiffs
(except Fromer & Associates, Inc.
Isaac Fromer, and Unlimited Brokerage, Inc.)
“John G. Koeltl
John G. Koeltl
United States District Judge”
“Al J. Daniel, Jr.
AI J. Daniel
[Address]
Of Counsel
Attorney for Plaintiffs
(except Fromer & Associates, Inc.
Isaac Fromer and Unlimited Brokerage, Inc.)”

The references to Daniel as “of counsel” made by Daniel and Lowenfeld in their submissions to the court offer little to the analysis. Nothing in the record suggests that Daniel acted simply as an advisor or as a law clerk to Lowenfeld, or that he merely assisted Lowenfeld in the case’s preparation. It is true that he appeared in the case, along with attorney Trope, some time after it had been initiated by Lowenfeld, but there is no doubt that he entered the case, allegedly with the plaintiffs’ consent, to serve as co-counsel with Lowenfeld and Trope.

Lowenfeld’s own statements are telling. In his Memorandum in Support of Motion for Award of Costs and Reasonable Attorney’s Fees seeking statutory compensation under 17 U.S.C. § 505, Lowenfeld states: “Daniel was retained on behalf of plaintiffs a month before trial in the case, when the discovery posture of the ease, the number of witnesses, exhibits, and issues involved, required that Daniel work full-time on this case, and prevented his pursuing other matters.” (emphasis added). This does not sound like a description of work by someone who is merely a legal advisor or assistant. Lowenfeld concedes in the same memorandum that he was “less experienced in litigation” than his co-counsel, Daniel and Trope. This statement begs the question of who might have in fact been assisting whom. Thus, despite what he may have called himself, Daniel was clearly not acting merely “of counsel” in this matter in the constricted sense put forth by appellees. Daniel was an attorney of record for the purposes of Section 475.

It should be noted that Daniel’s permitted withdrawal as attorney of record does not affect his entitlement to the statutory lien under Section 475. The New York Court of Appeals has stated that “[i]t has long been held that attorneys who terminate their representation for just cause continue to be entitled to enforce their liens.” Klein v. Eubank, 87 N.Y.2d 459, 462, 640 N.Y.S.2d 443, 663 N.E.2d 599 (1996). Moreover, “an attorney’s participation in the proceeding at one point as counsel of record is a sufficient predicate for invoking the statute’s protection.” Id. In this case, Daniel properly ter*452minated his relationship with his clients when he was not paid as agreed.

3. Mon than one Attorney of Record

Appellees also contend that there may be only one “attorney of record” in a case. They point to Kitsch v. Riker Oil Co., 23 A.D.2d 502, 503, 256 N.Y.S.2d 536 (2d Dept.1965) which found that there “is no authority for a party to be represented by more than one attorney in an action” and Stinnett v. Sears Roebuck & Co., 201 A.D.2d 362, 364, 607 N.Y.S.2d 646 (1st Dept.1994) which held that “[w]here a case ... does not involve special circumstances or highly complex litigation, a party may not be represented by more than one attorney of record.”

These cases are incongruous with statements made by the New York Court of Appeals in Rodriguez, which indicate that, in New York, there may indeed be more than one “attorney of record” for the purposes of Section 475. In Rodriguez, the attorney moving to have a charging lien declared in his favor asserted that the “attorney of record” should be estopped from denying that the movant attorney was also an attorney of record. He claimed that it was the “attorney of record” who had control of all the pleadings and papers in the ease and failed to place the movant attorney’s name on them. The court disagreed, stating: “There is evidence in the record ... that respondent sent movant copies of motions and pleadings for his review and that movant apparently approved and returned them without requesting that his name be added to them as an attorney of record.” Id. at 828, 498 N.Y.S.2d at 353, 489 N.E.2d at 240. The clear implication of this statement is that the court would not have had any objection to an additional attorney of record.

Furthermore, we have found no cases which have limited the charging lien to only one attorney in an action where both attorneys appeared together on behalf of plaintiffs in the case. Indeed, as far back as 1873, the New York Court of Appeals held that a lien could exist in favor of more than one attorney in a case. In Zogbaum v. Parker, 55 N.Y. 120 (1873), an attorney was hired by plaintiff as “attorney and counsel” and a second attorney, referred to as a “counselor of the court,” was also hired by plaintiff to assist the first attorney in the trial. It was held that the attorneys “had a valid lien upon the recovery, by virtue of their agreement with [plaintiff]” and because “[attorneys have a lien for their compensation for services upon claims which they prosecute and judgments they recover, whether arising out of tort or contract.” Id. at 122. Nowhere did the court consider that one of them might not be entitled to a charging lien by virtue of the other’s like claim.

4. Equitable Assignment

Although we find Daniel to have been an attorney of record, and thereby entitled to have had his charging lien determined by the district court under Section 475, it is worth noting that even if we were to conclude that Daniel was not an attorney of record, there are cases to suggest that he would nonetheless have been an equitable assignee of the cause of action by virtue of his agreement with Lowenfeld and plaintiffs.

In Woodbury v. Andrew Jergens Co., 69 F.2d 49 (2d Cir.1934) (L.Hand), the court held that, under a plaintiffs agreement to pay one third of the amount recovered from the defendant to his attorneys, one of whom was not an attorney of record, the attorney who was not of record “became by the law of New York an equitable assignee of the cause of action pro tanto,” though he had no charging lien for fees. Id. at 50. Moreover, the court held that “it can scarcely be that an equitable assignee is in a weaker position than a statutory lienor, who is often referred to as such an assignee.” Id. Thus, the distinction between an “attorney of record” and one who is “of counsel” may be of little practical significance in cases where attorneys have agreed among themselves to share in the fruits of their combined labor. As noted in In re Coleman, 87 F.2d 753 (2d Cir.1937), “[w]hile the statute refers to the ‘attorney who appears for a party,’ and it has therefore been said that the attorney of record alone is entitled to a lien, it has been held by the lower state courts that the lien may be transferred or assigned.” Id. at 754.

*453Similarly, in Harwood v. La Grange, 137 N.Y. 538, 540, 32 N.E. 1000 (1893), plaintiff, who had been employed as an attorney to assist as counsel in the prosecution of several cases on a contingency basis, was held to be the holder of an “equitable lien” of the amount received for such services. The court there stated that “[i]t was not important for the plaintiff to show that the agreement was made directly with the plaintiffs in those actions. It was sufficient that he was employed under the agreement made with [co-counsel], who acted, in making it, with the authority of [the clients], and on their behalf.” Id. The court held that it was “not open to doubt” that the “agreement gave the plaintiff an equitable lien on or ownership as equitable assignee in the proceeds of the action” and that it had “been so determined in many cases.” Id.

5. Failure of district court to administer Daniel’s charging lien under Section 475

Because Daniel possessed a charging lien under Section 475 by operation of law, the district court’s task would have been simply to fix the amount of the lien under the retainers as found by the court. The fact that Daniel may have been accorded a measure of justice in this case under the Copyright Act under 17 U.S.C. § 505 does not change the result here. Section 475 “has provided a lien in all cases, and not merely where the client fails to provide some other form of security or protection, and the courts cannot themselves substitute another form of protection for that provided in the statute.” Robinson v. Rogers, 237 N.Y. 467, 472, 143 N.E. 647 (1924). If the amount of the charging hen has been fixed by agreement, as here, execution is appropriate on the judgment for the amount agreed to by the parties. Machcinski v. Lehigh Valley R. Co., 272 F. 920, 922 (2d Cir.1921).

CONCLUSION

Appellants were entitled to have their contract fee claims considered and decided under the district court’s supplemental jurisdiction and the appellant attorney was entitled to the district court’s determination of a statutory hen under New York Judiciary Law § 475.

Reversed for proceedings consistent with the foregoing under 28 U.S.C. § 1367 and remanded for proceedings pursuant to New York Judiciary Law § 475.

7.5.5 Supplemental Jurisdiction Problems 7.5.5 Supplemental Jurisdiction Problems

     Take a look at these hypotheticals and see if you can get to the correct response about whether there is supplemental jurisdiction:

     Pam Plaintiff (CA) brings a state law claim against Doris Defendo (NV).  The claim is for $80,000. She wishes to add a related a claim against Ima Sued (WA) for $30,000, arising from the same transaction. 

     Pam Plaintiff (CA) brings a federal law claim against Doris Defendo (NV). The claim is for $25,000. She wishes to add a related claim against Amy Adverse (CA), seeking $25,000.

     Pam Plaintiff (CA) brings a state law claim against Doris Defendo (NV). The claim is for $80,000. She wishes to add a related claim against Amy Adverse (CA), seeking $125,000.

     Pam Plaintiff (CA) brings a state law claim against Doris Defendo (NV). Defendo wishes to bring a counterclaim against Plaintiff for $25,000, arising from a different, unrelated transaction.

     Pam Plaintiff (CA) brings a state law claim against Doris Defendo (NV) seeking $50,000, and includes in the same lawsuit a federal law claim for $25,000.

     Pam Plaintiff (CA) brings a state law claim against Doris Defendo (NV) seeking $80,000 and includes in the same lawsuit a federal law claim for $25,000. She wishes to add to the lawsuit a state law claim against Ima Sued (CA), arising from the same facts, seeking $25,000.

     Remember that the court has discretion with regard to whether supplemental jurisdiction should be extended. The following case deals with that issue.

     Limited LLC was created under the laws of Delaware and is headquartered in New York. Its members (owners) are from New Jersey, Pennsylvania, and Connecticut. It wishes to bring a lawsuit against BigCo, Inc., a Delaware corporation headquartered in New York. The claim is a state law contract claim for $76,000. Limited LLC would like to add another state law claim - this one for fraud - against Dennis Defendo, a major shareholder of BigCo. The claim is related to the claim against BigCo, alleging that Defendo committed fraud in connection with the Limited - Defendo agreement. This claim is for $70,000.  Please analyze.

     Limited LLC was created under the laws of Delaware and is headquartered in New York. Its members (owners) are from New Jersey, Pennsylvania, and Connecticut.  It brings a federal law claim for $100,000 against Ima Sued, a resident of New Jersey. To the same litigation it adds a state law claim against Sued for $40,000. It then adds an additional state law claim against Sued's cousin and business partner, Bennie Sued, of Connecticut, and yet another claim against Ima Sued's friend, Denise Defendo, of New York. All of these claims arise from the same transaction as the original federal claim. Bennie Sued then brings two counterclaims arising under state law against Limited, both seeking $30,000. The first is related to the original federal claim and arises from the same transaction. The second is completely unrelated, arises from a separate business deal, and is for $50,000.  Denise Defendo also brings two state law claims against Limited. One is for $10,000, and is related to the original federal claim. One is for $70,000, and is completely unrelated to any of the other claims in the lawsuit. Please analyze the subject matter jurisdiction issues.

7.6 Removal To Federal Court 7.6 Removal To Federal Court

7.6.1 General Removal Rule - 28 U.S.C. § 1441 - Actions removable generally 7.6.1 General Removal Rule - 28 U.S.C. § 1441 - Actions removable generally

(a) Generally.—

Except as otherwise expressly provided by Act of Congress, any civil action brought in a State court of which the district courts of the United States have original jurisdiction, may be removed by the defendant or the defendants, to the district court of the United States for the district and division embracing the place where such action is pending.

(b) Removal Based on Diversity of Citizenship.—

     (1) In determining whether a civil action is removable on the basis of the jurisdiction under section 1332(a) of this title, the citizenship of defendants sued under fictitious names shall be disregarded.

     (2) A civil action otherwise removable solely on the basis of the jurisdiction under section 1332(a) of this title may not be removed if any of the parties in interest properly joined and served as defendants is a citizen of the State in which such action is brought.

(c) Joinder of Federal Law Claims and State Law Claims.—

     (1) If a civil action includes—

          (A) a claim arising under the Constitution, laws, or treaties of the United States (within the meaning of section 1331 of this title), and

          (B) a claim not within the original or supplemental jurisdiction of the district court or a claim that has been made nonremovable by statute,

the entire action may be removed if the action would be removable without the inclusion of the claim described in subparagraph (B).

     (2) Upon removal of an action described in paragraph (1), the district court shall sever from the action all claims described in paragraph (1)(B) and shall remand the severed claims to the State court from which the action was removed. Only defendants against whom a claim described in paragraph (1)(A) has been asserted are required to join in or consent to the removal under paragraph (1).

(d) Actions Against Foreign States.—

Any civil action brought in a State court against a foreign state as defined in section 1603(a) of this title may be removed by the foreign state to the district court of the United States for the district and division embracing the place where such action is pending. Upon removal the action shall be tried by the court without jury. Where removal is based upon this subsection, the time limitations of section 1446(b) of this chapter may be enlarged at any time for cause shown.

(e) Multiparty, Multiforum Jurisdiction.—

     (1) Notwithstanding the provisions of subsection (b) of this section, a defendant in a civil action in a State court may remove the action to the district court of the United States for the district and division embracing the place where the action is pending if—

          (A) the action could have been brought in a United States district court under section 1369 of this title; or

          (B) the defendant is a party to an action which is or could have been brought, in whole or in part, under section 1369 in a United States district court and arises from the same accident as the action in State court, even if the action to be removed could not have been brought in a district court as an original matter.

The removal of an action under this subsection shall be made in accordance with section 1446 of this title, except that a notice of removal may also be filed before trial of the action in State court within 30 days after the date on which the defendant first becomes a party to an action under section 1369 in a United States district court that arises from the same accident as the action in State court, or at a later time with leave of the district court.

     (2) Whenever an action is removed under this subsection and the district court to which it is removed or transferred under section 1407(j) [1] has made a liability determination requiring further proceedings as to damages, the district court shall remand the action to the State court from which it had been removed for the determination of damages, unless the court finds that, for the convenience of parties and witnesses and in the interest of justice, the action should be retained for the determination of damages.

     (3) Any remand under paragraph (2) shall not be effective until 60 days after the district court has issued an order determining liability and has certified its intention to remand the removed action for the determination of damages. An appeal with respect to the liability determination of the district court may be taken during that 60-day period to the court of appeals with appellate jurisdiction over the district court. In the event a party files such an appeal, the remand shall not be effective until the appeal has been finally disposed of. Once the remand has become effective, the liability determination shall not be subject to further review by appeal or otherwise.

     (4) Any decision under this subsection concerning remand for the determination of damages shall not be reviewable by appeal or otherwise.

     (5) An action removed under this subsection shall be deemed to be an action under section 1369 and an action in which jurisdiction is based on section 1369 of this title for purposes of this section and sections 1407, 1697, and 1785 of this title.

     (6) Nothing in this subsection shall restrict the authority of the district court to transfer or dismiss an action on the ground of inconvenient forum.

(f) Derivative Removal Jurisdiction.—

The court to which a civil action is removed under this section is not precluded from hearing and determining any claim in such civil action because the State court from which such civil action is removed did not have jurisdiction over that claim.

7.6.2 28 U.S. Code § 1446 - Procedure for removal of civil action 7.6.2 28 U.S. Code § 1446 - Procedure for removal of civil action

(a) Generally.--A defendant or defendants desiring to remove any civil action from a State court shall file in the district court of the United States for the district and division within which such action is pending a notice of removal signed pursuant to Rule 11 of the Federal Rules of Civil Procedure and containing a short and plain statement of the grounds for removal, together with a copy of all process, pleadings, and orders served upon such defendant or defendants in such action.
(b) Requirements; generally.--

     (1) The notice of removal of a civil action or proceeding shall be filed within 30 days after the receipt by the defendant, through service or otherwise, of a copy of the initial pleading setting forth the claim for relief upon which such action or proceeding is based, or within 30 days after the service of summons upon the defendant if such initial pleading has then been filed in court and is not required to be served on the defendant, whichever period is shorter.
     (2)

           (A) When a civil action is removed solely under section 1441(a), all defendants who have been properly joined and served must join in or consent to the removal of the action.
           (B) Each defendant shall have 30 days after receipt by or service on that defendant of the initial pleading or summons described in paragraph (1) to file the notice of removal.
          (C) If defendants are served at different times, and a later-served defendant files a notice of removal, any earlier-served defendant may consent to the removal even though that earlier-served defendant did not previously initiate or consent to removal.
     (3) Except as provided in subsection (c), if the case stated by the initial pleading is not removable, a notice of removal may be filed within 30 days after receipt by the defendant, through service or otherwise, of a copy of an amended pleading, motion, order or other paper from which it may first be ascertained that the case is one which is or has become removable.
(c) Requirements; removal based on diversity of citizenship.--

     (1) A case may not be removed under subsection (b)(3) on the basis of jurisdiction conferred by section 1332 more than 1 year after commencement of the action, unless the district court finds that the plaintiff has acted in bad faith in order to prevent a defendant from removing the action.
     (2) If removal of a civil action is sought on the basis of the jurisdiction conferred by section 1332(a), the sum demanded in good faith in the initial pleading shall be deemed to be the amount in controversy, except that--
          (A) the notice of removal may assert the amount in controversy if the initial pleading seeks--
               (i) nonmonetary relief; or
               (ii) a money judgment, but the State practice either does not permit demand for a specific sum or permits recovery of damages in excess of the amount demanded; and
          (B) removal of the action is proper on the basis of an amount in controversy asserted under subparagraph (A) if the district court finds, by the preponderance of the evidence, that the amount in controversy exceeds the amount specified in section 1332(a).
     (3)

          (A) If the case stated by the initial pleading is not removable solely because the amount in controversy does not exceed the amount specified in section 1332(a), information relating to the amount in controversy in the record of the State proceeding, or in responses to discovery, shall be treated as an “other paper” under subsection (b)(3).
          (B) If the notice of removal is filed more than 1 year after commencement of the action and the district court finds that the plaintiff deliberately failed to disclose the actual amount in controversy to prevent removal, that finding shall be deemed bad faith under paragraph (1).
(d) Notice to adverse parties and State court.--Promptly after the filing of such notice of removal of a civil action the defendant or defendants shall give written notice thereof to all adverse parties and shall file a copy of the notice with the clerk of such State court, which shall effect the removal and the State court shall proceed no further unless and until the case is remanded.
(e) Counterclaim in 337 proceeding.--With respect to any counterclaim removed to a district court pursuant to section 337(c) of the Tariff Act of 1930, the district court shall resolve such counterclaim in the same manner as an original complaint under the Federal Rules of Civil Procedure, except that the payment of a filing fee shall not be required in such cases and the counterclaim shall relate back to the date of the original complaint in the proceeding before the International Trade Commission under section 337 of that Act.
[(f) Redesignated (e)]
(g) Where the civil action or criminal prosecution that is removable under section 1442(a) is a proceeding in which a judicial order for testimony or documents is sought or issued or sought to be enforced, the 30-day requirement of subsection (b) of this section and paragraph (1) of section 1455(b) is satisfied if the person or entity desiring to remove the proceeding files the notice of removal not later than 30 days after receiving, through service, notice of any such proceeding.

7.6.3 Background For Removal - Concurrent Jurisdiction of State and Federal Courts 7.6.3 Background For Removal - Concurrent Jurisdiction of State and Federal Courts

     In general, courts defer to a plaintiff’s choice of forum. One of the advantages that comes from being a plaintiff is that you get to choose the location and the court where you file, subject to limitations such as venue and personal jurisdiction, and courts consistently express a reluctance to disturb that choice. Because of concurrent jurisdiction, almost all cases that can be heard in federal court can also be heard in state court, and even if federal subject matter jurisdiction exists the choice in the first instance is the plaintiff’s to make.

     That said, there are occasions where the plaintiff’s choice is overruled, and removal is one of those situations. Removal provides a way for a defendant to remove a case for which federal subject matter jurisdiction exists to federal court. The Constitutional requirement that there be federal subject matter jurisdiction continues; in addition, the requirements of the removal statutes must be met.

     1. Federal Subject Matter Jurisdiction Must Exist. A necessary but not sufficient element for removal is that federal subject matter jurisdiction exists. If there is no federal subject matter jurisdiction, there can be no removal.

     2. All Defendants Must Join. In a multidefendant case, what happens when some defendants want to remove the case to federal court and some are happy to stay in state court? The case stays in state court. Removal only is allowed when all defendants served in the case join in the notice of removal. Chicago, Rock Island, & Pacific Railway Co. v. Martin, 178 U.S. 245 (1900).

     3. Diversity Cases Can Be Different. The statute treats differently cases whose only basis for federal subject matter jurisdiction is diversity. In such a case, if any of the defendants – just one out of a 1,000 will do – is from the forum state, the case is not removable. That said, courts have been known to look closely at defendants whose nominal inclusion seems to be solely for the purpose of frustrating removal, and responding appropriately.

     4. Procedure and Deadlines Matter.  To remove a case, a notice of removal is filed not with the state court, but with the federal district court in the location where the state court is located (you can’t pick a different geographic location; you must go to the nearest federal courthouse). The time to remove starts when the defendant is served in a case where federal subject matter jurisdiction could exist, and the defendant has only 30 days to file the notice. In some cases, as a case develops claims will be added or parties dropped in a way that creates federal subject matter jurisdiction, and in that case it may be possible to remove more than 30 days after filing – but in no case more than a year after the case begins. If a plaintiff voluntarily nonsuits a defendant from the forum state 13 months after filing, creating a case that would have been removable under the statute, tough beans – it’s too late.

     5. It Does Not Work in Reverse.  There is no procedure for sending a case from federal court to state court. If a case is filed in federal court but there is no federal subject matter jurisdiction, it must be dismissed and, if possible, refiled. 

     6. Plaintiffs May Not Remove. Imagine a situation in which a plaintiff files a case in state court for which there is no federal subject matter jurisdiction. The defendant counterclaims and asserts a claim for which there is federal subject matter jurisdiction. Can the plaintiff now remove? No. Even though effectively a defendant on this claim, the plaintiff does not satisfy the statute.

     7. Appeal from Removal Decisions By The Trial Court is Difficult At Best.  If a federal court chooses not to accept a case that has been noticed for removal (the technical term is to remand it), there is no case in federal court from which any kind of review can be had. If it accepts the case, that will not be a final decision, and, as you recall from our study of appeals, that means that the normal process of appeal is a ways off. 

     8. Plaintiffs Can Manipulate Removal But Within Limits.  A plaintiff can structure a case so as to reduce the chance of removal. For example, in a diversity case the plaintiff can join a defendant from the forum state or add a defendant that eliminates complete diversity (as you remember, WWVW was motivated in large part by whether defendants who would defeat diversity could stay in the case).  The amount sought in damages can also be kept below the jurisdictional limit. Certain claims can be held back and added later, if state law would permit such amendments. That said, judges are generally not fools, and will disregard fraudulent joinder or excessively ‘artful’ pleading in considering a removal petition.

     9. Other Removal Statutes. While Section 1441 is the main removal statute, there are others. Section 1442 deals with federal officers and agencies. The Class Action Fairness Act, which we will cover briefly when we get to class actions, provides a special method for removing certain kinds of class actions. Always remember that this course surveys U.S. Civil Procedure but we have neither the time nor the inclination to cover every single procedural device that might matter in practice.

     10. Remand By Defendant. Once having removed successfully, can a defendant move to remand? In Avitts v. Amoco Production Co., 53 F.3d 690 (1995) the defendant removed a somewhat vague complaint that only referred to 'federal law.' Plaintiff subsequently amended the complaint in federal court, asserting only state law claims. The federal court entered an order requiring the defendant to conduct an expensive environmental study, at which point the defendant moved to remand the case to state court. On interlocutory appeal, the Court of Appeals found that a federal question had never been successfully stated, and that so the case should be remanded, thereby vacating all relief entered by the federal court. Note that if a federal claim had been at some point successfully stated, the result might not have been the same, as federal jurisdiction would have attached.

7.6.4 Spencer v. United States District Court 7.6.4 Spencer v. United States District Court

Richard R. SPENCER, Executor of the Estate of Lindsay C. Spencer and Carmen West, Guardian Ad Litem for Wyley Spencer a minor, Petitioner, v. UNITED STATES DISTRICT COURT FOR the NORTHERN DISTRICT OF CALIFORNIA, Respondent, Altec Industries, Inc., and Does 1-30, jointly and severally, Real Party in Interest.

No. 04-72409.

United States Court of Appeals, Ninth Circuit.

Argued and Submitted Nov. 3, 2004.

Filed Dec. 14, 2004.

Michael J. Haddad, Oakland, CA, for the petitioner.

Jeffrey R. Williams, Bita A. Karabian, Morgenstein & Jubelirer LLP, San Francisco, CA, for the real party in interest.

*868Before: REINHARDT, THOMPSON, and BERZON, Circuit Judges.

DAVID R. THOMPSON, Senior Circuit Judge:

Petitioners seek a writ of mandamus ordering the district court to remand this action to state court. Petitioners argue the district court must remand pursuant to a bankruptcy court’s order which petitioners claim requires the district court to abstain from exercising federal jurisdiction. In the alternative, petitioners contend that the joinder of a local, albeit diverse,- defendant following removal from state to federal court, destroyed subject-matter jurisdiction, requiring remand. See. 28 U.S.C. §§ 1441(b), 1447(c). Because we conclude that the district court did not clearly err in determining that the bankruptcy court’s order does not require the district court to - abstain from exercising federal jurisdiction, and because we find no error in the district court’s determination that federal diversity jurisdiction is not' destroyed by the joinder of a local, diverse defendant subsequent to removal, we deny the petition for a writ of mandamus.

I.

Lindsay C. Spencer, an electrical lineman, died as a result of injuries he sustained while working in an aerial lift bucket to repair and upgrade a Pacific Gas & Electric Company (“PG & E”) utility pole. According to the petitioners, the operating controls of the lift bucket were unintentionally activated, causing the lift mechanism and the bucket to move suddenly and forcefully into the adjacent utility pole, injuring Mr. Spencer. The aerial lift truck then catapulted Mr. Spencer into the air, throwing him against a high voltage wire, causing his death by electrocution.

Mr. Spencer’s son and estate brought the present wrongful death action in the superior court in California, alleging state law product liability claims against the manufacturer of the lift bucket, Altec Industries, and several Doe defendants.

Altec timely removed the case to the United States District Court for the Northern District of California on the basis of federal diversity jurisdiction. The plaintiffs are resident citizens of Alaska, and Altec asserts it is a citizen of Alabama. There is no dispute that the parties are diverse and that the required statutory amount in controversy is satisfied.

During discovery in the district court, the Spencers learned that possible negligence by PG & E may have caused or contributed to activating the lift bucket controls. They then moved to amend their complaint to name PG & -E as a defendant in the place of one of the Doe defendants. The Spencers concurrently moved to remand the action to state court, arguing that remand would be required due to the joinder of PG & E. Specifically, the Spenc-ers contended that because PG & E is a citizen of California for purposes of diversity jurisdiction, and because 28 U.S.C. § 1441(b) prohibits removal from state to federal court when at least one defendant is a citizen of the state in which the action is filed, the joinder of PG & E would destroy federal removal jurisdiction and require remand under 28 U.S.C. § 1447(c).

At the time the Spencers sought to join PG & E as a defendant, PG & E was the Debtor in Chapter 11 bankruptcy proceedings in the Northern District of California. Accordingly, before the proposed joinder of PG & E could proceed, the Spencers *869had to obtain relief from the automatic stay imposed by 11 U.S.C. § 362(a). The Spencers, and PG & E by joint stipulation, obtained that relief by order of the bankruptcy court. The bankruptcy court order modified the automatic stay to permit join-der of PG & E “as a defendant in the State Court Action.”

Relying on the language of the bankruptcy court’s order, the Spencers supplemented their argument for remand to state court, contending that the order permitted their action to proceed exclusively in state court and therefore required the district court to abstain from exercising federal jurisdiction.

The district court granted the Spencers’ motion to join PG & E as a defendant, but denied their motion to remand the action to state court. The district court rejected the notion that the bankruptcy court’s order limited federal court non-bankruptcy jurisdiction. The district court concluded that the bankruptcy court’s order was limited to lifting the automatic stay and did not require abstention. The district court also rejected the Spencers’ contention that the § 1441(b) “forum defendant” rule, which limits federal removal jurisdiction, required remand to state court. The district court determined that the “forum defendant” rule is procedural rather than jurisdictional, and thus the addition of a local defendant did not require remand so long as removal was proper at the time the case was removed to federal court. The district court declined to certify its order for interlocutory appeal, and the petitioners then filed this petition for a writ of mandamus.

II.

"The remedy of mandamus is a drastic one, to be involved only in extraordinary situations.” Kerr v. United States Dist. Court, 426 U.S. 394, 402, 96 S.Ct. 2119, 48 L.Ed.2d 725 (1976). The writ of mandamus “has traditionally been used in the federal courts only ‘to confine an inferior court to a lawful exercise of its prescribed jurisdiction or to compel it to exercise its authority when it is its duty to do so.’ ” Id. (quoting Will v. United States, 389 U.S. 90, 95, 88 S.Ct. 269, 19 L.Ed.2d 305 (1967)) (quoting Roche v. Evaporated Milk Ass’n, 319 U.S. 21, 26, 63 S.Ct. 938, 87 L.Ed. 1185 (1943)). Because of the exceptional and extraordinary nature of mandamus, we have developed a five-factor test for evaluating the propriety of mandamus:

(1) The party seeking the writ has no other adequate means, such as a direct appeal, to attain the relief he or she desires.
(2) The petitioner will be damaged or prejudiced in a way not correctable on appeal.
(3) The district court’s order is clearly erroneous as a matter of law.
(4) The district court’s order is an oft-repeated error, or manifests a persistent disregard of the federal rules.
(5) The district court’s order raises new and important problems, or issues of law of first impression.

See Bauman v. United States Dist. Court, 557 F.2d 650, 654-55 (9th Cir.1977).

Of the foregoing five Bauman factors, we have stated “ ‘it is clear that the third factor, the existence of clear error as a matter of law, is dispositive.’ ” Calderon v. United States Dist. Court, 134 F.3d 981, 984 (9th Cir.1998) (quoting Executive Software N. Am., Inc. v. United States Dist. Court, 24 F.3d 1545, 1551 (9th Cir.1994)). Given the dispositive nature of the third Bauman factor, we consider that factor first.

*870A.

Applying the third Bauman factor to the petitioners’ first asserted ground for relief, we conclude the district court did not commit clear error in its determination that the bankruptcy court’s order lifting the automatic stay did not purport to require the district court to abstain from exercising otherwise proper federal jurisdiction.

The bankruptcy-court’s order lifting the automatic stay provides, in pertinent part, that the stay is lifted for the “limited purpose of allowing [the Spencers] to add PG & E as a defendant in the State Court Action and for the parties to litigate the State Court Action to final judgment in the Superior Court of the State of California.” In re Pacific Gas & Elec., No. 01-30923 (Bankr.N.D.Ca. Dec. 28, 2003). As the district court noted, the terms of the order do not reflect an intention that the district court abstain from exercising its otherwise proper jurisdiction. - Even assuming (without deciding) that the bankruptcy court could require such abstention, the order simply reflects the bankruptcy court’s intention, pursuant to the stipulation of the parties, to lift the automatic stay to permit the litigation to proceed.

We conclude the district court did not clearly err as a matter of law in determining that the bankruptcy court’s order did not limit the district court’s jurisdiction. Thus, the petitioners have failed to satisfy the third Bauman factor with regard to the first ground on which they predicate their application for a writ of mandamus.

B.

We next consider the petitioners’ contention that the district court should have remanded the case to state court because, once PG & E was added as a defendant, the district court lost subject-matter jurisdiction.

A civil action brought in • a state court over which federal Courts have original jurisdiction may be removed by the defendant to the appropriate district court. 28 U.S.C. § 1441(a). However, § 1441(b) imposes a limitation on actions removed pursuant to diversity jurisdiction: “such action[s] shall be removable only if none of the parties in-interest properly joined-and served as defendants, is a citizen of the State in which such action is brought.” 28 U.S.C. § 1441(b). This “forum defendant” rule “reflects the belief that [federal] diversity jurisdiction is unnecessary because there is less reason to fear state court prejudice against the defendants if one or more of them is from the forum state.” Erwin Chemerinsky, Federal Jurisdiction § 5.5, at 345 (4th ed.2003).

I-t is thus clear that the presence of,a local defendant at the time removal is sought bars removal. 28 U.S.C. § 1441(b).

What is less clear is whether the joinder of a local, but completely diverse defendant, after an action has been removed to federal court, requires remand. This is the question we confront in this case. The district court concluded it was not required to remand the case to state court, and we agree.1

*871Challenges to removal jurisdiction require an inquiry into the circumstances at the time the notice of removal is filed. When removal is proper at that time, subsequent events, at least those that do not destroy original subject-matter jurisdiction, do not require remand. See, e.g., Van Meter v. State Farm, Fire & Cas. Co., 1 F.3d 445, 450 (6th Cir.1993) (characterizing removal jurisdiction as “necessarily tied to a temporal reference point, namely the time of removal”); In re Shell Oil, 966 F.2d 1130, 1133 (7th Cir.1992) (stating that nothing in the text or legislative history of § 1447(c) alters the “traditional view” that “jurisdiction present at the time a suit is filed or removed is unaffected by subsequent acts”).

Because the joinder of PG & E did not affect the propriety of the district court’s original subject-matter jurisdiction, we need not decide whether an event occurring subsequent to removal which would defeat original subject-matter jurisdiction divests a district court of jurisdiction and requires remand. Compare Van Meter, 1 F.3d at 450, Shell Oil, 966 F.3d at 1133, and Poore v. Am.-Amicable Life Ins. Co. of Tex., 218 F.3d 1287 (11th Cir.2000) (events subsequent to removal do not divest a district court of subject-matter jurisdiction), with Mayes v. Rapoport, 198 F.3d 457, 461-63 (4th Cir.1999), Cobb v. Delta Exps., Inc., 186 F.3d 675, 677 (5th Cir. 1999), and Casas Office Mach., Inc. v. Mita Copystar Am., Inc., 42 F.3d 668, 673-75 (1st Cir.1995) (events subsequent to removal which destroy federal subject-matter jurisdiction require remand).

We conclude that the post-removal join-der of PG & E, a “forum defendant,” did not oust the district court of subject-matter jurisdiction. The forum defendant rule of 28 U.S.C. § 1441(b) is only applicable at the time a notice of removal is filed. Because no local defendant was a party to the action at that time, and given the preservation of complete diversity of the parties thereafter, the district court did not err in denying the Spencers’ motion to remand.2 As stated above, we do not decide what the result would be if PG & E were a non-diverse defendant.

Petition for mandamus DENIED.

7.6.5 Removal Problems 7.6.5 Removal Problems

     Perry Plaintiff (NJ) sues Donnie Defendant (NY)  and Ima Sued (CT) in New York state court. Plaintiff asserts a state law claim but, like the state law claim in Grable, it requires interpretation of a federal statute. This claim is for $50,000 against each defendant. There also is a pure state law claim, unrelated, for $50,000, again against each defendant. Sued and Defendant then quickly bring federal question counterclaims against Plaintiff for $150,000 each. Both defendants then join in a timely notice of removal. Please analyze the removal issues.

     Connie Claimant (AZ) brings a claim against Tommy Target (NY) and Bill Yuns (VT) in state court in New York. She asserts state law claims against each for $100,000 each. The state law claims require interpretation of a federal statute. Both defendants join in seeking removal. Please analyze. 

      

7.7 Conclusion of Subject Matter Jurisdiction 7.7 Conclusion of Subject Matter Jurisdiction

7.7.3 Suggested Approaches to Subject Matter Jurisdiction Questions 7.7.3 Suggested Approaches to Subject Matter Jurisdiction Questions

When faced with a complicated subject matter jurisdiction (and I promise you will be), remember that you need to establish subject matter jurisdiction for each count. Remember that subject matter jurisdiction alone does not guarantee removability. For each count this might be helpful:

·        Is there federal question jurisdiction?

·        If not, is there diversity/alienage jurisdiction?

·        If not, is there supplemental jurisdiction?  

·        If filed in state court, is the case removable?

7.7.4 Problems on Federal Subject Matter Jurisdiction 7.7.4 Problems on Federal Subject Matter Jurisdiction

     Perry Plaintiff (NY) has sued Don Defendant (NJ) and Denise Defendo (CT)  in state court in New York. Against Defendant, he brings a state law contract claim on which he seeks $30,000, a related federal claim on which he seeks $20,000, and a completely unrelated state law claim on which he seeks $30.000. Against Defendo, he brings a state law contract claim related to the state law contract claim against Defendant, on which he seeks $40,000. Both Defendant and Defendo file proper and timely notices of removal. Please analyze.,

     Perry Plaintiff (NY) files a state law claim against Don Defendant (NJ) in state court in Delaware, alleging breach of contract under which Defendants was supposed to make repairs on Plaintiff's home in New York. He seeks $100,000. Defendant files a notice of removal with the federal court  in New York. Please analyze.

     Perry Plaintiff (NY) files a federal law claim against Samuel Sued (NY) in federal court in New York. He asserts a federal law claim on which he seeks $50,000 and a state law claim arising from the same cluster of operative facts on which he seeks $50,000.  Sued counterclaims against Plaintiff for $50,000 on a state law claim, asserting claims that again arise from the same cluster of operative facts. He also brings another counterclaim against Plaintiff on a federal law claim on completely unrelated facts, seeking $30,000. On this claim he adds Terry Third  as another third party (counterclaim) defendant, seeking $30,000 on a state law claim arising from the same cluster of operative facts as his $30,000 federal law claim against Plaintiff. At the time of the incident, Third was a resident of Connecticut but moved to New York one day before Plaintiff filed his lawsuit.  He has kept his Connecticut driver's license, however, as well as his voter registration in Connecticut, and has told friends he is in New York indefinitely until he finishes a work assignment. Third adds two counterclaims of his own back against Sued, the first of which seeks $40,000 on state law claims arising from the same facts and circumstances as the claim on which Sued asserted against him. He adds a $40,000 counterclaim against Sued that arises from utterly unrelated facts. Please analyze.